You are on page 1of 232

QATAR

PROMETRIC
PRACTICE
FOR
NURSES
1- An alert oriented 77 year-old woman lives in a long-term care nursing home although she is
usuallybusy participating in the coordinated activities of the nursing home, the nurse finds
her sitting alone in the corridor. Various tests are ordered including a urine analysis (see X-
ray).
Blood pressure 128/74 mmHg
Heart rate 98/min

Respiratory rate 26/min


Body temperature 38.3 °C oral
Which additional clinical finding would most likely be an indicator of the early onset of her
problem?
a- Low urea concentration
b- Acute confusion
c- Increased blood pressure
d- Respiratory rate > 22/minute

2- A home care nurse changes the dressing of a patient who has a chronic lower leg ulcer and a
history of leg swelling. The wound bed is 3 cm diameter, red in color and is covered with
fibrousyellow tissue (see image)
What type of ulcer is most likely?
a- Arterial
b- Venous
c- Decubitus
d- Neuropathic
1
3- According to information provided in the accompanying graphic, a transverse colostomy would be
expected to produce:
a- Fluid feces
b- Mushy feces
c- Semi-fluid feces
d- Solid feces

4- According to information provided in the accompanying graphic. A descending colon would be


expected to produce
a. Fluid feces
b. Mushy faces
c. Semi-solid faces
d. Solid faces

5- What is the name of The process of movement of a dissolved substance (solutes) from an area of
high concentration to an area of low concentration:
a- Effusion
b- Diffusion
c- Emulsion
d- Excretion

2
Percentage of children with chronic illnesses

6- Which of the following statements is most accurate?


a- Girls have more ADHD than asthma
b- Boys are more frequently diagnosed with chronic illnesses
c- Boys are less frequently diagnosed with chronic illnesses
d- Girls have more other chronic illnesses than ADHD

7- What is the percentage of boy with attention deficit hyperactivity disorder?


a- 12%
b- 16%
c- 8%
d- 4%

3
8- A 70 year-old quadriplegic man has ulceration on the left buttocks with full thickness skin loss.
Although several interventions have been used to promote healing, the ulcer does not improve (see
image)
Which dietary insufficiency is most likely?
a- Vitamin D
b- Calcium
c- Protein
d- Vitamin C

9- A home care nurse is instructing parents to perform chest Physiotherapy (CPT) on their child with
cystic fibrosis the nurse has the parents place their child in a prone position with the body tilted
downward before performing CPT.
Based on this information, the nurse is instructing the parents to remove drainage from which of
the following Jung lobe segment?
a. Upper posterior
b. Lower anterior
c. Lower posterior
d. Upper anterior

4
10- A patient is on oxygen using a nasal cannula at home. The doctor has ordered this patient to
receive oxygen delivered at a concentration of 35 %. The nurse would set the oxygen at:
a- 2L
b- 3L
c- 4L
d- 5L

11- A patient has a blood glucose level of 60mg/dl. The patient is to receive 15 g of carbohydrate
if the blood glucose is less than 70mg/dl.
How many candies should the patient receive?
a- 4 candies
b- 6 candies
c- 8 candies
d- 10 candies

12- A patient has a blood glucose level of 60mg/dl. The patient is to receive 15 g of carbohydrate
if the blood glucose is less than 70mg/dl.
How many mls of orange juice should the patient receive?
a- 120 ml
b- 90 ml
c- 60 ml
d- 30 ml

5
13- On assessing the client’s pressure ulcer (see photo), the nurse would document this as which
stage:
a- Stage I
b- Stage II
c- Stage III
d- Stage IV

14- A 22 year-old man presented to the Mental Health Clinic with a low-mood, a general loss of
interest in activities and inability to experience pleasure. He admitted to suicidal thoughts and
extreme lack of energy. He was prescribed a selective serotonin reuptake inhibitor to be taken
daily. One month later, he presented to the clinic and reports feeling more energetic, but still has a
low-mood.
What is the patient’s level of risk committing suicide at this time?
a- None
b- Low
c- Medium
d- High

15- A patient requiring long-term antibiotics has a central line catheter inserted into the right
subclavian vein by the physician.
Which of the following must be verified prior to the first use of the catheter?
a- Blood return
b- X-ray
c- Catheter patency
d- Length of catheter

16- A 35 year-old woman with excessive facial and body hair and acne has had weight gain in the
shoulder, upper back and face. She does not drink alcohol and does not have diabetes. She takes
glucocorticoids regularly. Which dietary plan would be most beneficial for this patient?
a- Increased calcium
b- Decreased sodium
c- Increased potassium
d- Decreased protein

6
17- A 45 year-old woman presents to the clinic with urinary incontinence. Point-to-point testing, in
which she must touch the tip of her nose with the doctor’s index finger on command, reveals
slowness and uncertainty. Alternate hand movement testing showed asymmetry, slowness and
increased concentration. Rhythmic, involuntary movements of the hands are noted at rest. During
Romberg’s test, the patient easily loses balance.
Which type of urinary incontinence treatment is most appropriate?
a- Bladder retraining and anti-cholinergic agents
b- Timed voiding and biofeedback training
c- Intermittent straight catheter and biofeedback
d- Timed voiding and anti-cholinergic agents

18- A nurse is caring for an eight year-old male with cystic fibrosis.

Based on the nurse’s understanding of the disease what nursing intervention should the nurse expect
to perform?
a- Restrict sodium and fluid intake
b- Give antidiarrheal medications
c- Discourage coughing after postural drainage
d- Administer pancreatic enzymes with each meal

19- A patient hospitalized with tuberculosis (TB) has a productive cough and hemoptysis.
Which of the following types of isolation room would be the BEST choice for the patient?
a- Reverse isolation
b- Standard isolation
c- Positive-pressure
d- Negative-pressure

20- A 78 year-old man presents to his care provider with persistent abdominal and back pain. He also
occasionally has numbness and pain in the legs. On examination a palpable mass in the lower
abdominal midline is palpated. On auscultation a blowing sound is heard over his mass. The nurse
is concerned about possible rupture.
Which additional finding most significantly indicates the patient is at an increased risk?
a- Hypertension
b- High blood sugar
c- Constipation
d- Decreased oxygen saturation

7
21- When administrating an enema to an adult patient, how far should the nurse insert the tubing
into the rectum?
a- 2.2 to 4.4 cm (1 to 2 inches)
b- 4.4 to 6.6 cm (2 to 3 inches)
c- 6.6 to 8.8 cm (3 to 4 inches)
d- 8.8 to 11 cm (4 to 5 inches)

22- A nurse plans to teach a group of 20 to 25 year-old women about oral contraceptives. The nurse
should instruct that oral contraceptives may:
a- Increase risk of pelvic inflammatory disease
b- Cause acne to worsen
c- Decrease the risk of breast and cervical cancer
d- Decrease the risk of endometriosis

23- A 35 year-old man twisted his right ankle. Since then, he has had swelling and stiffness of the
ankle in the morning hours. He presented to the clinic six months after the injury and complains
that he is in too much pain to participate in sports. An arthroscopy reveals thickening and
inflammation of the lining of the ankle joint.
Which treatment is most likely to be ordered for this client?
a- Aspiration of fluids
b- Surgical intervention
c- Physical therapy
d- NSAID administration

24- An eight year-old boy presents with abdominal cramping and a feeling of fullness. He frequently
passes semi-formed stools that are light yellow colored and oily, with an abnormally foul odor. His
mother states that the child has lost his appetite lately and is concerned about weight loss.
Endoscopic examination reveals villous atrophy.
Which food would be most suitable for this child?
a- Rye
b- Oats
c- Barley
d- Wheat

25- Following an open-cholecystectomy, the nurse would instruct the patient to expect to resume
normal activities in:
a- 1 to 2 weeks
b- 2 to 3 weeks
c- 4 to 6 weeks
d- 6 to 8 weeks

8
26- A three year-old child is day three on the postoperative care unit following a heart transplant. He
is prescribed cyclosporine by intravenous infusion.
Blood pressure 140/80 mmHg
Heart rate 100/min
Respiratory rate 22/m
Temperature 37.0 °C
Oxygen saturation 92% on room air
Which vital sign most likely indicates an adverse medication effect?
a- Hypertension
b- Decreased O2 saturation
c- Tachycardia
d- Tachypnea

27- A 6-month-old premature infant is being discharged from the neonatal intensive care unit after
having no apneic periods for the past week. Although the child has had no apneic periods for the
past week, an apnea monitor will be sent home with the parents. The BEST short term goal
relating to the nursing diagnosis of ineffective breathing pattern related to apnea is that the:
a- Apnea monitor will sound loudly and stimulate the child to breath with episodes of apnea
b- Apnea monitor will sound loudly and alert the parents to stimulate the child to breath with
episodes of apnea
c- Child will have no episodes of apnea or cyanosis while on apnea monitor
d- Child will sleep in their parent’s bed each night until there is no apnea two weeks

28- A 25 year-old female presented to the emergency room with lethargy, decreased reflexes,
hypoventilation, hypotension, and fixed and dilated pupils. A family member who is
accompanying the patient has an empty bottle of diazepam (Valium) which the label states was
recently refilled. The family member also indicates that the patient has a history of depression.
What intervention should the nurse expect to administer?
a- Naloxone (Narcan)
b- Activated charcoal
c- Magnesium sulfate to reduce the risk of seizures
d- A tap water enema

29- An 82 year-old patient has Parkinson’s disease. During the assessment, the nurse would expect
which of the following actions to produce the MOST tremor activity of the hands?
a- Eating with a fork
b- Resting hands in lap
c- Standing with hands loose at sides
d- Rolling a small pill between the fingers
9
30- A nurse is documenting in patient records several events that occurred during home visits.
Which of the following is an example of the correct way to document patient information?

a- Patient fell walking to bathroom. Busy preparing for sterile dressing change when patient left the
room
b- Patient got out of bed without assistance. Denies any symptoms when ambulating alone
c- Patient sitting in chair. Strict bedrest orders ignored
d- Patient showering. Electronic epidural infusion pump on floor by shower stall

31- A 50 year-old man is hospitalized after presenting with long-term fatigue, weight gain and
hematemesis. On examination he is jaundiced and the abdomen distended and tense, and there is
shifting dullness present. Following treatment, the nurse provides discharge instructions.
Which instructions would be most appropriate?
a- Decrease daily iron intake
b- Maintain constant bed rest
c- Consume a low protein diet
d- Restrict fluid intake

32- The nurse is assessing an infant with Gastroeosophageal reflux disease (GERD). To help identify
any complications of GERD, what is the most important question the nurse should ask the infant’s
parents?
a- Frequency of respiratory tract infection
b- Frequency of vomiting
c- Time between bowl movements
d- Time between feeds

33- A patient with an unsteady gait and a history of falls has care plan interventions that include
keeping the walker in reach and pathways free of obstacles. On evaluation after 1 week, the
patient has had no falls, but the gait remains unsteady. The nurse should:
a- Continue the plan of care as written
b- Allow the patient to replace the walker with a cane
c- Allow the patient to ambulate short distances without the walker
d- Have the patient practice stepping over small objects

34- A patient who is preparing for hip surgery has an order for external pneumatic compression
devices. The nurse teaches the patient that pneumatic compression can help prevent:
a- Upper respiratory infection
b- Decreased breath sounds
c- Deep vein thrombosis
d- Bleeding at the surgical site

10
35- A patient complains of severe menstrual cramping. Bleeding is not usually heavy, and the patient
has no uterine disorders.
Which of the following interventions should the nurse anticipate the doctor will order to promote
comfort?
a- Acetaminophen (Tylenol)
b- Strict bed rest
c- Heating pad to the back of neck
d- Ibuprofen (Motrin)

36- A patient is admitted to the emergency department with stable angina. The physician writes an
order for nitroglycerin 1/150 grains sublingually (SL).
Which is the correct dose in milligrams (mg)?
a- 0.04 mg
b- 0.4 mg
c- 4 mg
d- 40 mg

37- A perimenopausal woman with a small body frame is at high risk for a condition in which bone
resorption exceeds the rate of bone formation. She drinks one-three cups of coffee per day since
she was 25 years old and smokes five-ten cigarettes per day for the past 20 years. The nurse
advises the patient to increase her dietary intake of milk and green leafy vegetables.
Which additional foods are most appropriate to recommend?
a- Egg yolk, cheese
b- Peanuts, poultry
c- Whole grains, cereals
d- Lentils and black beans

38- A 30 year-old woman with type 1 diabetes mellitus receives mixed type of insulin in the morning
and before bed time. She reports that the level of her fasting blood sugar is constantly high when
she checks it every morning at home.
Which dose of insulin is most likely causing this problem?
a- Low morning, regular insulin
b- High morning NPH1
c- High evening regular insulin
d- Low evening NPH insulin

11
39- A five month-old boy has been vomiting green colored vomit for ten hours. He has intermittent
abdominal pain during which he draws his legs up to his chest, turns pale and cries forcefully. On
observation, there is bleeding in the stool which has a jelly-like consistency. Abdominal palpation
reveals a long tube-like mass. There is no fever, rash nor diarrhea. Bowel sounds are hyperactive
in all quadrants.
Which is the most likely form of initial treatment?
a- Manual manipulation
b- Surgical resection
c- Barium enema
d- Endoscopy

40- The nurse is assisting a patient to ambulate in hall. The patient has a history of coronary artery
disease (CAD) and had coronary artery bypass graft surgery (CABG) 3 days ago. The patient
reports chest pain rated 3 on a scale of 0 (no pain) to 10 (severe pain). The nurse should first:
a- Determine how long it has been since the patient’s last dose of aspirin
b- Obtain a chair for the patient to sit down
c- Assess the patient’s radial pulse
d- Ask the patient to take several slow, deep breaths.

41- A 52 year-old man with congestive heart failure presents to the Emergency Department with
rapid and irregular heartbeats, and feeling dizzy and light-headed. The attending physician
verbally calls out an order to the nurse to administer digoxin 0.25 milligrams by intravenous
injection. How will the nurse complete the paperwork for this order?
a- Write, sign and repeat order back to the physician
b- Ask the physician to write and sign the order
c- Write the order and ask the physician to sign
d- Verbally repeat the order and administer drug

42- A 67 year-old man was admitted to the hospital following a closed bone fracture. An
intramedullary nail is inserted and the patient is placed in balanced skeletal traction. The
following day, the patient becomes restless, drowsy and confused, he has difficulty breathing and
appears very tired.
Which additional sign or symptom would require immediate intervention?
a- Anxiety
b- Cold skin
c- Constipation
d- Petechiae on chest

12
43- A patient is preparing for a total knee replacement. During the preoperative interview process the
patient reports an allergic reaction to penicillin.
Which of the following is considered a side effect and not a true allergy to medication?
a- Shortness of breath
b- Tingling lips and tongue
c- Rash
d- Upset stomach

44- A 54 year-old woman presented to the Emergency Department with sharp upper-right abdominal
pain that radiates to the right scapula. While performing the admission assessment, the patient
becomes nauseous and begins vomiting. She states that she has had pain in the upper right
quadrant previously but that this time it was far worse. There’s a positive Murphy’s sign and an
ultrasound confirms gallbladder wall thickening and pericholecystic fluid collections. Which of
the following would most likely be associated with her clinical findings?
a- Relief by drinking milk
b- Alleviation with exercise
c- Triggered by fatty meal
d- Worsening on empty stomach

45- A 68 year-old woman is receiving parenteral nutrition at home. The district nurse visits the
woman and notes that she has gained one kilogram of weight since a health provider had visited
one week ago. There is pitting edema of 2+ of the lower extremities. The patient is alert, active and
oriented.
Which nursing diagnosis is most appropriate?
a- Non-compliance
b- Impaired gas exchange
c- Imbalanced nutrition
d- Fluid volume overload

46- A patient has a central line catheter and is receiving a three-in-one total parenteral nutrition that
contains glucose, proteins and lipids. The pump is set to deliver the infusion over a 12-hour period.
After how many hours should the intravenous administration set be changed?
a- 12
b- 24
c- 48
d- 72

13
47- A 45 year-old man who is hospitalized feels the constant need to keep things in order, particularly
whilst eating. The nurse observes him arranging the food on his plate into symmetrical and equal
bite-sized pieces. He constantly worries that food served could be outdated and potentially cause
illness.
Which nursing diagnosis is most important?
a- Ineffective verbal communication
b- Self-esteem disturbance
c- Impaired social interaction
d- Anxiety

48- A 12 year- old patient had a cast removed from the left leg after wearing if for eight weeks. The
patient wants to resume sports as soon as possible. In order to regain muscle strength lost while
wearing cast, the nurse will instruct the patient in performance of:
a- Resistive range of motion exercises to left leg
b- Passive range of motion exercises to right leg
c- Active- assistive range of motion exercises to the right leg
d- Active range of motion exercises to both legs

49- A three year-old has returned to the clinic 4 days after being diagnosed with gastroenteritis and
dehydration. A parent reports that the vomiting has stopped, and the child is tolerating liquids,
rice, applesauce, and bananas. The diarrhea persists, but seems to be decreasing in volume. When
evaluating for signs of dehydration, the nurse will assess the patient’s skin turgor by:
a- Grasping the skin over the abdomen with two fingers and raising the skin with two fingers
b- Grasping the skin over the forehead with two fingers and raising the skin with two fingers
c- Holding the patient’s mouth open and assessing the tongue for deep creases or furrows
d- Drawing two tubes of blood and running a blood urea nitrogen (BUN) and creatinine (Cr)

50- A patient has had hepatitis B (HBV) and is now a chronic carrier. In planning care, the nurse
would explain an HBV carrier would most likely be at risk for developing a super infection with
which other type of hepatitis?
a- A c- C
b- B d- D

51- A female patient has been advised that laboratory tests confirm herpes simplex virus (HSV), type
2. The nurse should teach the patient that a Papanicolaou test (Pap smear) is recommended:
a- Every 6 months if symptoms persist despite treatment
b- Every year even if asymptomatic
c- Whenever symptoms recur
d- Every 3 years if other Pap smears have been negative

14
52- A 30 year-old married man presents to the clinic with complaints of feeling sad for the past three
months. He is unable to maintain a regular sleep routine, has lost his appetite and has difficulty
concentrating. He is prescribed a medication which prevents the reuptake of specific
neurotransmitters that could contribute to his mental health problem. Which side effects would be
most important for the nurse to advise the patient of?
a- Polyuria
b- Photophobia
c- Fluid retention
d- Sexual dysfunction

53- A patient presents with pneumonia and dyspnea. The patient has an ineffective airway clearance
related to excessive tracheobronchial secretions.
Which of the following interventions would the nurse implement to enhance the patient’s airway
clearance?
Heart rate 80/min
Respiratory rate 32/min
Temperature 40 °C
a- Administer oxygen as ordered
b- Maintain a comfortable position
c- Increase fluid intake
d- Administer prescribed analgesics

54- A nurse evaluates a patient who had a modified radical mastectomy two weeks ago.
Which of the following observations would indicate that the patient is progressing as expected?
a- Incision intact; numbness along suture line
b- Affected arm supported in sling; hand at level of the heart
c- Reports cessation of intimate contact with spouse
d- Indicates plan to replace wardress with loose fitting clothing

55- A one month old boy present with the head tilted towards the left side and the chin rotated to the
right side. There is a palpable mass of soft tissue on the right side of the neck near the clavicle:
a- Passive muscle stretching
b- Surgical release of the muscle
c- Surgical removal of the mass
d- It’s a normal mass in infants

15
56- A patient with bowlegs due to abnormal bone formations and deformities has calcium level of 7.5
mg/100ml.
Which of the following foods would the nurse most likely instruct the patient to add to a diet?
a- Organ meats
b- Whole grains
c- Egg yolks
d- Chicken meat

57- A patient is one day post-operative repair of a large umbilical hernia. The patient complaints of
abdominal pain and describe feeling the sutures give way. Upon assessment of the abdomen the
nurse observes an evisceration. The nurse’s IMMEDIATE response should be to:
a- Medicate the patient for pain
b- Instruct the patient to cough hard
c- Have the patient perform the Valsalva maneuver
d- Cover the abdomen with a sterile saline spiked dressing

58- While reviewing stress management techniques with a patient diagnosed with multiple sclerosis,
what would the nurse identify as MOST appropriate?
a- Relaxing in a warm bubble bath
b- Yoga in a cool room
c- Sunbathing
d- Cross-country running

59- A four year – old boy presented with a firm and non- tender mass that his mother had discovered
in the right upper abdominal quadrant He was admitted to the hospital and underwent complete
surgical excision of a renal tumor that had extended beyond the kidney. The child was then
scheduled for radiation and chemotherapy treatment.
Which stage was the tumor?
a- I
b- II
c- III
d- IV

60- When caring for a patient with an ostomy the nurse knows that extra skin protection for the
personal skin is MOST important for those patients with a (an):
a- Ileostomy
b- Ascending colostomy
c- Transverse colostomy
d- Sigmoid colostomy

16
61- The nurse is assessing the growth and development of a healthy three year-old child. The nurse
should expect the child to be able to:
a- Ride a bicycle
b- Jump rope
c- Throw a ball overhead
d- Hop on one foot

62- The nurse is monitoring a patient receiving parenteral nutrition via a central line catheter for the
24 hours. The patient has polyuria and complaint thirst and headache.
Blood pressure 120/76 mmHg
Heart rate 88/min
Respiratory rate 16/mm
Temperature 37.1°C
Oxygen Saturation 99% an room air
What will the nurse most likely administers?

a- Insulin
b- Dextrose 5% in water
c- Normal saline
d- Calcium gluconate

63- A 22 year-old man suffered major injuries to the right leg in a road traffic accident that resulted
in an above –the- knee amputation with a rigid dressing. Twelve hours later the patient complains
of increasing pressure from the dressing and the feeling that something is crawling around the
stump. What would be the first nursing action?
a- Notify the physician
b- Remove and reapply the dressing
c- Remind patient phantom limb pain is normal
d- Elevate and apply ice to the residual limb

64- A patient with congestive heart failure and severe peripheral edema has a nursing diagnosis of
fluid volume excess
What are the two MOST important interventions for the nurse to initiate?
a- Diuretic therapy and intake and output
b- Nutritional education and low-sodium diet
c- Daily weights and intake and output
d- Low-sodium diet and elevate legs when in bed

17
65- A nurse is providing care to a patient with a new skin graft on the leg. The patient is upset and the
nurse notes copious red drainage oozing around the dressing the nurse should immediately:
a- Lift the dressing to assess the area
b- Ask if the patient is having any pain
c- Apply firm pressure for 10 to 15 minutes
d- Assess the apical pulse

66- A 25 years-old male patient suffered a spinal cord injury at the T-4 level and is being cared for in
hospital. The nurse enters the patient’s room and finds the patient sitting upright and looking
anxious and restless. He complains of sudden headache and nausea. Sweat forms on his forehead
yet his feet are cool to touch
Blood pressure 150/100 mmhg
Heart rate 55/min
Respiratory rate 28/min
Temperature 37.1C
What nursing intervention is initially most appropriate?
a- Assess for a full bladder
b- Lower the head of the bed 30 degrees
c- Loosen clothing and bed sheets
d- Apply heating pad to lower extremities

67- To minimize a toddler from scratching and picking at healing skin graft, the nurse should utilize:
a- Mild sedatives c- Punishment for picking
b- Hand mittens d- Distractions

68- A 30 year-old diabetic woman complains of blood in the urine and dull pain over the left lower
abdomen, middle back and above the pubic bone. She feels the urge to urinate frequently and
urgently. For the past two days, she has had nausea and vomiting. Abdominal assessment reveals
tenderness over the pubic bone; no guarding and bowel sounds are active in all quadrants.
Which home intervention would be most effective?
a- Oral antibiotics
b- Increased fluid intake
c- Increased dietary protein
d- Cold application to left hand

69- A patient recently underwent coronary artery graft (CABG) surgery.


Which of the following nursing diagnose PRIORITY?
a- Anxiety
b- Impaired gas exchange
c- Acute pain
d- Sleep deprivation

18
70- The nurse calls together an interdisciplinary team with members from medicine, social service,
the clergy, and nutritional services to care for a patient with a terminal illness.
Which of the following types of care would the team MOST likely be providing?
a- Palliative
b- Curative
c- Respite
d- Preventive

71- Pulse Volume Scale


Scale Description of pulse
0 Absent pulse
1 Weak and Thready pulse
2 Normal
3 Bounding pulse
A 26 year- old female patient’s radial pulse is assessed for a full minute. The nurse notes the pulse
is difficult to palpate and count.
What should the nurse chart?
Blood pressure 82/48 mmHg
Respiratory rate 20/min
Temperature 38.8°C
a- 0 c- 2
b- 1 d- 3

72- A child with asthma has an order for albuterol, before administration of the medication the nurse
MUST.
a- Pre-oxygenate the patient
b- Assess the patient’s heart rate
c- Obtain venous Access
d- Feed the patient a snack

73- A 52 year- old woman is scheduled to undergo an abdomino- perineal resection in three days for
removal of a cancer of the rectum. The nurse reviews the care plan with the patient. The patient
will receive prophylactic antibiotics and will be given a mechanical bowel preparation the day
before.
Which additional preparation should the patient undertake at this time?
a- Wear pressure stockings
b- Perform leg strengthening exercises
c- Maintain high- protein, low- residue diet
d- Take daily ferrous iron tablets

19
74- A 32 year-old female has an adrenocortical hormone disorder. The nurse notes the patient has
thin scalp hair, a large trunk and thin extremities.
Blood pressure 152/84 mmHg
Heart rate 64/min
Respiratory rate 16/min
Temperature 37.2°C
Oxygen Saturation 98% on room air
Which clinical findings are most likely?
a- Decreased bowel sounds and muscle soreness
b- Cardiac arrhythmias and abdominal cramping
c- Headache, confusion and muscle twitching
d- Hunger, trembling and nervousness

75- A 62 year-old man is diagnosed with localized non cell tumor of the lung without metastasis and
under lobectomy. After the procedure, he is brought to Postoperative Anesthesia Care Unit for
recovery with chest tube drainage system in place and administered oxygen by nasal cannula.
Which nursing action is initially most important?
a- Assess level of consciousness
b- Attach chest tubes to the bed sheets
c- Perform deep breathing and coughing
d- Apply elastic stockings to the legs

76- A 67 year-old man is admitted to the Post-anesthesia Recovery unit following chest surgery. The
patient has a right chest tube that is attached to low suction. Three hours after admission to the
unit, the nurse observes the drainage output from the chest tube is 300 milliliters.
What is the most appropriate initial intervention?
a- Notify the doctor
b- Reduce IV infusion rate
c- Strip tube with roller device
d- Re-position in left lateral decubitus

77- An elderly patient with severe degenerative joint comes to the clinic for routine follow up of
management. The patient reports that over the month, the pain has begun to increase in severity
patient requests an increase in dosage of the medication. The nurse recognizes that this is most
likely due to?
a- Drug addiction
b- Drug tolerance
c- An improvement in condition
d- Lack of efficacy of the current medication

20
78- A community health nurse visits a patient who has suffered a stroke. The patient’s spouse
explains to the nurse that the patient chokes while eating some times.
Which of the following referral orders would the nurse anticipate needing for this patient?
a- Speech therapist
b- Dietician
c- Physical therapist
d- Neurologist

79- A patient is transferred to the Intensive Care following a craniotomy. The patient is difficult to
arouse and the pupils are pinpoint and non-reactive.
Blood pressure 118/70 mmHg
Heart rate 58/min
Respiratory rate 11/min
Temperature 37.2°C
Which medication should the nurse prepare administer?
a- Adrenaline
b- Thiamine
c- Naloxone
d- Dextrose 50%

80- The nurse has been assigned to care for a 60 year old critically ill patient with a triple-lumen
central venous line. The doctor’s orders include daily care of the insertion site and catheter device.
The nurse creates care plane based on the nursing diagnosis, Risk for infection related to insertion
of a venous catheter.
Which intervention is most likely to prevent infection?
a- Re-cap access hub after drawing blood
b- Maintain clean technique
c- Wash hands before performing catheter care
d- Clean catheter tubing with isopropyl alcohol

81- A patient who underwent a right knee arthroplasty two days ago has a nursing diagnosis of
impaired mobile. The patient refuses to get out of bed and ambulate due chest pain.
Which of the following action would the nurse MOST likely implement?
a- Medicate the patient prior to ambulation
b- Add a nursing diagnosis of non-compliance
c- Let the patient rest now and then try to ambulate later
d- Assess to determine the cause of the chest pain

21
82- Prior to providing care for a hospitalized infant the nurse who focuses on preventive measures
must.
a- Introduce self-parent
b- Perform hand hygiene
c- Have a witness present
d- Assess the child’s developmental level

83- The nurse is inserting a nasogastric (NG) tube into patient as prescribed. The nurse has advanced
the tube into the patient’s posterior pharynx. The nurse should now ask the patient to?
a- Hold the breath
b- Stare upwards with the eyes towards the ceiling
c- Perform the Valsalva maneuver
d- Lower the chin towards the chest

84- What would be the long-term management goal for a patient with diabetes mellitus, type II?
a- Quickly lower blood glucose level by administering insulin
b- Achieve hypoglycemic state
c- Maintain daily exercise
d- Understand how to self-administer msulax

85- The nurse administered a dose of morphine sulfate, as prescribed to a patient who is in the post-
anesthesia care unit (PACU). The patient appears to be resting comfortably; the respiratory rate
is 8 and the 𝑶𝟐 saturation on 2L of oxygen via nasal cannula is 86%. The nurse should
IMMEDIETLY administer.
a- Flumazenil (Romazicon)
b- Midazolam (Versed)
c- Naloxone (Narcan)
d- Ondansetron (Zofran)

86- A patient is 90-minutes post-abdominal hysterectomy and complains of pain in the post anesthesia
care unit (PACU). The nurse administers the prescribed dose unit intravenous morphine. Five
minutes later, the patient is difficult to arouse, with the following vital signs.
What should the nurse do NEXT?
Respiratory rate 6/min
Saturation 86% on room air
a- Document the efficacy of analgesic therapy
b- Administer naloxone (Narcan)
c- Place in supine position
d- Check the surgical site for bleeding

22
87- The stage of dying as identified by Dr. Elizabeth Kubler-Ross occurs in what order?
a- Anger, depression, bargaining, denial, acceptance
b- Bargaining, denial, Anger, acceptance, depression
c- Denial, Anger, bargaining, depression, acceptance
d- Depression, denial, Anger, bargaining, acceptance

88- A 42 year- old patient is in a lower body cast following a motor vehicle accident. In order to
minimize muscle strength loss while in the cast, the nurse will instruct the patient in the
performance of?
a- Isometric exercises
b- Passive range of motion exercises
c- Active-assistive range of motion exercises
d- Resistive range of motion exercises

89- A patient has a history of severe, uncontrolled epistaxis. The patient’s blood pressure and platelet
count are normal. To minimize the occurrence of bleeding episodes the nurse should teach the
patient to.
a- Sleep with the head elevated on at least two to three pillows
b- Apply firm pressure to the nostrils four times a day
c- Apply a water- soluble lubricant to the nasal septum twice daily
d- Minimize the intake of caffeine and increase fluids intake

90- A six year-old patient has been diagnosed with acute rheumatic fever. The nurse knows that the
antibiotic of choice for this illness is:
a- Benzathine penicillin (Megacillin)
b- Amoxicillin (Amoxil)
c- Erythromycin (Erythrosine)
d- Vancomycin (Vancocin)

91- A 20 year- old woman takes cyclosporine 100 mg by mouth twice per day. She had a heart
transplant two months ago and complains of dizziness, shortness of breath, light headedness and
fatigue. Capillary refill >3.
Blood pressure 90/50 mmHg
Heart rate, irregular 100/min
Respiratory rate 28/mm
Temperature 38.0°C
What is the most likely underlying cause of the patient’s problem?
a- Cyclosporine toxicity
b- Hyperkalemia
c- Organ rejection

23
d- Infection
92- A 23 year- old male comes to the Emergency Department in a sickle cell crisis. He reports that his
pain level is a 10/10 in all extremities. During the assessment, he cannot lie still because of the
pain. There is no cyanosis or clubbing in the extremities and all examination findings are normal.
The vital signs recorded were
Blood pressure 132/82 mmHg
Heart rate 110/min
Respiratory rate 18/mm
Temperature 38.4°C
Oxygen Saturation 94% an room air
Which nursing diagnosis is first priority?
a- Acute pain
b- Fluid volume deficit
c- Ineffective tissue perfusion
d- Ineffective airway clearance

93- Upon assessment of an obese patient’s back, the nurse observes a forward curvature of the
lumbar spine. These findings are indicative of.
a- Kyphosis
b- List
c- Lordosis
d- Scoliosis

94- A newborn was delivered pre-term weighing 2700 grams with. APGAR scores of 4 and 6,
respectively. When the mother had presented to the Obstetrical Triage Unit, she was already 7
centimeters dilated and fully effaced. Her due date was unknown as she had no parental care. The
infant showed signs of fetal distress and was finally delivered by Cesarean section. At birth a
large, thin, membranous sac was protruding from the umbilical base.
What is the priority nursing intervention at birth?
a- Maintain cardio respiratory stability
b- Protect the herniated viscera
c- Manage fluid intake and output
d- Establish vascular access

95- A surgeon instructs a nurse to serve as a witness to an elderly patient’s informed consent for
surgery. During the surgeon’s explanation to the patient, it becomes clear that the patient is
confused and does not understand the procedure, but reluctantly signs the consent from. The
nurse should.
a- Sign the form as a witness, making a notation that the patient did not appear to understand
b- Not sign the form as a witness and notify the nurse supervisor
c- Not sign the form and answer the patient’s questions after the surgeon leaves the room

24
96- A child is treated for superficial (first-degree) thermal burns to the thigh. The child is in great
discomfort and does not eat.
Which of the following diagnoses should receive PRIORITY?
a- Altered nutrition c- Risk for infection
b- Impaired skin integrity d- Acute pain

97- A patient is being prepared for a right breast biopsy under general anesthesia. The patient asks
the nurse about the surgical scar and possible postoperative complications.
Which of the following actions would be appropriate for the nurse to take?
a- Review the postoperative risks with the patient
b- Notify the surgeon about the patient’s questions
c- Compete the patient’s preoperative check list
d- Show the patient photos of breast surgical scar

98- A patient who is 4 days postoperative after a total hip replacement surgery, is obese and has not
been able to ambulate since the surgery. The patient is now diaphoretic, has chills, and complains
of pain in the thigh. There is tenderness over the anteromedial surface of the thigh. The MOST
likely cause is.
a- Wound infection
b- Deep vein thrombosis (DVT)
c- Pulmonary edema
d- Dehydration

99- A nurse is communicating with a moderately autistic child during a developmental and behavioral
assessment. The nurse uses non-verbal techniques to engage the child’s cooperation. She uses
facial expressions when responding and simple body language to communicate commends.
What is the most likely age of the child?
a- One c- Six
b- Three d- Nine

100- A 27 year-old woman presents with stomach cramping with alternating constipation and
diarrhea. She had been diagnosed with irritable bowel syndrome two years before and has so far
managed the illness with lifestyle changes, including diet and exercise.
What is the most appropriate advice?
a- Increase dairy intake
b- Use antacids to relive pain
c- Increase dietary fiber
d- Avoid emotional stress triggers

25
101- A nine year-old child has been diagnosed with external otitis. In addition to assessing severity of
symptoms and need for pain relief, the nurse should assess.
a- The hearing of the child
b- The tympanic temperature
c- Speech and language development
d- Ear hygiene and the need for earplugs

102- A 40 year-old woman is undergoing an elective rhinoplasty under general anesthesia. The patient
is in the pre-operative room and the nurse is prepared to administer pre-operative intravenous
medications. The patient states that she does not have any drug allergies.
Which additional nursing action is most important prior to administering the medicine?
a- Request the patient urinate
b- Perform blood typing and cross matching
c- Ensure the consent form has been signed
d- Clarify contact numbers of her family members

103- A 24 year-old man was admitted to the hospital after receiving a closed head injury in a motor
vehicle accident. He had a Glasgow Coma Score of 6 on admission and was being mechanically
ventilated. The nurse observes the intracranial pressure has slowly and steadily increased over the
past hour.
What is the most appropriate initial nursing action?
a- Administer mannitol
b- Administer Phenobarbital
c- Increase respiratory rate
d- Reposition while maintaining neck alignment
104- During the immediate postoperative period, a patient reveals an oxygen saturation level of 91%.
The nurse should
a- Position the patient on the left side
b- Administer supplemental oxygen
c- Continue to provide supportive care
d- Lower the temperature of the room

105- A patient who has a diagnosis of multiple sclerosis speaks slowly with long pauses between
syllables.
The patient is MOST likely in what stage of multiple sclerosis?
a- Prodromal
b- Early
c- Middle
d- Late

26
106- A 68 year-old man is admitted to the hospital with an exacerbation of chronic obstructive
pulmonary disorder. He has breathing difficulties, restlessness and anxiety. He also has a moist
and productive cough. The lower extremities are swollen with pitting edema 4+. A blood gas
specimen is collected and sent to the laboratory. The patient has not been on supplemental oxygen
therapy at home (see lab results)
Blood pressure 180/90 mmHg
Heart rate 90/min
Respiratory rate 28/mm
Body Temperature 37.1°C
Oxygen Saturation 86% an room air
Test Result Normal Values
ABG PC𝑶𝟐 7.33 4.7-6.0 kPa
PH 7.32 7.36-7.45
ABG P𝑶𝟐 7.73 10.6-14.2 kPa
What is the most likely percentage rate per liter for oxygen administration via nasal cannula for
this patient?
a- 0.5-1
b- 1.5-2
c- 2.5-3
d- 5-6
107- A 32 year-old man is brought to the Emergency Department by a friend with whom he had been
playing football with earlier that evening. His symptoms came on after twenty minutes of the
football match. He appears anxious and restless. Auscultation of the lungs reveals wheezing on
exhalation. The patient states that he is unable to get a full breath of air. He had this problem as a
child a couple of times after exercising in cold weather.
Blood pressure 126/72 mmHg
Heart rate 90/min
Respiratory rate 28/mm
Temperature 37.0°C
Oxygen Saturation 94% on room air
Which position is best for this patient?
a- Supine
b- Reverse Trendelenburg’s
c- Prone
d- High fowler’s

108- A nurse is caring for an infant with respiratory distress syndrome.


Which of the following nursing intervention is appropriate?
a- Measure oxygen saturation level once a shift
b- Suction frequently for 30-45 seconds each time
c- Monitor for symptoms of hyperglycemia
d- Maintain infant temperature 36.7° and 37.8°C

27
109- In evaluating a patient after a cerebrovascular accident (CVA), which of the following is an
expected outcome of the nursing diagnosis of impaired physical mobility related to motor deficits?
a- Oriented to person, lace, and time
b- Maintains body alignment, no contractures
c- Communicates appropriately
d- Voids on command at 2-hour intervals

110- The nurse is caring for a 4 year-old patient with a diagnosis of cystic fibrosis and pneumonia.
The child is feeling better on the 3rd day of the hospitalization and “wants to play”
What would be the BEST choice of entertainment?
a- Blowing bubbles
b- Looking at picture books
c- Watching videos
d- Riding in a wagon

111- An 83 year-old woman lives in a long term care facility. During the past two weeks, she has
suffered two falls that resulted in minor injuries and now complains of pain in the left shoulder.
On auscultation, bowel sounds are diminished in all four quadrants. On palpation, she has
tenderness and guarding over the upper left quadrant and complain of a feeling of fullness. What
could be the most likely underlying cause of the finding?
a- Constipation
b- Liver distention
c- Splenic rupture
d- Intestinal obstruction

112- In planning a lecture for a community group effect of acquired immunodeficiency syndrome
gerontological community, the nurse would be correct in including which of the following stat
regarding the incidence of the illness in people be 55 and 64 years of age?
a- The illness is almost unheard of in this age group prior to 2003
b- The illness has more than doubled between 1998 and 2003
c- There are not many people living with the disease as it quickly fatal
d- Treatment is more effective in this age group

113- A home care nurse visits a patient who is wheelchair bound due to a recent motor vehicle
accident. The patient has been sitting in the wheelchair for extended periods of time, which has
resulted in the development of a stage I pressure sore on the right buttocks.
What is the BEST nursing intervention?
a- Instruct the caretaker to change the patient’s position every 2 hours
b- Apply hydrogel to the stage I pressure sore every 8 hours
c- Refer the patient to a wound care specialist for debridement
d- Encourage the patient to consume an increased amount of calcium

28
114- A patient who sustained extensive abdominal injuries in a motor vehicle accident has developed
a large stage II pressure ulcer on the coccyx. A new diagnosis of alteration in skin integrity is
added to the care plan.
What is the BEST short-term goal for the patient?
a- Show evidence of healing within one week
b- Have no discomfort from the pressure ulcer
c- Eat at least 50% of each meal
d- Verbalize strategies to prevent further skin breakdown

115- A nursing home resident with chronic osteoarthritis complains of knee pain. A routine order of
acetaminophen (Tylenol) 500 mg every 6 hours was administered 2 hours ago with no relief. The
patient also has an order for ibuprofen (Motrin) 400 mg every 6 hours as needed. The patient
reports some relief, but is still bothered by mild pain. The nurse should
a- Administer ibuprofen (Motrin) 400mg
b- Educate the patient that mild pain is expected with osteoarthritis
c- Call the physician for additional pain medication
d- Administer acetaminophen (Tylenol) 500 mg

116- A 55 year-old man has become very anxious about skin lesions he has developed. On the lower
right forearm, there is a well demarcated round patch of skin that he feels could be cancerous. It is
2.5 centimeters in diameter and slightly raised. On palpation it is scaly, dry and rough. The
affected area appears sun tanned and reddened. The condition has been persistent for the past
four years but has only recently become itchy.
What is the most likely underlying problem?
a- Seborrheic keratosis
b- Actinic keratosis
c- Eczematous dermatitis
d- Lupus erythematosus

117- The nurse is performing a routine examination for a 24 year-old man with diabetes mellitus. He
takes insulin injections twice daily and monitors his blood sugar regularly. He is vegetarian with a
body mass index of 17. The nurse is concerned about his weight.
Which additional information is most likely required to calculate his ideal daily caloric intake?
a- Daily elimination pattern
b- Protein and carbohydrate intake
c- Activity and exercise level
d- HDL and LDL cholesterol levels

29
Patient Age Past Medical History Assessment of the normal aging process
A 70-years old Atrophic Vaginitis “Are you abstaining from sexual activity?”
B 76-years old Osteoporosis “Do you spend at least ten hours testing each day?”
C 81-years old Urinary incontinence “Are you limiting your daily fluid intake to 1000ml?”
D 88-years old Diverticulosis “Do you eat small frequent meals?”
118- A nurse is assessing several patients who are over the age of 65 years.
Which question would demonstrate the nurse’s understanding of changes associated with the
normal process of aging?
a- Patient A c- Patient C
b- Patient B d- Patient D

119- A 65 year-old woman with a history of unstable angina and hypertension presents to the
Emergency Department with a dull chest pain that she describes as similar to heartburn. The pain
radiates down the left arm. She had taken sublingual nitroglycerin tablets without any relief. An
electrocardiograph is performed and shows elevated S-T segments. The nurse is to administer a
thrombolytic by intravenous infusion.
Which factor places this candidate at high risk for bleeding?
a- Unstable angina
b- Hypertension
c- Age
d- Elevated S-T segments

BP Classification Systolic Blood Pressure Diastolic Blood Pressure


Normal Less than 120 More than 80
Prehypertension 120-139 80-89
Stage 1 hypertension 140-159 90-99
Stage 2 hypertension More than 160 More than 100
120- The following blood pressures have been documented in the patient’s chart for the past six
months; 124/76 mmHg, 136/83 mmHg and 128/80 mmHg. The Most likely diagnosis is:
a- Normal blood pressure
b- Prehypertension
c- Stage 1 hypertension
d- Stage 2 hypertension

121- A nurse is assessing a child with suspected lead poisoning.


For which of the following should the nurse assess?
a- Developmental delay, constipation, anorexia, clumsiness, and pallor
b- Confusion, delirium, seizures, lethargy, stupor, and coma
c- Respiratory depression, obstruction, pulmonary edema, and tachypnea

30
d- Peripheral circulatory collapse and heart failure
122- The following pain medications are ordered for a patient who had a right leg amputation.
Oxycodone 5 mg every 4 hours as needed and morphine 5 mg every 4 hours as needed. The nurse
administered oxycodone 2 hours ago, but the patient reports pain rated 8 on a scale of 0 (no pain)
to 10 (severe pain) as the dressing change beings.
After evaluating the effectiveness of the pain medication, what action should the nurse take?
Blood pressure 169/98 mmHg
Heart rate 112/min
Respiratory rate 22/mm
Temperature 36.7°C
a- Administer additional oxycodone 5 mg
b- Administer morphine 5 mg
c- Change the dressing quickly
d- Encourage deep breathing

123- A 72 year-old woman who is alert and oriented has been hospitalized with atrial fibrillation.
Since admission four days ago, she has been receiving digoxin intravenous injection. Today is the
first day the digoxin injection will be discontinued and she will change to oral tablets. Although
she acknowledges the medication benefits, she refuses to take it as it makes her feel nauseous.
What is the most appropriate nursing action?
a- Continue administering by intravenous push
b- Withhold medication and notify the physician
c- Withhold medication and file an incident report
d- Provide it in liquid from mixed into a drink

124- A patient has returned to the medical-surgical unit after surgery for a benign ovarian tumor.
The nurse observes that in the first 30 minutes after admission to the unit, the patient’s abdomen
is rapidly distending and the patient complains of increasing pain. The nurse would anticipate.
a- Aspiration pneumonia
b- Surgical site infection
c- Retention of surgical supplies
d- Intra-abdominal hemorrhage

125- The district nurse visits a 30 year-old woman at home following the delivery of her second child,
a full term girl. Following the delivery of her first child, she had developed a breast infection and
stopped breastfeeding because of the pain. She asks the nurse how she can best prevent it with this
infant.
What is the most appropriate response?
a- Provide feedings on demand
b- Apply vitamin E cream to the nipples
c- Request a prophylactic antibiotic

31
d- Apply heat to the breasts after feedings
126- The nurse caring for a mobile patient with a destroyed tympanic membrane and an intact stapes.
What type of repair process should the nurse anticipate?
a- Myringoplasty
b- Tymoanostapedopexy
c- Cavum minor
d- Tympanoplasty and fenestration of lateral semicircular canal
127- A nurse is caring for a postoperative patient who is on subcutaneous, low-dose heparin. When
administering an injection on the abdomen, the nurse avoids the umbilicus area because of the
possibility of.
a- Entering a larger blood vessel
b- Causing increased pain
c- Precipitating hyperventilation
d- Umbilical infection
Problems The patient will
A Altered tissue perfusion to hypertension Maintain blood pressure below 130/90 mm/Hs
B Fluid volume excess related in to lower Not have signs of edema or unexpected weight gain
extremity edema
C Knowledge deficit to process hypertension Verbalizes understanding of disease process
symptoms and medication management
D Alteration in comfort related chronic headaches Report an absence of headaches each week
128- A patient has come to the clinic for follow-up one week after being discharged from the hospital
for treatment of a hypertensive crisis. Blood pressure stabilized at 124/78 mm hg. The patient
report feeling well, has no edema, and no longer has daily headaches. Blood pressure is 156/90
mmHg. During evaluation the patient admits to having stopped taking medication that had been
ordered because headaches are no longer present. Unless the symptoms return, the patient states
they will not be returning to the clinic.
What should the nurse do?
a- Resolve and discontinue the entire care plan per patient request, suggest psychology consult
b- Resolve B and D, continue A and C
c- Continue entire care plan as written
d- Add a new problem to the care plan, non-compliance and interventions to determine potential

129- An 82 year-old woman with Alzheimer’s disease had moved into a long-term care facility two
weeks previously. Since then, the staff has found her wondering in the hallways in middle of the
night. When approached, she is confused and frustrated, often forgetting where she is.
Which intervention would most likely decrease the patient’s confusion?
a- Administer a sleeping sedative
b- Provide full-time nursing care

32
c- Place a nightlight in the room
d- Provide a large meal before bed
130- A 25 year-old woman resents to the clinic with abdominal pain. She reports the last bowel
movement was two days previously. There is no vomiting or diarrhea and her body temperature is
normal. During percussion of the lower left quadrant the nurse hears tympanic sounds.
What is the most likely underlying cause for the finding?
a- Gas
b- Tumor
c- Liquid
d- Feces

131- A 17 year-old is brought to the clinic by her mother. She complains of episodes during which her
fingers become cold, numb and white. After this subsides, the fingers turn red, swell and cause
severe and throbbing pain. On examination, the fingers all appear normal and the patient has no
pain. The nurse performs a history assessment to identify potential causes.
Which trigger is most likely?
a- Excessive exercise
b- Ingestion of tea
c- Exposure to cold
d- Washing up

132- A community is experiencing an outbreak of staphylococcal infections. The nurse instructs


residents that the MOST common mode of transmission is by:
a- Respiratory droplets c- Hands
b- Contaminated foods d- Soil

133- A patient on a medical floor is diagnosed with pneumonia.


Which of the following interventions is appropriate for a nursing diagnosis of ineffective airway
clearance related to tracheobronchial secretions secondary to infection?
a- Explain diagnostic procedures and address any of patient’s concerns
b- Assess patient for signs of discomfort and pain
c- Monitor oxygen therapy and encourage to cough and deep breathe
d- Monitor patient for shock

134- The nurse is inserting a nasogastric (NG) tube into a patient as prescribed. The nurse has
advanced the tube into the patient’s posterior pharynx. The nurse should now ask the patient to.
a- Hold the breath
b- Stare upwards with the eyes towards the ceiling
c- Perform the Valsalva maneuver
d- Lower the chin towards the chest

33
135- Your patient's finger stick blood sugar at 07:30 is 300 mg/dl, according to the following sliding
scale; how many unites you would give your patient:
a- 6 i.u
b- 8 i.u
c- 14 i.u
d- 18 i.u

136- The nurse is caring for a diabetic child whose 11 a.m. blood glucose monitoring check is 269
mg/dL. The physician orders the following coverage schedule:
150 to 200 mg/dL—2 units of Humulin R
201 to 250 mg/dL—4 units of Humulin R
251 to 300 mg/dL—6 units of Humulin R
301 to 350 mg/dL—8 units of Humulin R
351 to 399 mg/dL—10 units of Humulin R
Over 400 mg/dL—call the physician
According to the coverage schedule; how many unites you would give your patient:
a- 4 units
b- 6 units
c- 8 units
d- 10 units

34
137- A patient with bacterial pneumonia reports increased pain during deep breathing and coughing
exercises. To minimize the patient’s pain, the nurse should teach the patient to.
a- Take a cough suppressant prescribed P.R.N at regular time intervals
b- Turn onto the side before doing the respiratory exercises
c- Hold a pillow tightly against the chest while coughing
d- Drink warm liquids right before taking deep breaths

138- A 40 year-old woman present with painful leg cramps and muscle twitching, which wakens her
at night. Her menstrual cycles are irregular and cause painful abdominal cramping. The hair,
skin and nails appear dry and brittle (see lab results)
Test Result Normal Values
Calcium 2.09 2.15-2.62mmol/L
Phosphate, inorganic 1.69 0.82-1.51mmol/L
Magnesium 0.55 0.7-1.0mmol/L
What type of lifelong diet would be most beneficial for this patient?
a- Increased magnesium and vitamin C
b- Increased calcium and vitamin D
c- Increased dairy and iron
d- High protein and high calorie

139- A 30 year-old woman has been prescribed albuterol and prophylactic inhaled corticosteroids to
be taken once per day. She has been taking the prophylactic as prescribed but has needed to use
the albuterol more often than usual. She has a chronic cough and often has air hunger.
Which intervention is initially most appropriate?
a- Refer for a chest X-ray
b- Administer magnesium sulfate
c- Assess peak flow measurement
d- Perform arterial blood gas analysis

140- A first time mother of a three week-old breastfed baby brings the infant to the clinic and
complains that her child has been forcefully vomiting after feeding. He was born at 40 weeks
gestation, weighing 3.6 kilograms. He is constantly hungry and irritable. Examination reveals a
swollen abdomen and a palpable mass in the middle upper right quadrant.
What is the most likely health problem?
a- Intussusception
b- Pyloric stenosis
c- Gastroeosophageal reflux
d- Diaphragmatic hernia

35
141- A 40 year-old man present to the clinic with a weight loss of three kilograms over the past
month. His health problem produces moderate spastic body movements, hyper-reflexia, muscular
contractions followed by relaxation, and a positive Babinski reflex. He has difficulty speaking due
to poor muscle tone and has recently developed swallowing difficulties. The nurse creates a care
plan to address his developing problem.
Which is the priority nursing intervention?
a- Increase caloric intake
b- Aspiration prevention
c- Infection control
d- Improved muscle control

142- A 62 year-old diabetes type II patient is admitted to hospital for amputation of the right foot due
to gangrene and ischemia, Pre-operatively, the patient rates the pain as 9 (using the 1-10 pain
scale). How can this patient’s phantom (sprit) pain best be controlled?
a- Post-operative elevation of limp
b- Apply pressure bandage to stump
c- Control pain pre-operatively
d- Apply ice to the site for twenty minutes

143- The nurse cares for a 60 year-old woman has a history of hypertension, hypothyroidism and
elevated cholesterol levels. She takes tablets daily for each of the health problems. The doctor
orders a routine dual-energy X-ray absorptiometry test shows decreased bone density.
Which medication most likely contributed to the test results?
a- Statins
b- Anti-hypertensive
c- Synthetic thyroid hormones
d- Cholesterol absorption inhibitors

144- A 32 year-old woman with diabetes mellitus type 1 underwent a cholecystectomy and is now on
day two of recovery. The patient’s bowel sounds have returned and she has resumed a normal
diet but has been finishing less than half of each meal on the tray. The nurse enters the room to
perform a routine assessment and finds the patient confused and shaky
Blood pressure 110/60 mmHg
Heart rate 96/min
Respiratory rate 22/mm
Temperature 37.0°C
What is the most appropriate initial intervention?
a- Administer glucagon
b- Notify the physician
c- Given an insulin injection
d- Offer a glass of juice

36
145- A nurse is admitting a two year-old child with an umbilical hernia.
Which of the following interventions does NOT meet the child’s developmental needs?
a- Allowing the child to make choices when possible
b- Providing rooming in and unlimited visitation
c- Attempting to continue rituals used house
d- Maintaining strict bed rest

146- A woman who has been discharged from hospital with a permanent colostomy in lace is
performing irrigation at home for the first time. After five minutes from the start of the
procedure, she begins to have stomach cramps.
Which intervention would most likely decrease the symptoms?
a- Increase the flow of irrigation solution
b- Decrease the temperature of the water
c- Clamp the catheter for 1-2 minutes
d- Elevate the bag of irrigation solution

147- A patient has an acute inflammation of the gallbladder. The physician orders the nurse to
schedule the patient for surgery
Which of the following surgical procedures will the physician MOST likely perform?
a- Pancreatectomy
b- Cholecystectomy
c- Hepatectomy
d- Cricothoracotom

148- A patient with a history of chronic obstructive pulmonary disease prepares for a colonoscopy
with conserve sedation. Due to the patient’s history the nurse should very cautious about
administering.
a- Antibiotics
b- Sedation
c- Oxygen
d- Intravenous fluid

149- A 45 year-old woman is receiving chemotherapy for breast cancer. Two weeks after the initial
treatment she telephoned the nurse at the cancer center and reports she has hair loss, nausea,
tiredness, a body temperature is 38.1 C, and air hunger
Which finding most likely indicates she needs to report the clinic?
a- Pyrexia
b- Nausea
c- Hair loss

37
d- Air hunger
150- A patient present to the clinic voicing a concern about being exposed to hepatitis A (HAV) one
week ago. Upon questioning, the nurse finds the patient purchased for from a person recently
diagnosed with HAV. The number would be MOST correct when instructing the patient that:
a- The incubation period is 3 to 5 weeks
b- HAV is spread by sexual transmission
c- HAV is spread by blood contact
d- The incubation period is 2 to 5 months

151- A gravid 3, para 2 presents to the Labor and Delivery Unit with contractions occurring every 3-
5 minutes and lasting for 60-90 seconds. During contractions, she closed her eyes and is focused
on breathing to cope with the pain. At the end of each contraction she opens her eyes and resumes
talking with the nurse and her husband.
What is the expected cervical dilatation in centimeters?
a- 1-3
b- 4-7
c- 8-10
d- 11-13

152- The nurse is preparing a discharge plan for a 65 year-old man with a new diagnosis of congestive
heart failure. The discharge orders include furosemide 40 milligrams by mouth twice per day.
The nurse recommends foods to reduce unwanted medication side effects.
Which of the following foods would be most appropriate to include in the teaching plan?
a- Green and leafy vegetables
b- Bananas and oranges
c- Chicken and Fish

153- A patient who is scheduled for a tonsillectomy is in the preoperative unit. The nurse notes an
order for preanesthetic medication to be given “on call to operating room.” The nurse should
give this medication.
a- Immediately upon being notified to prepare the patient for transport
b- When the operating room staff arrives to transport the patient
c- Only if clearly needed after

154- A 76 year-old woman present with facial drooping on the left side and muscle weakness of the
left arm and leg after suffering a fall at home.
Which type of paresis is most likely?
a- Monoparesis
b- Paraparesis
c- Hemiparesis

38
d- Quadriparesis

Admission Report at 22.45(10.45 P.M) Diagnosis


14 years old Remain fibrils, maximum Temperature Bacterial
admitted 18 that shift 39.3°C(102.8°F)
hours ago Respiration rate 12-11per min
Blood pressure level within base line
normal range
Next dose of intravenous active
infective is done at 08.00(8.00a.m)
155- Refer to the accompanying figure.
Which of the following goals should receive PRIORITY during the shift?
a- The patient will rest for 6 hours without interruption
b- The patient will consume 90% of each meal
c- The patient will perform deep breathing and coughing exercises
d- The patient will drink 1920 ml (64 ounces of fluid)

156- A 45 year-old man has a family history of heart disease. His father died for heart disease at the
age of 61 and he is concerned that he is at high risk. He reports that he walks for twenty minutes
three times per week. He drinks three to five cup of coffee daily and does not smoke cigarettes.
His medical history includes diabetes, which is well controlled with to sit for long periods of time
(see lab result).
Blood pressure 138/88 mmHg
Heart rate 70/min
Respiratory rate 18/min
Temperature 37°C
Test Result Normal Values
Cholesterol (HDL) 1.0 >1.03mmol/L
Cholesterol (LDL) 4.0 <3.36mmol/L
Total Cholesterol 5.2 <5.1mmol/L
Triglycerides 2.2 <2.16mmol/L
Which lifestyle modification would most likely decrease this patient risk factor?
a- Decrease stress
b- Increase exercise

157- A 66 year-old woman is admitted to the hospital with a history hypertension. She present with
breathing difficulties that worsen with activity and while sleeping, she is generally weak and feels
that her heart misses beats and that it sometimes beats loudly. An electrocardiogram shows atrial
fibrillation, right ventricular hypertrophy and deviation towards the right.
Which nursing intervention is most appropriate for this patient?
a- Provide a bedside commode (portable toilet)
b- Place in right side lying position
c- Encourage family and friends to visit
39
d- Encourage independent hygienic activities

158- A 66 year-old woman is admitted to the hospital with a history hypertension. She present with
breathing difficulties that worsen with activity and while sleeping, she is generally weak and feels
that her heart misses beats An electrocardiogram Conform atrial fibrillation, right ventricular
hypertrophy and deviation towards the right
What was the most likely origin of the disorder?
a- Hypertension
b- Rheumatic fever
c- Atherosclerosis
d- Genetic predisposition

159- The nurse has administered an intravenous infusion of Cyclophosphamide to a 43 year-old


patient with breast cancer. The nurse formulates a care plan with the nursing problem of risk for
infection related to leucopenia secondary to chemotherapy. The patient is discharged home
following treatment.
Which findings should the patient report immediately?
a- Dyspnea
b- Tachycardia
c- Sore throat
d- Discoloration of urine

160- When given postoperative discharge instruction to a patient who had abdominal surgery, all of
the following regarding wound healing is true EXPECT
a- Bathing to soak abdomen is preferred over showering
b- Avoid tight belts and clothes with seams that may rub the wound
c- Pain medication may affect ability to drive
d- Irregular bowel habits can be expected

161- An eight year-old boy has a new diagnosis of Attention Deficit Hyperactive Disorder (ADHD)
and begins pharmacological and psychological therapies. His medication regimen includes short-
acting methylphenidate 5 mg orally every 12 hours.
Which side effect is most likely?
a- Insomnia
b- Initial increased hyperactivity
c- Lethargy
d- Hypertension

162- A brick-layer has been diagnosed with multiple sclerosis. The nurse knows that construction
work places has increased risk of aggravated symptoms because exposure to.
a- Dust

40
b- Sunlight
c- Heat
d- Heights
163- The nurse is caring for a full-term new born who was delivered vaginally 5 minutes ago. The
infant’s APGAR score was 8 at one minute and 10 at 5 minutes.
Which of the following has the highest PRIORITY?
a- Maintaining the infant in the supine position
b- Assessing the infant’s red reflex
c- Preventing heat loss from the infant
d- Administering humidified oxygen to the infant

164- The district nurse attends a 73 year-old woman on a daily basis. The woman’s husband had
passed away three years previously and she now lives alone. Besides assisting in activities of daily
living the nurse encourages the patient to talk about her past experiences and events.
Which theoretical frame work is most likely guiding the nurse’s care plan?
a- Erickson’s model of psychosocial development
b- Piaget’s theory of cognitive development
c- Watson’s theory of human caring
d- Roy’s adaptation model

165- A home care patient with chronic obstructive pulmonary disease (COPD) reports an upset
stomach. The patient is taking theophylline (Theo-Dur) and triamcinolone acetonide (Azmacort),
the nurse should instruct the patient to take
a- Theo-Dur on an empty stomach
b- Theo-Dur and Azmacort at the same time
c- Theo-Dur and Azmacort 12 hours apart
d- Theo-Dur with milk or crackers

166- A 64 year-old woman with hepatitis B has significant scarring on the liver and present to the
clinic with swelling around the eyes. She is disoriented to time and place and seems confused. Her
responses are slow and she has difficulty staying awake. A neurological assessment reveals
hypoactive reflexes.
Blood pressure 164/96 mmHg
Heart rate 58/min
Respiratory rate 22/min shallow
Temperature 37.0°C tympanic
Which of the following nursing interventions is most appropriate?
a- Prevent aspiration
b- Encourage small meals
c- Administer fluid bolus

41
d- Inject

167- A patient is being followed in the clinic for hypertension, adult onset diabetes, and obesity. The
patient is apathetic about learning about nutritional guidelines to reach the goals of weight loss
and consumption of a healthy diet. The patient admitted to eating “whatever is put in front of
me”
Which of the following actions would the nurse take?
a- Collaborate with the patient to set goals
b- Add a nursing diagnosis of non-compliance
c- Refer for psychiatric screening for depression
d- Discuss nutritional interventions with the spouse

168- A 16 year-old boy is in the Post-Operative Care Unit two hours after a tonsillectomy. He is alert
and oriented but complains of severe throat pain and difficulty swallowing. He rates the pain at a
level 7, on a scale of 1-10. The urine output from the folly catheter is 45 ml over the past two
hours.
Blood pressure 130/74 mmHg
Heart rate 64/min
Respiratory rate 18/min
Oxygen saturation 98% on room air
Which clinical finding is most important to report to the doctor?
a- Oxygen saturation
b- Difficulty swallowing
c- Urinary output
d- Pain level

169- A patient is being treated for pneumonia. Considering Maslow’s hierarchy of needs, which of the
following nursing diagnosis would have HIGHEST priority?
a- Risk for impaired skin integrity
b- Ineffective coughing.
c- Disturbed sleep pattern
d- Ineffective breathing pattern

A Knowledge deficit related to diabetes Mellitus


B Anxiety related to new diagnosis of diabetes Mellitus
C Altered nutrition more than body requirement
D Altered health maintenance related to elevated blood glucose
E Altered in fluid and electrolyte balance related to ketoacidosis
F Risk for infection related to chronic disease
170- An insulin dependent diabetic patient is admitted to the hospital in ketoacidosis.

42
Of the nursing diagnoses shown in the table which is the two with the HIGHEST priority?
a- A, C c- D, E
b- E, F d- B, E

171- A patient who is receiving chemotherapy has a platelet count of 49,000/𝐦𝐦𝟑 (normal value
150,000 to 400,000/𝐦𝐦𝟑)
Which of the following nursing action is necessary?
a- Minimize invasive procedures
b- Crush oral medications
c- Limit intake of vitamin k rich foods
d- Monitor the temperature every 4 hours

172- A child present to the emergency department with complaint of chest pain on exertion, shortness
of breath, and dizziness. Upon examination the nurse notes palpitation and a systolic ejection
murmur at the right upper sterna border. Skin color is normal. The nurse suspect.
a- Aortic stenosis
b- Aortic regurgitation
c- Mitral stenosis
d- Mitral valve prolapse

173- A four year-old girl was playing outside in a park when she came running to her mother crying
and holding her right, upper arm, with pain and itching at the site as well as swelling of the oral
mucosa. The child seems anxious
What is the next most appropriate step in management?
a- Administer subcutaneous epinephrine
b- Maintain a patent airway
c- Administer oral diphenhydramine
d- Prepare intubation equipment

174- A nurse who is assigned to care for a client with dementia and agitated behaviors knows that the
client worked as a housekeeper prior to becoming ill,
Which of the following activities is MOST likely to help the client if agitation develops?
a- Watching a movie about a house keeper
b- Reading a sheet of paper that contains stains removing tips
c- Dusting furniture in the assigned room with a cloth
d- Singing songs about cleaning

175- A 62 year-old patient has been treated for congestive heart failure and a nursing diagnosis of
fluid volume excess. After diuretic therapy and dietary interventions, the patient has met all
short-term goals. The nurse should.

43
a- Revise the care plan with a diagnosis of risk for alteration in fluid balance
b- Add a new diagnosis of risk of fluid volume deficit
c- Discontinue the care plan as the diagnosis is resolved
d- Continue the care plan as written

176- A diabetic patient with an infected right second toe is treated with a 14-day course of antibiotics.
The patient returns for a follow- up visit after finishing the prescribed course of antibiotics.
What can be inferred from the accompanying blood laboratory result drawn at the follow-up
visit?
Test Result Normal Value
WBC 12.0 4-10.5x 109 /L
HCT 0.41 0.41-0.50
Hemoglobin 153 135-171g/dl
Blood urea nitrogen 6.4 2.8-8.9nunol/L
Creatinine clearance 106. 70-103nunol/L
Glucose, fasting 16.6 3.5-6.5nunol/L
a- Diabetes is effectively managed but toe infection is not managed well
b- Diabetes and toe infection are effectively managed
c- Diabetes and toe infection are effectively managed but toe infection is effectively managed
d- Diabetes and toe infection are not effectively managed

177- A 60 year-old woman, with no previous medical history is diagnosed with an adenocarcinoma of
the duodenum head of the pancreas and the distal end of the bile duct. Staging shows it has not
grown into the large blood vessels and has not metastasized to any lymph node or distant site. The
patient undergoes surgery.
Which post-operative clinical finding is most likely?
a- Increased serum glucose
b- Constipation
c- Weight loss >15%
d- Paralytic ileus

178- The nurse assists with a lumbar puncture on a child with suspected bacterial meningitis. If the
diagnosis is correct the cerebrospinal fluid should have which of the following qualities?
a- Pink viscous appearance
b- Clear fluid appearance
c- Turbid appearance
d- Blood-tinged appearance

179- A patient presents with pneumonia. Which of the following intervention would the nurse
implement to improve the patient’s gas exchange?
Heart rate 80/min

44
Respiratory rate 32/min
Temperature 40°C
a- Encourage the patient to increase fluid intake
b- Position the patient in an upright position
c- Instruct the patient the patient to splint when coughing
d- Apply heat and or cold to the chest as ordered
180- While providing care to a hospitalized child with third-degree burns over 20% of the body, the
child rapidly becomes disoriented. The nurse knows this is an indicator of:
a- Septic shock
b- Separation anxiety
c- Dehydration
d- Depression

181- A 68 year-old male patient has a chest tube and properly fixed following cardiac surgery. The
chest tube is patent and functioning. When the nurse removes the dressing the tube falls out of
the chest.
Which is the most appropriate initial intervention the nurse should take?
a- Reinsert the chest tube and notify the surgeon
b- Apply an occlusive dressing to the insertion site
c- Place the open end of the tube in 20 cm of water
d- Administer 𝑂2at 10L/min via non-rebreather mask

182- During surgery requiring general anesthesia, the patient’s heart stops and a carotid pulse is not
palpated.
How many compressions per minute should be administered?
a- 50
b- 60
c- 80
d- 100

183- A 64 year-old man present with a weight gain of four kilograms in the past week. A physical
examination shows an enlarged liver, an enlarged abdomen with a fluid wave, and jugular venous
distension. The nurse creates a care plan based on the following nursing diagnosis. Decreased
cardiac output related to ventricular damage, ischemia and restriction secondary to fluid
overload.
Which would be an appropriate defining characteristic?
a- Altered preload
b- Altered afterload
c- Poor ventricular contractility
d- Increased pulmonary congestion

45
184- After the nurse has administered a heparin injection. The patient is observed rubbing the area.
The nurse should instruct the patient to not rub the area, as it may
a) Increase the risk of bleeding.
b) Prevent proper absorption of the medication.
c) Cause increased pain.
d) Result in tissue death.
185- A home nurse attends an obese patient, with decreased mobility. Who has developed pain in her
left calf muscle on examination, the leg appears swollen and red with large superficial veins. The
nurse refers the patient for investigation and treatment (see lab results ) :
Test Result Normal Values
Platelet count 49 140-400 x 10 / L
INR 1.5 0.7 – 1.2
What type of treatment is most appropriate for this patient?
a) Unfractured heparin
b) Anti-embolic stocking
c) Warm compresses and heating pads
d) Anti-inflammatory medications

186- A hospitalized patient eats 20% of the meal and states being too tired to eat more .What should
the nurse do?
a) Offer to feed the patient after short rest period.
b) Encourage the patient to finish the fluids.
c) Remove the meal tray and allow the patient to rest.
d) Encourage the patient to finish the protein portion of the meal

187- The district nurse attends a mother and a girl who was born five days previously. At birth, the
baby had weighed 3.5 kilograms. The mother becomes worried when she hears that the baby weighs
3.3 kilograms today. The infant is pink, alert and active with 6-8 wet diapers and four stools per
day. Blanching of the skin shows a light yellow color of the forehead but a pink sternum.
What is the most appropriate nursing action?
a) Notify the pediatrician
b) Obtain blood for bilirubin analysis
c) Reassure the mother and continue observing
d) Request the mother go to the clinic to re-weigh

188- A 6-month-old boy is admitted with failure to thrive. Based on his growth chart the nurse should
consider which of the following as the cause of the child's failure to thrive?
a) Parent-child interaction
b) Malrotation of the colon
c) Genetic chromosomal abnormalities
d) Congenital heart disease

46
189- A 40 year-old woman presents with right hip pain palpation of the pelvic girdle is normal. An X-
ray shows bone deformities, with osteolytic lesions and bone enlargement. The patient has not
suffered trauma and has been generally health.
Which serum laboratory analysis would be most useful?
a) Prothrombin time
b) Alkaline phosphatase
c) Acid phosphatase
d) Parathyroid hormone

190- A 22 years-old man is brought to the Emergency Department after falling four meters from a
rooftop. He complains of coldness, numbness and tingling in the right lower leg and foot. He is
unable to move the right leg and there is an angular deformity over the thigh muscle ( see X-ray )
Blood pressure 90/68 mmHg
Heart rate 110/min
Respiratory rate 26/min
Temperature 36.3 C
Oxygen saturation 94% on room air
Which type of intravenous fluid administration is most appropriate?
a) Lactated Ringers solution
b) 5% dextrose in water solution
c) 3.3% glucose in 0.3% sodium chloride
d) 5% dextrose in 0.45% normal saline

191- A 60 year-old man with a history of long-term alcohol use presents to the clinic with an enlarged
abdominal girth, hemoptysis and pruritus. His eyes and skin are visibly yellow-coloured. Which
additional finding would indicate the need for immediate hospital admission?
a) Azotemia
b) Asterixis
c) Tachycardia
d) Lower extremity edema

192- 192-
Diagnosis Medication order Past medical history
Glucophage 850mg po. qd Heart failure
Dilated cardiomyopathy Imdur 60mg. po .qd DM type 2
Lasix 80mg. po. qd Peripheral vascular disease
Zocor 40mg. po. qd (pvd)
primary hyperlipidemia

47
Based on the patient’s history which of the following medication orders should the nurse
verify?
a) Glucophage
b) Zocor
c) Lasix
d) Imdur
193- Patient experienced (CVA) one week ago. The doctor order the patient discharged from the
hospital to home health care. The visiting nurse evaluates the patient and notes the patient has
difficulty initiating speech and forming words.
Which of the following types of aphasia is most likely in this patient?
a) Receptive
b) Expressive
c) Fluent
d) Conduction

194- A community health nurse assesses a68 year-old patient who lives in a group home. The patient
reports decreased appetite after transferring to the group home because the food tastes bland.
What type of data is the nurse collecting from the above information?
a) Analytical
b) Derived
c) Objective
d) Subjective

195- A patient had a retinal detachment surgically repaired the nurse identifies that the detachment
would most likely be corrected and unlikely to reoccur if the retina remains attached at least :
a) Three days
b) Two weeks
c) Two months
d) Three months

196- A patient complains of incoordination, weakness and fatigue, double vision, and slurred speech
laboratory test results are normal except for magnetic resonance imaging (MRI) scan which shows
small plaques scattered throughout the white matter. The nurse would anticipate the diagnosis of
:
a) Multiple sclerosis
b) Brain metastasis
c) Amyotrophic lateral sclerosis (Lou Gehrig disease )
d) Polyradiculoneuritis ( Guillain-Barre syndrome )

197- A gravida 3 Para 2 presents to the Maternity Triage Unit after the amniotic membranes ruptured
at home. The fluid is noted to be clear. The neonate's head is engaged into the pelvis and the patient
is having contractions every 5 to 7 minutes. Each contraction lasts for 60-90 seconds. An

48
examination of the cervix finds 4 centimeters dilatation and 90% effacement. She is uncomfortable
during contractions and rates the pain at a level 7, on a pain scale of 1-10
Which finding is most indicative that she is in true labor?
a. Level of pain
b. Cervical dilatation and effacement
c. Engagement of presenting part fractions
d. Frequency and length of contraction

198- A nurse is caring for child who is post-tonsillectomy and adenoidectomy. The nurse should plan
to assess which of the following complications?
a) Pulmonary hypertension
b) Hemorrhage
c) Hearing loss
d) Cor pulmonale

199- A home care nurse visits a diabetic patient who was started on insulin injection, upon
examination, the nurse observes small lumps and dents on the right upper arm where the patient
has injected insulin.
What is the BEST nursing intervention?
a) Refer the patient to dermatologist for diabetic cellulitis
b) Instruct the patient to rotate the sites of injection
c) Refer the patient to an endocrinologist for better control of glucose level
d) Instruct the patient to inject in the muscular area instead of the subcutaneous area

200- A patient is planning to have an elective surgical procedure to repair an umbilical hernia. The
patient is 68 years old. Weighs 136 kg (300 lbs.) Has diabetes mellitus.
Which of the following approaches would be the MOST beneficial in order to reduce the patients
surgical risk?
a. Monitor blood glucose levels monthly
b. Avoid fluid overload by restricting fluids
c. Discourage any changes in routine before surgery
d. Encourage weight reduction

201- The nurse is discussing the human immunodeficiency virus (HIV) with a group of high - risk
patients. The nurse should state that this virus is found MOST commonly in which of the following
body fluids?
a. Blood
b. Saliva
c. Breast milk
d. vaginal secretions

49
202- The nurse has provided dietary counseling to a woman who is 12 weeks pregnant with her first
child. She is concerned about her nutrition as she is vegetarian. She was advised by a friend to eat
a variety of foods that contain proteins and asks the nurse which food might be best included in a
meal plan.
Which combination would be most appropriate?
a. Soya beans with carrots
b. chickpeas with whole grain bread
c. Nuts with a glass of cow's milk
d. Avocado with a slice of cheese

203- After total knee replacement a patient is being discharged to home after which he will ambulate
with a four-prong cane. When providing patient teaching regarding going up and down stairs with
the cane, the first step in going up stairs is to
a. Place the unaffected extremity up on the step
b. Place the cane up on the step
c. Step-up on the affected extremity
d. Step-up on the unaffected extremity

204- A nurse educates a patient about the use of incentive spirometry to prevent atelectasis after a
surgery.
The nurse is performing what step of the nursing process?
a. Diagnosis c- implementation
b. Assessment d- evaluation

205- A 65 year-old male presents with sever iron deficiency anemia.


Which of the following would the nurse expect to find in the initial physical assessment?
a. Clubbing
b. Koilonychia
c. Beus line
d. Paronychia

50
206- A 32-year-old patient presented to the clinic with upper right abdominal quadrant pain which
increases with fatty meals, on palpation, there is guarding and rebound tenderness. Further
investigation reveals a small stone which is trapped in the common bile duct On admission:
Blood pressure 148/80 mmHg
Heart rate 90min
Respiratory rate 21 shallow
Temperature 37.8
Which additional clinical finding is most likely?
a. Yellowing of the sclera
b. Dark and tarry stools
c. light-amber coloured urine
d. Clubbing of the fingers

Health seeking behavior related to therapeutic regimen – intervention – the nurse will.
A Provide written and oral education on diabetes management.
B Evaluate the child's knowledge of disease process.
C Evaluate the Mother's knowledge of the disease process.
D Evaluate the child's cognition and readiness of learn.
E Demonstrate capillary blood testing and insulin injections.
F Provide opportunity for return demonstration of capillary blood testing and insulin injection.
G Evaluate Mother's need to control the child through disease management.
207- A 10 year-old child with a seven year history of diabetes mellitus presents to the office with a
parent for routine follow-up. The patient wants to manage capillary blood testing and insulin
injections independently. The parent believes the child is not mature enough to manage the disease,
while the child is knowledgeable about dietary needs and can recognize the signs of hyperglycemia
and hypoglycemia.
What are the two HIGHEST priority interventions?
a. A , E
b. C , G
c. B , D
d. E , F

208- Which of the following medications would the patient be instructed to stop 2 weeks prior the
surgery?
a. Antiarrhythmic
b. Antifungal
c. Antidiabetics
d. antiplatelet

209- The nurse in preparing to insert RYLE’S tube (NGT) into an infant, the nurse knows that the
length of the tube should be taken as following:
a- From the nose down to the chin and then to the umbilicus
b- From the nose to the earlobe and then to the xiphoid process
c- From the nose to the mouth to the xiphoid process
51
d- From the nose to the earlobe to the umbilicus

210- A Seven year-old boy presents with a fine maculopapular rash over the entire body. The mother
report’s that prior to the rash developing there had been tiny grey-white spot over the buccal
mucosa (see clinical image).
Which precautions would most effectively prevent transmission?
a. Private room, gloves and gown for care takers.
b. A mask over the patient’s mouth and nose.
c. Negative pressure room and N95 mask for staff.
d. Hand washing and disposable patient equipment.

211- A community health nurse screens a group of high-risk adults for tuberculosis.
Which gauge needle should the nurse use for an intradermal injection on the volar surface of the
forearm?
a. 16 gauge needle
b. 20 gauge needle
c. 22 gauge needle
d. 26 gauge needle

212- A 45 year-old women is admitted for a surgical procedure. Local anesthetic medication,
bupivacaine, and fentanyl are administered through a tiny needle placed into the subarachnoid
space. The anesthetic is administered with the patient in an upright and seated position. five
minutes later, the nurse records the patients vital signs:
Blood pressure 88/58 mmHg
Heart rate 110/min
Respiratory rate 18/min
Temperature 38.0 oc tympanic
Which of the following vital signs should be reported at once?
a. Blood pressure
b. Pulse rate
c. Respiratory rate
d. Body temperature

213- The nurse should understand that one dose of which of the following vaccinations is recommended
for this patient before the age of 65 years:
a. Varicella
b. Influenza

52
c. Hepatitis B
d. Pneumococcal
214- A home care nurse visits a patient with a new below–the-knee amputation. The site of the incision
is red, warm and tender with purulent yellow drainage. The patient has a new prescription for
cephalexin (Keflex) and oxycodone (OxyContin)
Which would the nurse instruct the patient to do FIRST?
a. Take oxycodone as soon as possible
b. Take cephalexin as soon as possible
c. Wash the incision site and apply bacitracin cream
d. Wash the incision site and apply hydrocortisone.

215- 39 year-old male has the following vital signs the patient has received 2 unites of packed red blood
cells (PRBCs) and is intubated.
Which of the vital signs is considered out of normal range?
Blood pressure 126/78 mmHg
Heart rate 62/min
O2 saturation 89% on room air
Temperature 37 oC
a. Heart rate
b. O2 saturation rate
c. Core temperature
d. Blood pressure

216- According to the hypothetical graph of urinary output versus time of intravenous (IV)
administration, at what hour of IV administration dose the urinary output attain the desired
output of 50 ml/hr.:
a. 2 hours
b. 4 hours
c. 6 hours
d. 8 hours

217- Which of the following position is appropriate for the patient to use for self-administration of
fleet enema:
a- Left lateral position right leg flex
b- Face down and leg flex
c- Face upward and leg free (on back and flex hip and knee)
d- Chest- knee position

218- While caring for a patient in the post- anesthesia care unit (PACU) who has developed
hypovolemic shock, a nurse should position the patient:
a- Flat with legs elevated
b- In trendelenburg position
c- With the head of the bed elevated 45 degrees
53
d- Completely flat

219- The nurse performs a home visit for a 32 years- old woman who had given birth to her first
infant three days before. The mother has concerns about breastfeeding and the nurse
observes the infant feeding. The mother sits supported upright with cushions and the infant
positioned in a cradle hold. The infant’s head and body are aligned against the mother’s
abdomen. The infant sucks intermittently with the lips turned outwards and the nipple in
the mouth.
Which intervention is most appropriate?
a- Reposition the infant
b- Assist the infant to turn lips inwards
c- Ensure the latch includes the areola
d- Stimulate the infant to suck constantly

220- A patient is preparing for a scheduled hip replacement.


Which lab value should be reported to the physician?
Test Result Normal values
Sodium 145 134-164 mmol/L
Potassium 2.9 3.5-5.1 mmol/L
Calcium 4.80 2.15-2.62mmol/L
Magnesium 4.8 1.2-2 mmol/L
a- Sodium
b- Calcium
c- Potassium
d- Magnesium

221- A nurse is admitting a six month- old infant with pneumonia. Which of the following
interventions supports this infant’s emotional needs?
a- Allow the parents to leave the room during painful procedures
b- Encourage parents to distract the infant from crying
c- Interview the patents to learn the infant’s comforting habits
d- Enforce strict visiting schedule and routines

222- A five years – old child girl had presented with tenderness, headache and petechiae. She was
pale and complains of joint pain. On palpation there was an enlarged spleen, liver and
lymph nodes. A lumbar puncture showed central nervous system involvement. The child
underwent chemotherapy treatment and is now attended by the nurse for regular routine
examinations.
Which condition would require more attention from the nurse?
a- Increased leukocytes
b- Lack of muscle coordination
c- Bleeding while brushing teeth

54
d- Occasional nausea and vomiting

223- During the evaluation at a community clinic, the patient completes the medical history.
Which of the following is NOT a risk factor for an acute myocardial infarction?
a- Coronary artery disease
b- Smoking
c- Hemophilia
d- Hyperlipidemia

224- A postoperative patient has the nursing diagnosis of ineffective tissue perfusion. To assess
for tissue perfusion the nurse should check all of the following except:
a- Skin and nail bed color
b- Temperature of extremities
c- Respiratory rate
d- Peripheral pulses

225- A patient visits the clinic for a 2- week checkup after a corneal transplantation
(keratoplasty). The nurse observes the patent’s sclera is red and the patient complains of the
eye feeling irritated. The nurse suspects the patient may have:
a- Infection c- Graft rejection
b- Hemorrhage d- Postoperative glaucoma

226- The nurse is assessing a patient who just had surgery under general anesthesia. The
patient’s respiration rate is 4 per minute and the O2 saturation on 3L per minute of oxygen
via nasal cannula is 84%. The nurse is awaiting the results of an arterial blood gas (ABG)
and anticipates that which of the following will be ELEVATED?
a- Arterial oxygen saturation (SaO2)
b- Hydrogen ion concentration (pH)
c- Partial pressure of arterial oxygen (PaO2)
d- Partial pressure of arterial carbon dioxide (PaCO2)

227- During the immediate postoperative period, a patient reveals an oxygen saturation of 91%.
The nurse should:
a- Position the patient on the left side
b- Administer supplemental oxygen
c- Continue to provide supportive care
d- Lower the temperature of the room

228- Causes of primary hypothyroidism in adults include:


a- Malignant or benign thyroid nodules
b- Surgical removal or failure of the pituitary gland

55
c- Surgical removal or radiation of thyroid gland
d- Autoimmune-induced atrophy of the gland

229- The nurse performs an assessment of a 23 year- old man who believes that people are
spying on him. During the interview, he keeps his eyes to the floor and answers questions
awkwardly. He has never had an intimate relationship and avoids contact with his family
members. He has never been employed and tells the nurse that he is not looking for a job.
The nurse considers Erickson’s theory of psychosocial development. Which stage is this
patient most likely experiencing?
a- Autonomy versus shame and doubt
b- Initiative versus guilt
c- Trust versus mistrust
d- Identity versus confusion

230- A 32 year- old woman with a motor neuron disease has progressive weakness of cranial
nerves V, IX, XII. She has recently been experiencing sudden outbursts of crying and
laughing. Both stroke and myasthenia gravis have been ruled out.
Which voluntary muscle activity would be most significantly impacted?
a- Walking
b- Swallowing
c- Breathing
d- Smelling

231- When planning discharge teaching for the parent of an infant with bronchiolitis, the nurse
should EMPHASIZE:
a- Use of supplemental oxygen at night
b- Frequent hand washing
c- Sleeping in the supine position
d- Rice- thickened formula during night- time feedings

232- A 65 year- old man presents with a resting tremor in the right forearm. An assessment of
gait reveals decreased arm swinging and slight dragging of the foot on the right side. His
body movements are slow. He has not been sleeping well at night for the past six months due
to leg pain and says that he feels constantly tired and weak. He reports that he has not
suffered any recent fall and that the symptoms seem to be slowly worsening.
Which medication is most likely to be administered?
a- Levodopa c- Phenytoin
b- Haloperidol d- Benzodiazepine

233- A patient has an order for 1000 milliliters (ml) of intravenous (IV) fluid to infuse over eight
hours. The available IV tubing has a drip factor of 10 gtts/ml.
Which of the following rates is correct?

56
a- 125 ml/hour
b- 125 drop/minute
c- 21 drops/minute volume (ml) X drop factor (gtts / ml)
= gtts / min
---------------------------------------------
d- 21 ml/hour (flow rate)
time (min)

234- When is the correct time for the nurse to administer the child’s morning dose of a
combination regular and NPH insulin?
a- 30 minutes before breakfast is served
b- 15 minutes before breakfast is served
c- 30 minutes after breakfast is served
d- 15 minutes after breakfast is served

235- The client has an order for administration of 10 units of regular insulin to be given at 7:00
a.m. The nurse should offer a snack at:
a- 3:00 p.m. c- 11:00 a.m.
b- 1:00 p.m. d- 9:00 a.m.

236- The client is being treated with NPH insulin at 8:00 a.m. The nurse should offer a snack at:
a- 9:00 a.m. c- 6:00 p.m.
b- 11:00 a.m. d- 2:00 p.m.

237- A client with diabetes comes to the emergency department. The nurse obtains a blood
glucose measurement with a glucometer and notes that it is 510 mg/dL. The physician orders
I.V. insulin. Which type of insulin can be given both intravenously and subcutaneously?
a- Regular c- Lente
b- NPH d- 70/30

238- If a client with type 1 (insulin-dependent) diabetes mellitus receives 5 units of NPH insulin
every morning at 7 a.m., the nurse should closely monitor the client for signs of
hypoglycemia at what time:
a- 3:00 p.m. c- 7:30 a.m.
b- 12:00 a.m. d- 10:00 p.m.

239- NPH is an example of which type of insulin:


a- Short-acting c- Rapid-acting
b- Intermediate-acting d- Long-acting

240- A child comes into the clinic with several lesions to the scalp. The round lesions have
dandruff like scaling with hair loss.

57
What is the MOST likely diagnosis for this skin condition?
a- Impetigo
b- Ringworm (tinea capitis)
c- Pediculosis capitis
d- Scabie
241- A 70 years- old woman presents with increasing dyspnea on exertion. She feels breathless
and restless while performing household tasks, such as making the bed and sweeping the
floor. Her previous medical history includes a myocardial infarction at 57 years old. She
sleeps with her head elevated on three pillows. Examination reveals bilateral basal crackles
and cold, damp skin.
Blood pressure 172/94 mmhg
Heart rate 94/min
Respiratory rate 36/min
Temperature 37.1 C
Oxygen saturation 90% on room air
Which heart chamber most likely failed first?
a- Right atrial
b- Right ventricle
c- Left ventricle
d- Left atrial

242- A nine year- old girl has a nursing diagnosis of altered body image related to changes in
appearance secondary to varicella infection. The child’s body is covered with a rash and
many large, weeping pustules. The nurse provides counseling to the mother who is
concerned that the child will develop scarring.
Which intervention is most appropriate?
a- Keep the skin out of direct sunlight
b- Apply calamine lotion to skin every two hours
c- Wash pustules with soap and keep dry
d- Soak in colloidal oatmeal bath three times daily

243- A 40 years- old woman suffered an automobile accident one month ago. The accident
resulted in a C-4 spinal cord injury. Her preventive care includes independent daily
performance of coughing and deep breathing.
Which range- of- motion exercise would be most beneficial for this patient?
a- Active
b- Passive
c- Combined
d- Resistive

244- A nurse is assessing a client with possible Cushing's syndrome. In a client with Cushing's
syndrome, the nurse expects to find:
a- Weight gain in arms and legs

58
b- Thick, coarse skin
c- Hypotension
d- Deposits of adipose tissue in the trunk and dorsocervical area

245- Which vitamin deficiency is most likely to be a long-term consequence of a full-thickness


burn injury?
a- Vitamin A c- Vitamin C
b- Vitamin B d- Vitamin D

246- Sildenafil (Viagra) is prescribed to treat a client with erectile dysfunction. A nurse reviews
the client's medical record and would question the prescription if which of the following is
noted in the client's history:
a- Neuralgia c- Use of nitroglycerin
b- Insomnia d- Use of multivitamins

247- Patient obtained second degree burn to his abdomen and his entire back, according to rule
of nine what is the percentage of burn of total body surface area:
a- 9%
b- 18%
c- 27%
d- 36%

248- Which nursing diagnosis takes priority for a newly diagnosed patient with a left- sided
stroke?
a- Risk of impaired swallowing related to absent gag reflex
b- Risk for impaired skin integrity related to immobility
c- Risk for infection related to invasive line placement
d- Risk for impaired speech related to left sided stroke

249- When caring for a patient with left-sided homonymous hemianopsia resulting from a
stroke, which intervention should the nurse include in the plan of care during the acute
period of the stroke?
a- Apply an eye patch to the left eye
b- Approach the patient from the left side
c- Place objects needed for activities of daily living on the patient's right side
d- Reassure the patient that the visual deficit will resolve as the stroke progresses

250- A client with a serum glucose level of 618mg/dl is admitted to the facility. He’s awake and
oriented, has hot dry skin, and has the following vital signs:
Temperature 38.1º C

59
Heart rate 116 beats/minute
Blood pressure 108/70 mm Hg
Based on these assessment findings, which nursing diagnosis takes the highest priority?
a- Deficient fluid volume related to osmotic diuresis
b- Decreased cardiac output related to elevated heart rate
c- Imbalanced nutrition: less than body requirements related to insulin deficiency
d- Ineffective thermoregulation related to dehydration
251- The nurse is caring for a patient with a deep vein thrombosis (DVT). The patient’s heparin
sodium infusion has been discontinued and the patient is receiving prescribed warfarin
sodium (Coumadin). The nurse should advise the patient that which of the following needs to
be continued?
a- Daily complete blood count (CBC)
b- Laboratory tests for partial thromboplastin time (PTT)
c- Strict Bedrest
d- Wearing elasticized support stockings

252- A patient receiving a total parenteral nutrition through a central line suddenly has
difficulty breathing and is restless. Chest auscultation reveals a heart murmur.
Blood pressure 90/60 mmhg
Heart rate 120/min
Respiratory rate 22/min
Temperature 37.1 C
Oxygen saturation 90% on room air
What is the most appropriate initial nursing action?
a- Notify the physician
b- Administer 100% oxygen by face mask
c- Place patient in left side lateral decubitus
d- Obtain stat blood glucose measurement

253- A patient is scheduled for a pneumonectomy in the morning.


Which of the following diagnosis is the MOST likely indication for this type of surgery?
a- Lung carcinoma c- Benign pulmonary nodule
b- Pulmonary tuberculosis d- Mediastinal shift

254- A 16 year- old boy fractured his right tibia and fibula during a football match. Eight weeks
later, the fractures were successfully healed and the cast was removed.
Which range- of- motion exercises would be most beneficial?
a- Active c- Combined
b- Passive d- Resistive

255- The nurse is teaching a patient who was recently diagnosed with rheumatoid arthritis
affecting the hands.
Which of the following treatment should the nurse discuss with the patient at this time?
a- Transcutaneous electric nerve stimulation (TENS)
60
b- Iontophoresis
c- Nonsteroidal anti-inflammatory drugs (NSAIDs)
d- Synovectomy

256- What do we call the type of treatment of terminal illness patients?


a- Conservative c- Palliative
b- Curative d- Selective

61
257- A 34 year- old quadriplegia patient resides at home with his wife. In order to prevent
contractures of all extremities, the community care nurse will instruct the patient’s wife in
performance of:
a- Active range of motion exercises
b- Passive range of motion exercises
c- Active- assistive range of motion exercises
d- Resistive range of motion exercises

258- A 65 year- old woman who was diagnosed with hypothyroidism at the age of 45 is brought
to the clinic by ambulance. On arrival, she had a severely decreased level of consciousness.
Her breathing is shallow and irregular. The skin is cool, dry and pale. There is generalized
non- pitting edema of all extremities and face.
Blood pressure 70/40 mmhg
Heart rate 60/min
Respiratory rate 12/ min
Body temperature 35.5 C tympanic
What is the most appropriate method to re-warm this patient?
a- Warmed intravenous isotonic fluids
b- Place the patient is a warm bath
c- Apply heat packs to the head and neck
d- Cover the patient with hospital blankets

259- A six years- old patient has presented to the clinic with fever, malaise, and anorexia. The
patient was treated 2 weeks ago for a streptococcal infection of the throat.
The nurse should expect the physician to order what test?
a- Electrocardiogram
b- Jones test
c- Spinal tap
d- Heart biopsy

260- Which of the following actions is the first priority of care for a client exhibiting signs and
symptoms of coronary artery disease?
a- Decrease anxiety
b- Enhance myocardial oxygenation
c- Administer sublingual nitroglycerin
d- Educate the client about his symptoms

261- The parent of a child with chronic asthma is hesitant to discipline because the child often
doesn’t feel well. The nurse should encourage the parent to:
a- Set consistent behavior limits
b- Be more lenient during times of illness
c- Cherish the limited time the child has to live
d- Avoid upsetting the child with limit- setting

62
262- A 31 years- old woman with diabetes type 1 presents to the clinic with fatigue, blurred
vision, and loss of appetite. Her breath smells like fruit and she leaves the room twice during the
examination to use the toilet. She has brought a little bottle of water with her that she finishes while
at the clinic. She reports that she has had a cold for the past three days, but has not taken insulin
during the illness
Blood pressure 130/70 mmhg
Heart rate 90/min
Respiratory rate 20/min
Body temperature 38.0 C oral
What is the most appropriate nursing diagnosis
a- Risk for impaired skin integrity related to circulation
b- Deficient knowledge related to illness management
c- Risk for fluid volume excess related to fluid intake
d- Imbalanced nutrition related to decreased appetite

263- Adult patient suffers from hoarseness of voice, and difficulty in speech may be suffering
from an injury of which of the following cranial nerves:
a- IX c- XI
b- X d- XII

264- A 55 year- old woman presented with pain in the joints of the hands. She feels generally
tired and has had a low grade fever for the past week. On examination, the joints of the fingers on
both hands are symmetrically swollen.
Blood pressure 120/70 mmhg
Heart rate 82/min
Respiratory rate 18/min
Temperature 38.0 C
Oxygen saturation 99% on room air
Which type of arthritis is most likely?
a- Rheumatoid
b- Septic
c- Psoriatic
d- Osteoarthritis

265- A patient presented to the office for a physical examination. The patient is found to be
healthy and fit but occasionally drinks alcohol and has unprotected sex.
What is the BEST nursing diagnosis?
a- Health- seeking behavior
b- Knowledge deficit, high- risk behaviors
c- Low self- esteem
d- Altered thought process

63
266- The nurse is caring for a woman whose husband beats her regularly. Which is the most
important long-term goal for this woman?
a- Provide a long-term support group
b- Help her feel like a survivor
c- Point out the ways she behaved
d- Be able to blame the abuser

267- A seven year- old child presented to the emergency room with a fracture of the right arm.
What would be the long- term goal for this patient?
a- Patient verbalizes decreased pain level
b- Nurse notes neurovascular checks are within normal limits
c- Nurse notes decreased swelling of the right arm
d- Patient performs activities of daily living without difficulty

268- Which assessment finding would the nurse expect in an infant diagnosed with pyloric
stenosis?
a- Abdominal rigidity
b- Ribbon-like stools
c- Visible waves of peristalsis
d- Rectal prolapse

269- The following table represents the blood pressure value for a patient in 3 successive days.
Regarding the information given in the tablet, this patient is considered:
a- Normal blood pressure
b- Pre-hypertension
First day 120/80
c- Hypertensive
d- Hypotensive Second day 122/87

Third day 133/88

270- A patient with cerebrovascular accident, left-sided hemiplegia, and aphasia has nursing
diagnosis of risk for aspiration related to swallowing difficulties with a short-term goal that
the patient will not aspirate. The patient has undergone insertion of a percutaneous gastric
endoscopy and has all nutrition and fluid administered through the tube. The nurse should:
a- Continue the care plan as written
b- Discontinue the risk for aspiration diagnosis
c- Revise the goals and interventions for the diagnosis
d- Add a nursing diagnosis of ineffective health maintenance

64
271- A home care nurse reviews the second follow- up laboratory test results of a patient with
iron deficiency anemia. The nurse compares the recent laboratory results to the results
taken 3 months ago. The patient has been eating food rich in iron as prescribed by the
nutritionist.
What can be inferred about the progress of the treatment by comparing the two laboratory
values?
Laboratory values three months prior
Test Result Normal value
Albumin 35 34-56 g/L
Glucose 5.3 3.5-6.5 mmol/L
HCT 0.32 0.41-0.50
Hb 121 120-140 g/L
Blood urea nitrogen 6.1 2.8 to 8.9 mmol/L
Creatinine 60 58-145 μmol/L
Current laboratory values
Test Result Normal value
Albumin 33 34-56 g/L
Glucose 5.7 3.5-6.5 mmol/L
HCT 0.41 0.41-0.50
Hb 139 120-140 g/L
Blood urea nitrogen 5.7 2.8 to 8.9 mmol/L
Creatinine 60 58-145 μmol/L
a- Patient’s uptake of iron by diet is sufficient, and no additional intervention is necessary
b- Patient’s uptake of iron by diet is sufficient, but additional intervention is necessary
c- Patient’s uptake of iron by diet is insufficient, and additional intervention is necessary
d- Patient’s uptake of iron by diet is insufficient, and the physician’s immediate action is
necessary

272- A six year-old child with type 1 diabetes has an uncontrolled blood glucose level. The child
has been given regular insulin with minimal change in glucose level in the first 30 minutes.
Which type of insulin has an onset of 15 minutes and a duration of no more than 4 hours?
a- Aspart (Novolog) Type of insulin Onset(minutes) Maximal Duration
b- Lispro (Humalog) activity(hours) (hours)
c- Semilete Aspart(Novolog) 15 1-3 3-5
d- Neutral protamine hagedom (NPH) Lipsro(Humalog) 10-30 1-2 2-4
regular 30-60 2-4 6-9
semilente 30-60 2-4 10-12
NPH 120 4-12 24
Lente 120 8-10 24

273- When administrating an intramuscular injection to an infant, which of the following sites is
appropriate for the nurse to use?
a- Rectus femoris C- Dorsogluteal
65
b- Deltoid D- Ventrogluteal

274- A nurse is evaluating a patient 5 days after a right total hip replacement.
Which of the following goals is appropriate for the patient?
a- Maintain abduction without dislocation
b- Rest with legs elevated while sitting
c- Tie shoes and put on undergarments without assistive devices
d- Perform scissor-like exercises daily

275- A six year- old boy is in the postoperative care unit following a tonsillectomy. The nurse
observes that his gag reflex has returned and removes the artificial airway. The patient then
begins to cry and tells the nurse that his throat hurts badly. What type of data is the patient
providing the nurse?
a- Objective c- Comparative
b- Inferential d- Subjective

276- A three year- old child was admitted to the postoperative care unit following a heart
transplant. The nurse administers cyclosporine by intravenous infusion. Fifteen minutes
later the child has difficulty breathing, his skin feels cold and clammy and he appears
restless. Which is the most appropriate initial nursing action?
a- Ensure airway patency
b- Administer oxygen therapy
c- Discontinue intravenous infusion
d- Administer intramuscular epinephrine

277- A 40 year-old woman is a gravida2, para 2 and is currently trying to conceive. Her previous
pregnancy resulted in the birth of a baby with cleft lip and palate. The patient is anxious and
concerned about future pregnancies and the nurse provides genetic counseling and
reassurance.
Which food would most effectively prevent recurrence?
a- Green vegetables and citrus fruit
b- Eggs, milk and dairy products
c- Wheat, corn, rice, oats and rye
d- Beef, chicken and yellow vegetables

278- A 50 year-old woman presented with poor balance and coordination. She says that she has
developed pain on the outer aspect of the thighs and the inner side of the arch of the foot.
The nurse wishes to examine the integrity of the affected lumbar spinal nerve root and
performs a deep tendon reflex examination. Which reflexes would most likely be
diminished?
a- Planter c- Patellar
b- Brachioradialis d- Achilles tendon

66
279- In what position should a dyspneic patient be placed?
a- Prone
b- Recumbent
c- Semi-fowler’s
d- Trendelenburg

280- Oral iron supplements are prescribed for a 6-year-old child with iron deficiency anemia.
The nurse instructs the mother to administer the iron with which of the following food item
to enhance absorption of iron:
a- Milk
b- Water
c- Apple juice
d- Orange juice

281- While caring for a child with a ventriculoperitoneal shunt revision, the nurse finds the
patient lying with the head and feet flexed back. The nurse should call for help and prepare
for a (n):
a- Spinal tap
b- Shunt culture
c- Electrocardiogram
d- Ventricular tap

282- While assessing a child with pulmonary stenosis, the nurse should give PRIORITY to:
a- Deep tendon reflexes
b- Urinary output
c- Exercise tolerance
d- Pattern of food intake

283- Which instruction take priority in reducing anxiety related to surgery?


a- Surgical procedure and postoperative exercises
b- Risk of infection after surgery
c- Advanced directive and what it means
d- Pre-operative laboratory result and what to expect on it

284- After a hearing restoration operation, a patient has no signs of complications and soon
recovers. Which of the following is an expected outcome 5 days after the hearing restoration
surgery?
a- Regain full hearing

67
b- Minimal facial nerve paralysis
c- Minimal urinary incontinence
d- Ambulates without difficulty

285- Which of the following nursing diagnosis takes PRIORITY for a patient admitted to the
critical care unit with diabetic ketoacidosis?
a- Deficient fluid volume secondary to hyperglycemia
b- Risk of infection secondary to weakened immune system
c- Deficient knowledge of cause and prevention
d- Imbalanced nutrition related to hyperglycemic state

286- An 80 year-old man presents to the hospital with chronic fatigue, dyspepsia and
constipation. On examination he is jaundiced, has red palms, dilated veins around the
umbilicus, the abdomen is very distended and he has black, tarry stool on a rectal exam. He
is noted to be lethargic and have a flat tone. A paracentesis reveals clear colored fluid with
low protein content. What is most likely diagnosis?
a- Gilbert’s syndrome
b- Thalassemia
c- Sickle cell crisis
d- Cirrhosis

287- 287-
Admission Report at 22:45 (10:45p.m.) Diagnosis
14 years old admitted Remains febrile, maximum Bacterial
18 hours ago temperature this shift 39.3C pneumonia
Respiration rate 12-14 per minute
Heart rate 100-180 per minute
Blood pressure level within baseline
normal range
Next dose of intravenous anti-
infective is due at 08:00 (8:00 a.m.)
Refer to the accompanying figure.
The nurse should plan to monitor which of the following at least every 4 hours?
a- Vital signs c- Food intake
b- Urinary output d- Level of activity

288- The nurse is measuring the chest tube drainage of a patient who had open heart surgery 4
hours ago.
Which of the following is the MAXIMUM hourly amount of chest tube drainage is
expected in this timeframe?
a- 100 ml
b- 200 ml

68
c- 300 ml
d- 400 ml

289- A 69 year-old man is admitted to the intensive care unit following cardiac surgery. Two
hours after admission, the nurse performs a routine assessment and notes the patient’s chest
tube drainage is 200 milliliters and a dark red color. He has had 60 milliliters output from
the indwelling urinary catheter
Blood pressure 138/68 mmhg
Heart rate 76/min
Respiratory rate 16/min
Body temperature 37.0C oral
Oxygen saturation 94% 6L/min nasal cannula
Which finding should be reported to the doctor?
a- Dark red chest tube drainage
b- Urinary output
c- Oxygen saturation
d- Chest tube output volume

290- A 55 year-old man presented to the clinic with complaints of numbness and tingling in his
feet for the past 3 months. He appears thin and his skin is pale. Examination confirms the
loss of vibration sense. He also has altered proprioception. He had a subtotal gastrectomy
two years ago (see lab results)
Test result normal values
MCH 2.72 1.45-2.01 fmol/cell
HCT 0.31 0.41-0.50
MCV 105 81-97 fl
Reticulocyte count 1.5 0.5%-2.5%
Which supplement would most likely improve his condition?
a- Iron c- Folate
b- B12 d- Calcium

291- A 35 year-old man is hospitalized following a blunt chest injury. He has chest pain,
breathing difficulty and asymmetrical lung expansion. Chest auscultation reveals decreased
lung sounds on the left side. The jugular veins are distended and there is tracheal deviation
to the right.
Blood pressure 92/54 mmhg
Heart rate 120/min
Respiratory rate 24/min
Temperature 36.2C
SaO2 88% on oxygen
Which initial intervention is most appropriate?
a- Send patient for X-ray examination
b- Administer bronchodilators

69
c- Administer intravenous fluid bolus
d- Prepare for needle decompression

292- The following picture represents an auto immune disease known as:

a- Psoriasis
b- Systemic Lupus Erythematosus
c- Molluscum Contagiosum
d- Eczema

293- Which of the following anatomical terms best describes the motion in the picture above:
a- Adduction
b- Abduction
c- Supination
d- Pronation

294- Which of the above locations represents the best sites to inspect chest retractions in a child with
lower respiratory tract disorder:
a- A&B
b- B&C
c- C&D
d- D&E

295- The following condition may develop in females due to an increase in the production of which of
the following hormones:

70
a- Androgens
b- Progesterone
c- Estrogen
d- Growth

296- A nurse is caring for a three year-old child with a fractured arm.
Which of the following interventions is the MOST appropriate for pain management?
a- Administer analgesics when necessary
b- Assess pain once a shift
c- Anticipate pain and intervene early
d- Encourage the use of self-quieting techniques

297-
Goals- for the next three months, the patient will:
A Not have any migraine headaches
B Decrease the number of migraine headache
C Learn to tolerate her migraine headaches
D Continue to take prescribed preventative medications daily
E Initiate complementary pain management strategies as needed
F Wean self from all pain medications
G Keep a pain diary and recognize triggers and auras before migraine headaches commence
An adolescent patient with a history of migraine headaches for the past year has been
followed for pain management. The headaches have decreased in severity and now occur only
occasionally. What are the BEST three long term goals?
a- A, D, F c- C, E, F
b- B, D, F d- D, E, G

298- A community health nurse visits a patient who had a cerebrovascular accident. The patient
is at risk for deficient fluid volume due to voluntary reduction of fluid intake to avoid the use
of the bathroom. The nurse educates the patient on the importance of drinking fluids and
maintaining hydration. Which of the following indicates the efficacy of the nursing
intervention?
a- Amber color urine
b- Respiration of 35
c- Tachycardia
d- Moist mucous membrane

299- A patient with diabetes mellitus and multiple sclerosis has been prescribed baclofen
(Lioresal). The nurse knows this medication may result in?
a- Increased insulin needs c- Optic neuritis
b- Renal failure d- Muscle tremors

300- A patient is recovering from surgery using spinal anesthesia. The patient developed a spinal
headache. Which of the following nursing actions would be MOST appropriate?
71
a- Elevate the head of the bed 30 degrees
b- Keep the patient well hydrated
c- Limit intake of salty foods
d- Lower the temperature of the room

Nursing diagnosis
A Impaired physical mobility
B Activity intolerance
C Alternation of comfort
D Risk of alternation in skin integrity
E Knowledge deficit
F Decreased cardiac output
301- A patient with Alzheimer’s disease and severe cardiomyopathy presents to the hospital with
a fractured left hip. The patient is on bed rest until a cardiologist clears the patient to have
surgery. Which nursing diagnosis have the highest priority?
a- E and F
b- D and C
c- C and F
d- D and F

302- A 19 year-old woman telephones the nurse and complains of difficulty of breathing. The
symptoms developed while exercising that morning. She has a productive cough with thick
mucous secretions and wheezing. The nurse hears that the breathing problems do not
interfere with talking on the phone. The woman plans to attend the clinic but must wait for
two hours before she can be driven there. Which treatment should be recommended first by
the physician?
a- Purse-lip breathing
b- Deep breathing and coughing
c- Decrease physical activity
d- B2 agonist administration

303- A 50 year-old male presents to the medical office 3 weeks after cardiac surgery with
complaints of a feeling of weakness, difficulty breathing, and joint pains. Upon examination
the nurse finds a fever and a friction rub on auscultation of the chest. The nurse recognizes
that the MOST likely surgical complication is:
a- Neuropsychological dysfunction
b- Postpericardiotomy syndrome
c- Cardiac tamponade
d- Phrenic nerve damage

304- As identified by Dr. Elizabeth Kubler-Ross, which stage of dying is characterized by the
transition from, “NO, not me” to “Yes, me, but…”

72
a- Anger
b- Depression
c- Acceptance
d- Bargaining

305- A patient with heart failure has the following vital signs, which of these vital signs should be
reported to the physician prior to administrating the next dose of digoxin?
Blood pressure 136/84 mmhg
Heart rate 48/min
Respiratory rate 20/min
Temperature 37.1C
a- Blood pressure
b- Pulse
c- Temperature
d- Respiratory rate

306- A pediatric nurse is providing medication instructions to the mother of a 13 year-old boy
who is starting anti-depressant therapy. The mother appears unfocused, agitated and
confused and asks the nurse to repeat the instructions several times. She tells the nurse that
she is concerned that she might administer the medication incorrectly. The nurse is
concerned that the mother is not following the instructions.
Which intervention would be most appropriate?
a- Reassure her that doubts are normal
b- Reinforce the importance of correct dosage
c- Refocus the teaching with printed material
d- Take a small break and the continue

307- A child is treated for superficial (first-degree) thermal burns to the thigh. The child is in
great discomfort and does not eat.
Which of the following diagnosis should receive PRIORITY?
a- Altered nutrition
b- Impaired skin integrity
c- Risk of infection
d- Acute pain

308- A 3-week-old infant is hospitalized with jaundice. When considering the fluid needs relative
to body size of the infant as compared to the fluid needs of an adult. The nurse knows this
infant requires:
a- Less fluids
b- More fluids
c- Same amount of fluids
d- Much less fluids

73
309- A nurse obtains a urine dipstick analysis sample from a 35 year-old woman who reports
having burning sensation with urination and a sense of urgency and frequency. She had
been diagnosed with the condition six months previously and was prescribed a course of
antibiotics.
Urinalysis Results Normal values
Colour Dark yellow Straw-coloured
Odour Abnormal Almost nothing
Appearance Turbid Clear
Leukocyte esterase Positive Negative
Nitrites Positive Negative
Which type of pharmacological treatment is most likely?
a- Anti-viral
b- Anti-fungal
c- Anti-bacterial
d- Anti-parasitic

310- What is the average incubation period of Hepatitis A?


a- 30 days
b- 60 days
c- 50 days
d- 14 days

311- A client has an order to have a set of arterial blood gases (ABG's) drawn. The intended site is the
radial artery. The nurse ensures that which of the following is positive before the ABGs are
drawn
a- Homan’s sign c- Babinski reflex
b- Brudzinski's sign d- Allen's test

312- 16 years old girl tried to suicide by taking a large amount of valium (benzodiazepines), the
nurse knows that the antidote for this drug is:
a- Flumazenil c- Meperidine
b- Benztropine d- Naloxone

313- A client is scheduled for blood to be drawn from the radial artery for an ABG determination. A
nurse assists with performing Allen's test before drawing the blood to determine the adequacy of
the:
a- Ulnar circulation
b- Carotid circulation
c- Femoral circulation
d- Brachial circulation

74
314- A nurse is assigned to care for a patient with a diagnosis of thrombotic stroke. The nurse
knows that this type of stroke is MOST LIKELY caused by:
a- Blockage of large vessels as a result of atherosclerosis
b- Emboli produced from valvular heart disease
c- Decreased cerebral blood flow due to circulatory failure
d- A temporary disruption in oxygenation of the brain

315- A 65 year-old woman presented to her care provider with complaints of bright red blood in
the stool, a loss of appetite, a feeling of fullness and fatigue. She had lost 5 kilograms in the
past three weeks without dieting. A faecal occult blood test is positive and the patient is
scheduled for additional screening test. Which screening test is most likely for this patient?
a- Barium enema
b- Colonoscopy
c- Endoscopy
d- Computed tomography scan

316- A community health nurse is teaching a health class about infectious diseases process.
The nurse instructs the class that rabies would be considered which of the following type of
infection?
a- Viral c- Fungal
b- Protozoan d- Bacterial

317- A baby girl was born prematurely at 33 weeks gestation due to placenta abruption. She is
now two months old and has a potential hearing deficit. In the neonatal intensive care unit
(NICU) she had passed the initial screening tests for hearing but has significant risk factors
that suggest further evaluation. Which autonomic reflex should be further evaluated?
a- Moro
b- Babinski
c- Pupillary
d- Tonic neck

318- A surgery procedure that must be done within 24 hours is called:


a- Selective surgery
b- Elective surgery
c- Urgent surgery
d- Emergency surgery

319- The day after an amputation, the client begins to hemorrhage from his stump. What action
should the nurse take first?
a- Apply a pressure dressing to the stump
b- Place a tourniquet above the stump

75
c- Notify the physician
d- Apply an ice pack to the stump
320- Which laboratory test is most important for the nurse to monitor to determine how
effectively the client’s diabetes is being managed?
a- Fasting blood glucose
b- Blood chemistry profile
c- Complete blood count
d- Glycosylated hemoglobin (HbA1c)

321- Blood sugar for new employee at company with sedentary life style and obese and by result
of Fasting blood Sugar 6.9 mmol (abnormal reading ) what is the best next step to do to
estimate his blood sugar condition:
a- Draw blood sample for HbA1c
b- Repeat the test for the next 2 days
c- Estimate the postprandial sugar level
d- Write a recommendation note for not hiring him

322- HbA1c as a measurement instrument for glucose level in the blood, it measures blood
glucose level within:
a- Last 3 months c- Last 7 months
b- Last 5 months d- Last 1 year

323- A client receiving hemodialysis treatment arrives at the hospital. He complains of shortness
of breath, and pedal edema is noted. His last hemodialysis treatment was yesterday.
Blood pressure 200/100
Heart rate 110/min
Respiratory rate 36/m
Oxygen saturation 89% on room air
Which of the following interventions should be done first?
a- Administer oxygen
b- Elevate the foot of the bed
c- Restrict the client’s fluids
d- Prepare the client for hemodialysis

324- Which of the following is the purpose of providing atropine prior to anesthesia and surgery:
a- To decrease salivation (decrease secretions)
b- To increase blood pressure
c- To facilitate breathing
d- To decrease muscle tone

325- Which finding provides the best evidence that peritoneal dialysis is achieving a therapeutic
effect:
a- Urine output increases
76
b- Appetite improves
c- Potassium level falls
d- Red blood cell count is lower
326- The nurse is caring for a client with renal failure who is being treated with peritoneal
dialysis. Which assessment before and after peritoneal dialysis is most valuable in evaluating
the outcome of treatment:
a- Pulse rate
b- Body weight
c- Abdominal girth
d- Urine output

327- Changes in personality and judgment are often associated with a lesion in which of the
following:
a- Frontal lobe
b- Parietal lobe
c- Broca's area
d- Wernicke's area

328- Anterior lobe of the brain is responsible for which of the following:
a- Personality
b- Movement
c- Speech
d- Memory

329- The nurse would assess which laboratory value to determine the effectiveness of intravenous
heparin:
a- Complete blood count (CBC)
b- Activated partial thromboplastin time (aPTT)
c- Prothrombin time (PT)
d- BUN

330- The antidote to heparin is:


a- Aspirin c- Warfarin
b- Vitamin k d- Protamine sulfate

331- How do we monitor the effectiveness of Coumadin (Warfarin):


a- Monitor vital signs
b- PT/INR
c- Weight
d- Ask the patient how they feel

332- An 18 year-old woman who broke her right ankle is seen in the physician’s office one week
after the cast was removed.
Which of the following is the short term goal for this patient?
77
a- Walk 100 feet with crutches
b- Walk completely independent
c- Relieve the pain
d- Strict bedrest
333- A client with iron deficiency anemia is scheduled for discharge. Which instruction about
prescribed ferrous gluconate therapy should the nurse include in the teaching plan?
a- “Take the medication with an antacid.”
b- “Take the medication with a glass of milk.”
c- “Take the medication with cereal.”
d- “Take the medication on an empty stomach.”

334- MRSA (methicillin resistant Staphylococcus aureus) is transmitted by:


a- Contact c- Droplet
b- Airborne d- Standard

335- Which of the following drugs is not used in the treatment of pulmonary embolism?
a- Heparin c- Digoxin
b- Warfarin d- Streptokinase

336- 36 years old male patient complaining of fever and headache for 3 days now, when
examining this patent which of the following signs would indicate that this patient may has
meningitis:
a- Positive Kernig’s sign
b- Negative Brudzinski’s sign
c- Positive homan’s sign
d- Negative Kernig’s sign

337- A 45 year-old woman presented with a generalized rash that is not itchy. She reports that
she has had the problem for the past 15 years. Examination reveals a well-outlined, reddish
plaque over the right gluteal fold. The plaque has scales over it and is cracked in some areas.
Which intervention is initially appropriate?
a- Apply topical cream to the affected area
b- Expose area to sunlight for twenty minutes daily
c- Maintain immunosuppressant therapy regimen
d- Increase dietary intake of vitamin A

338- The patient is receiving mechanical ventilation set at fraction of inspired oxygen (FiO2)
100%.
The nurse should understand that which of the following can improve this patient’s
oxygenation?
a- Adding positive end expiratory pressure (PEEP)
b- Place the patient in trendelenburg position

78
c- Increasing the FiO2
d- Suctioning the patient hourly

339- Psychiatric patient appear violent for himself and others was put in the room alone during
the period of exacerbation, then patient calm down and informed the nurse I am ok now let
me with others, but the nurse refused that as a punishment way. At which underline Label
will nurse accused under the court:
a- Abandonment
b- False imprisonment
c- Negligence
d- Duty to act

340- Which of the following is the best position to increase the brain perfusion( cerebral perfusion):
a- Prone c- Semi- Fowler’s
b- Supine d- Trendelenburg’s

341- If the maintenance requirement of fluid for a child is 900 ml/ day, each daily feeding
account 120 ml, what is the number of feeding is required to achieve this maintenance:
a- 5 feedings
b- 6 feedings
c- 7 feedings
d- 8 feedings

342- A nurse is caring for a 3-week-old infant who just admitted to the hospital.
Which of the following nursing interventions does NOT support this infant’s basic
emotional and social needs?
a- Provide for continual contact between parents and infant
b- Actively involve parents in caring for the infant
c- Keep the infant’s environment quite, dim and free of sensory stimulation
d- Foster infant-sibling relationships as appropriate

343- The following syringe contains morphine sulphate in a concentration of 2 mg/cc, how much total
morphine this syringe contains:
a- 0.5 mg
b- 1 mg
c- 2 mg
d- 4 mg

344- The following syringe contains 2 gm of morphine sulphate, how many grams are there in 1 ml:
a- 0.5 gm

79
b- 1 gm
c- 2 gm
d- 4 gm

345- Which of the following is the incision site for an open cholecystectomy operation:
a- 1
b- 2
c- 3
d- 4

346- Which of the following is the incision site for an open gastrectomy operation:
a- 1
b- 2
c- 3
d- 4

347- Which of the following is the incision site for surgical appendectomy operation:
a- A
b- B
c- C
d- D

348- What is the most probable type of arrhythmia in the following ECG (lead ll):
a- Atrial fibrillation
b- Atrial flutter
c- Ventricular fibrillation
d- Supra-ventricular fibrillation

349- What is the most probable type of arrhythmia in the following ECG (lead V1):

80
a- Atrial fibrillation
b- Atrial flutter
c- Ventricular tachycardia
d- Supra-ventricular fibrillation

350- The following picture suggests that the patient has:


a- Renal disease
b- Hepatic disease
c- Pulmonary disease
d- Cerebral disease

351- When a patient is vomiting postoperatively, which of the following positions would the
nurse use to protect the patient?

a- c-

b- d-

352- The following picture represents the pain distribution of which nerve affection:
a- C2
b- C6
c- T3
d- T6

81
353- What is the name of the equipment this picture shows?
a- Oropharyngeal airway (OPA)
b- Nasopharyngeal airway (NPA)
c- Laryngeal mask airway (LMA)
d- Urinary catheter

354- A 24 year-old woman was prescribed loratidine (Claritin) 10 mg tablet q 12 hours a.c. for
allergy. The nurse reviews the medication order and explains to the patient the relation of
this drug to meals is as following:
a- The drug is to be taken after meals
b- The drug is to be taken during meals
c- The drug is to be taken before meals
d- The drug is to be taken away from meals

355- A patient who has required prolonged mechanical ventilation has the following arterial
blood gas results: pH 7.48, PaO2 85 mm Hg, PaCO2 32 mm Hg, and HCO3 25 mEq/L. The
nurse interprets these results as:
ABG Element Normal Value Range
a- Metabolic acidosis pH 7.4 7.35 to 7.45
b- Metabolic alkalosis Pa02 90mmHg 80 to 100 mmHg
Sa02 93 to 100%
c- Respiratory acidosis PaC02 40mmHg 35 to 45 mmHg
HC03 24mEq/L 22 to 26mEq/L
d- Respiratory alkalosis

356- A patient has the following ABG results: pH 7.32, PaO2 88 mm Hg, PaCO2 37 mm Hg, and
HCO3 16 mEq/L. The nurse interprets these results as:
a- Respiratory acidosis
b- Respiratory alkalosis
c- Metabolic acidosis
d- Metabolic alkalosis

357- Which of the following instructions are appropriate for the nurse to give the patient about
the time of taking omeprazole tablet:
a- Take the tablet during the meal
b- Take the tablet 2 hours after the meal
c- Take the tablet 30-45 minute before the meal
d- Take the tablet before bed time

358- When the nurse puts an infant with high temperature in cold water in order to lower his
temperature, she knows that the mechanism of heat loss is called:
a- Evaporation c- Radiation
82
b- Conduction d- Convection

359- Which of the following devices can deliver the highest concentration of oxygen?
a- Nasal cannula
b- Nonrebreather mask
c- Simple oxygen face mask
d- Venturi mask

360- The nurse is teaching a 45 year old woman how to increase the potassium in her diet. The
woman says she knows bananas are high in potassium, but she doesn't like their taste. What
foods should the nurse recommend the client include in her diet:
a- Carrots, broccoli, yogurt
b- Rhubarb, tofu, celery
c- Potatoes, spinach, raisins
d- Onions, corn, oatmeal

361- A nurse has administered approximately half of an enema solution to a preoperative client
when the client complains of pain and cramping. Which nursing action is the most
appropriate:
a- Raise the enema bag so that the solution can be instilled quickly.
b- Clamp the tubing for 30 seconds and restart the flow at a slower rate.
c- Reassure the client and continue the flow.
d- Discontinue the enema and notify the physician

362- An adult is admitted with heart failure. The nurse notes that he has neck vein distention
and slight peripheral edema. The nurse knows that these signs indicate which of the
following:
a- Pneumothorax
b- Right-sided heart failure
c- Cardiogenic shock
d- Left-sided heart failure

363- An adult client is admitted with a diagnosis of left-sided congestive heart failure. Which
assessment finding would most likely be present?
a- Distended neck veins
b- Dyspnea
c- Hepatomegaly
d- Pitting edema

83
364- When the nurse on duty accidentally bumps the bassinet, the neonate throws out its arms,
hands opened, and begins to cry. The nurse interprets this reaction as indicative of which of
the following reflexes:
a- Moro Reflex c- Grasping Reflex
b- Babinski reflex d- Tonic Neck Reflex

365- A nurse examines a client’s level of responsiveness. She finds that the patient opens his eyes
to verbal commands, obeys verbal commands, and is oriented to time, place, and person.
What’s the client’s Glasgow Coma Scale:
a- 11/15
b- 12/15
c- 13/15
d- 14/15

366- A 73 year old patient looks at you when you speak to her. When you ask her the date, she
says "blue." You note left-sided weakness when she grips your fingers. What is her Glasgow
Coma Score?
a- 11/15
b- 12/15
c- 13/15
d- 14/15

367- A patient during examination opens his eyes in response to pain, makes no verbal response,
but withdraws from pain. What is the Glasgow Coma Score (GCS) for this patient?
a- 3 c- 7
b- 5 d- 11

368- Which of the following pulses should be checked before administrating Digoxin:
a- Apical pulse
b- Radial pulse
c- Femoral pulse
d- Dorsalis pedis pulse

84
369- which of the following interventions must the nurse take when administrating digoxin to the
patient:
a- Give him the medication with a glass of orange juice
b- Check him for signs of hypokalemia before giving the medication
c- Instruct him to place the medication under the tongue
d- Withhold the medication if his pulse is less than 60 beats/ minute

370- The nurse must withhold Digoxin from a patient if his pulse rate is:
a- Less than 45/m
b- More than 60/m
c- Less than 60/m
d- More than 100/m

371- A newly admitted client takes digoxin 0.25 mg/day. The nurse knows that the serum therapeutic
range for digoxin is:
a- 0.1 to 1.5 ng/mL
b- 0.5 to 2.0 ng/mL
c- 1.0 to 2.5 ng/mL
d- 2.0 to 4.0 ng/mL

372- A patient who has been anticoagulated with warfarin (Coumadin) is admitted
with gastrointestinal bleeding. The nurse will anticipate administering which
substance: a- Vitamin E c- Protamine sulfate
b- Vitamin K d- Calcium gluconate

373- If the client develops a thrombus in one of the leg veins, which client response would the nurse
expect when eliciting Homans’ sign?
a- Sharp, immediate calf pain
b- Sudden numbness in the foot
c- Inability to bend the knee when asked
d- Tingling throughout the affected leg

85
374- A nurse in the delivery room is assisting with the delivery of a newborn infant. After the
delivery of the newborn, the nurse assists in delivering the placenta. Which observation
would indicate that the placenta has separated from the uterine wall and is ready for
delivery?
a- The umbilical cord shortens in length and changes in color
b- A soft and boggy uterus
c- Maternal complaints of severe uterine cramping
d- A sudden gush of dark blood from the introitus

375- Which of the following is the end of the first stage of labor:
a- Cervix dilated to 10 cm
b- Crowning of the presenting part
c- Increased bloody show
d- Contractions lasting up to 60 seconds

376- which type of isolation category is indicated for a burn patient:


a- Airborne isolation
b- Strict isolation
c- Reverse isolation
d- No isolation required

377- Which of the following is the best breathing pattern you should teach a client with COPD:
a- Pursed-lip breathing c- Abdominal breathing
b- Deep breathing d- Slow lite breathing

378- A nurse instructs a female client to use the pursed-lip method of breathing and the client
asks the nurse about the purpose of this type of breathing. The nurse responds, knowing that
the primary purpose of pursed-lip breathing is to:
a- Promote oxygen intake
b- Strengthen the diaphragm
c- Strengthen the intercostal muscles
d- Promote carbon dioxide elimination

379- A client who has had an above-the-knee amputation develops a dime-sized bright red spot
on the dressing after 45 minutes in the postanesthesia recovery unit. The nurse should:
a- Elevate the stump
b- Reinforce the dressing
c- Call the surgeon
d- Draw a mark around the site

86
380- The nurse is caring for a client who has had a chest tube inserted and connected to water seal
drainage. The nurse determines the drainage system is functioning correctly when which of the
following is observed:
a- Continuous bubbling in the water seal chamber
b- Fluctuation in the water seal chamber
c- Suction tubing attached to a wall unit
d- Vesicular breath sounds throughout the lung fields

381- The physician ordered to give the patient 10,000 unit of heparin, the preparation available is
40,000/ml. how much would the nurse give this patient:
a- 4 ml c- 0.25 ml
b- 2.5 ml d- 25 ml

382- A nurse is administering a cleansing enema to a client with a fecal impaction. Before
administering the enema, the nurse places the client in which position:
a- Left Sims’ position
b- Right Sims’ position
c- On the left side of the body, with the head of the bed elevated 45 degrees
d- On the right side of the body, with the head of the bed elevated 45 degrees

383- After the client undergoes a total hip replacement, how should the nurse position the affected
hip?
a- Adduct the hip c- Flex the hip
b- Abduct the hip d- Extend the hip

384- The following picture represents a newborn reflex known as?


a- Rooting reflex
b- Moro reflex
c- Grasping reflex
d- Startle reflex

385- The nurse is teaching a patient with diabetes mellitus about self-administration of insulin. Which
syringe angle is correct when administering insulin?
a- A
b- B
c- C

87
d- D
386- …. He does not swallow and excessive amounts of saliva run out of his mouth.
Blood pressure 91/47 mmHg
Heart rate 130/min
Respiratory rate 38/min
Body temperature 38.5 c tympanic

Which immunization could have most likely prevented the illness?


a- Pneumococcal
b- Haemophilus influenza type B.
c- Measles, mumps, rubella.
d- Meningococcal.

387- The nurse assesses a patient who is 16-weeks pregnant. The patient states that she had taken
isotretinoin (Accutane) , a known teratogen for acne during her third , fourth , and fifth week of
pregnancy According to the chart , the nurse CAN expect fetal damage to the central nervous
system as well as the :
a- Palate and ears.
b- Heart, lower limbs, and palate.
c- Limbs, eyes, and teeth.
d- Heart, eyes, and limbs.

388- …. Bleeding that has saturated the patient's underclothing and bed sheets. She appears agitated
and restless packed red blood cells and colloids are ordered.
Blood pressure 86/40 mmHg
Heart rate 110/min
Respiratory rate 24/min
Temperature 37.3 c
Oxygen saturation 90 % on room air
Which additional clotting component should be administered?
a- Fresh frozen plasma.
b- Albumin.
c- Washed red blood cells.
d- Vitamin K.
88
389- Has had difficulty sleeping and has lost ten kilograms despite having a large appetite on
examination there is a palpable thyroid gland.
Blood pressure 108/58 mmHg
Heart rate 116/min
Respiratory rate 22/min
Body temperature 38.0 c oral
Height 164
Weight 50 kilograms
Which additional symptom is most likely?
a- Heart palpitations.
b- Depression.
c- Anorexia.
d- Paresthesia.

390- … Proper procedure for doing a breast self-exam?


a- Use the palm of the hand to feel for lumps.
b- Apply three different levels of pressure to feel breast tissue.
c- Stand when performing a breast self-exam.
d- Perform self-exam annually.

391- Nurse prepares to delegate tasks to the nursing assistant Among her patients is a 50 year-old
woman who is day two of recovery following a laparoscopic resection of the colon post-operative
orders are follow :
Ambulate every six hours.
Evaluate vital signs every two hours.
Lactated Ringer's IV at 50 ml/hour.
Wound assessment every eight hours.
Nasogastric tube until bowel sounds present.
Which is most appropriate to delegate?

a- Ambulate the patient.


b- Evaluation of vital signs.
c- Change intravenous fluid bags.
d- Assess nasogastric tube placement.

392- Which of the following is a desired expected outcome 24 hours postoperatively?

a- Gag reflex present.


b- Cerebral perfusion pressure, 68mmHg
c- Intracranial pressure, 21mmHg.
d- Decreased lacrimation.

89
393- RTA. The patient appears restless confused and disoriented. He reports that he had hit his head
against the steering wheel of the car when it had collided with the car directly in front of him.
Blood pressure 110/68 mmHg
Heart rate 100/min
Respiratory rate 22/min
Body Temperature 37.0 c oral
Oxygen saturation 98 % on room air
What is the most important next step in management?

a- Immobilize head and neck.


b- Administer oxygen.
c- Establish an intravenous line.
d- Arrange for an MRI scan.

A B C D

394- The nurse is teaching a patient with diabetes mellitus about self-administration of
insulin. Which syringe angle is correct when administering insulin?
a- A
b- B
c- C
d- D

90
395- The nurse assesses a patient who is 16-weeks pregnant. The patient states that she
had taken isotretinoin (Accutane) , a known teratogen for acne during her third ,
fourth , and fifth week of pregnancy According to the chart , the nurse CAN expect
fetal damage to the central nervous system as well as the :
a- Palate and ears.
b- Heart, lower limbs, and palate.
c- Limbs, eyes, and teeth.
d- Heart, eyes, and limbs.

396- RTA. The patient appears restless confused and disoriented. He reports that he
had hit his head against the steering wheel of the car when it had collided with the
car directly in front of him.
Blood pressure 110/68 mmHg
Heart rate 100/min
Respiratory rate 22/min
Body Temperature 37.0 c oral
Oxygen saturation 98 % on room air
What is the most important next step in management?
a- Immobilize head and neck.
b- Administer oxygen.
c- Establish an intravenous line.
d- Arrange for an MRI scan.

397- Which of the following is a desired expected outcome 24 hours postoperatively?


a- Gag reflex present.
b- Cerebral perfusion pressure, 68mmHg
c- Intracranial pressure, 21mmHg.
d- Decreased lacrimation.

91
398- What is the Proper procedure for doing a breast self-exam?
a- Use the palm of the hand to feel for lumps.
b- Apply three different levels of pressure to feel breast tissue.
c- Stand when performing a breast self-exam.
d- Perform self-exam annually

399- A 45 year-old patient has had difficulty sleeping and has lost ten kilograms despite
having a large appetite on examination there is a palpable thyroid gland.
Blood pressure 108/58 mmHg
Heart rate 116/min
Respiratory rate 22/min
Body temperature 38.0 c oral
Height 164
Weight 50 kilograms
Which additional symptom is most likely?
a- Heart palpitations.
b- Depression.
c- Anorexia.
d- Paresthesia

400- Nurse prepares to delegate tasks to the nursing assistant Among her patients is a
50 year-old woman who is day two of recovery following a laparoscopic resection
of the colon post-operative orders are follow :
Ambulate every six hours.
Evaluate vital signs every two hours.
Lactated Ringer's IV at 50 ml/hour.
Wound assessment every eight hours.
Nasogastric tube until bowel sounds present.
Which is most appropriate to delegate?
a- Ambulate the patient.
b- Evaluation of vital signs.
c- Change intravenous fluid bags.
d- Assess nasogastric tube placement.

92
‫عزمي ‪2‬‬
1- Which of the following anatomical terms best describes the motion in the picture
above:
a- Adduction
b- Abduction
c- Supination
d- Pronation

2- Which of the above locations represents the best sites to inspect chest retractions in a
child with lower respiratory tract disorder:
a- A&B
b- B&C
c- C&D
d- D&E
1
3- Your patient's finger stick blood sugar at 07:30 is 300 mg/dl, according to the
following sliding scale; how many unites you would give your patient:
a- 6 i.u
b- 8 i.u
c- 14 i.u
d- 18 i.u

4- The nurse is caring for a diabetic child whose 11 a.m. blood glucose monitoring
check is 269 mg/dL. The physician orders the following coverage schedule:

150 to 200 mg/dL—2 units of Humulin R


201 to 250 mg/dL—4 units of Humulin R
251 to 300 mg/dL—6 units of Humulin R
301 to 350 mg/dL—8 units of Humulin R
351 to 399 mg/dL—10 units of Humulin R
Over 400 mg/dL—call the physician

According to the coverage schedule; how many unites you would give your patient:
a- 4 units
b- 6 units
c- 8 units
d- 10 units

2
5- The following syringe contains morphine sulphate in a concentration of 2 mg/cc, how
much total morphine this syringe contains:
a- 0.5 mg
b- 1 mg
c- 2 mg
d- 4 mg

6- The following syringe contains 2 gm of morphine sulphate, how many grams are
there in 1 ml:
a- 0.5 gm
b- 1 gm
c- 2 gm
d- 4 gm

7- The following condition may develop in females due to an increase in the production
of which of the following hormones:
a- Androgens
b- Progesterone
c- Estrogen
d- Growth

8- What is the name of The process of movement of a dissolved substance (solutes)


from an area of high concentration to an area of low concentration:
a- Effusion
b- Diffusion
c- Emulsion
d- Excretion
3
9- Which of the following is the incision site for an open cholecystectomy operation:
a- 1
b- 2
c- 3
d- 4

10- Which of the following is the incision site for an open gastrectomy operation:
a- 1
b- 2
c- 3
d- 4

11- Which of the following is the incision site for surgical appendectomy operation:
a- A
b- B
c- C
d- D

4
12- What is the most probable type of arrhythmia in the following ECG (lead ll):
a- Atrial fibrillation
b- Atrial flutter
c- Ventricular fibrillation
d- Supra-ventricular fibrillation

13- What is the most probable type of arrhythmia in the following ECG (lead V1):
a- Atrial fibrillation
b- Atrial flutter
c- Ventricular tachycardia
d- Supra-ventricular fibrillation

14- The following picture suggests that the patient has:


a- Renal disease
b- Hepatic disease
c- Pulmonary disease
d- Cerebral disease

15- A patient with colostomy, what type of stool you should suspect:
a- With a sigmoid colostomy, the feces are formed
b- With a descending colostomy, the feces are semiformed
c- With a transverse colostomy, the feces are unformed
d- With an ascending colostomy, the feces are fluid

5
16- When a patient is vomiting postoperatively, which of the following positions
would the nurse use to protect the patient?

a- c-

b- d-

17- The following picture represents the pain distribution of which nerve affection:
a- C2
b- C6
c- T3
d- T6

18- On assessing the client’s pressure ulcer (see photo), the nurse would document
this as which stage:
a- Stage I
b- Stage II
c- Stage III
d- Stage IV

6
19- The following picture represents an auto immune disease known as:
a- Psoriasis
b- Systemic Lupus Erythematosus
c- Molluscum Contagiosum
d- Eczema

20- What is the name of the equipment this picture shows?


a- Oropharyngeal airway (OPA)
b- Nasopharyngeal airway (NPA)
c- Laryngeal mask airway (LMA)
d- Urinary catheter

21- A client is being prepared for a thoracentesis. A nurse assists the client to which
position for the procedure:
a- Lying in bed on the affected side
b- Lying in bed on the unaffected side
c- Sims’ position with the head of the bed flat
d- Prone with the head turned to the side and supported by a pillow

7
22- A nurse is preparing to insert a nasogastric tube into a client. The nurse places the
client in which position for insertion:
a- Right side c- High Fowler’s
b- Low Fowler’s d- Supine with the head flat

23- A nurse develops a plan of care for a client with deep vein thrombosis. Which client
position or activity in the plan will be included:
a- Out-of-bed activities as desired
b- Bedrest with the affected extremity kept flat
c- Bedrest with elevation of the affected extremity
d- Bedrest with the affected extremity in a dependent position

24- A client in labor is transported to the delivery room and is prepared for a cesarean
delivery. The client is transferred to the delivery room table, and the nurse places
the client in the:
a- Trendelenburg's position with the legs in stirrups
b- Semi-Fowler position with a pillow under the knees
c- Prone position with the legs separated and elevated
d- Supine position with a wedge under the right hip

25- The nurse is caring for a client who is 1 day postoperative for a total hip
replacement. Which is the best position in which the nurse should place the client:
a- Side-lying on the operative side
b- On the nonoperative side with the legs abducted
c- Side-lying with the affected leg internally rotated
d- Side-lying with the affected leg externally rotated

8
26- Which of the following is the best position to increase the brain perfusion( cerebral
perfusion):
a- Prone c- Semi- Fowler’s
b- Supine d- Trendelenburg’s

27- A nurse is administering a cleansing enema to a client with a fecal impaction.


Before administering the enema, the nurse places the client in which position:
a- Left Sims’ position
b- Right Sims’ position
c- On the left side of the body, with the head of the bed elevated 45 degrees
d- On the right side of the body, with the head of the bed elevated 45 degrees

28- A client has just returned to a nursing unit after an above-knee amputation of the
right leg. A nurse places the client in which position:
a- Prone
b- Reverse Trendelenburg’s
c- Supine, with the amputated limb flat on the bed
d- Supine, with the amputated limb supported with pillows

29- A nurse is caring for a client with a severe burn who is scheduled for an autograft
to be placed on the lower extremity. The nurse develops a postoperative plan of care
for the client and includes which of the following in the plan:
a- Maintain the client in a prone position
b- Elevate and immobilize the grafted extremity
c- Maintain the surgical extremity in a flat position
d- Keep the surgical extremity covered with a blanket

9
30- A nurse is preparing to care for a client who has returned to the nursing unit
following cardiac catheterization performed through the femoral artery. The nurse
checks the physician’s prescription and plans to allow which client position or
activity following the procedure:
a- Bedrest in high Fowler’s position
b- Bedrest with bathroom privileges only
c- Bedrest with head elevation at 60 degrees
d- Bedrest with head elevation no greater than 30 degrees

31- After the client undergoes a total hip replacement, how should the nurse position
the affected hip?
a- Adduct the hip c- Flex the hip
b- Abduct the hip d- Extend the hip

32- A client has an open reduction and internal fixation for a fractured hip.
Postoperatively the nurse should position the client's affected extremity in:
a- External rotation
b- Slight hip flexion
c- Moderate abduction
d- Anatomical body alignment

33- Which of the following positions should the nurse place a client for rectal tube
insertion:
a- left lateral position with hyperextension of right knee
b- left lateral position with hyperflextion of left knee
c- left lateral position with hyperflextion of right knee
d- left lateral position with hyperextension of left knee

10
34- Which of the following position is appropriate for the patient to use for self-
administration of fleet enema:
a- Left lateral position right leg flex
b- Face down and leg flex
c- Face upward and leg free (on back and flex hip and knee)
d- Chest- knee position

35- The best position for a child with myelomeningocele is?


a- Prone
b- supine
c- semi-fowler’s
d- modified trendelenburg

36- Unless the physician orders otherwise, in which position should the nurse place the
infant during the postoperative period of surgical repair of the myelomeningocele:
a- Supine
b- Prone
c- Right or left side-lying
d- Whichever position is most comfortable for the infant

37- The nurse is caring for a client with an above-the-knee amputation (AKA). To
prevent contractures, the nurse should:
a- Place the client in a prone position 15–30 minutes twice a day
b- Keep the foot of the bed elevated on shock blocks
c- Place trochanter rolls on either side of the affected leg
d- Keep the client’s leg elevated on two pillows

11
38- Postoperative nursing care of the infant following surgical repair of a cleft lip
would include:
a- Feeding the infant with a spoon to avoid sucking
b- Positioning the infant on the abdomen to facilitate drainage
c- Applying elbow restraints to protect the surgical area
d- Providing minimal stimulation to prevent injury to the incision

39- The client has returned to the nursing unit following a right below-the-knee
amputation. How should the nurse position the client?
a- Supine with head turned to the side
b- With shock blocks placed under the foot of the bed
c- Semi-sitting position with knees bent
d- Left lateral with pillows between the knees

40- What position should the nurse place the head of the bed in to obtain the most
accurate reading of jugular vein distention:
a- High fowler’s
b- Raised 10 degrees
c- Raised 30 degrees
d- Supine position

41- Which of the following body positions is best for a patient with increased ICP:
a- Prone
b- Trendelenburg
c- HOB elevated 90 degrees and hips flexed
d- HOB elevated 20 degrees

12
42- Immediately after a percutaneous liver biopsy, the nurse should place the client in
which of the following positions:
a- Left- sided lying
b- Right- sided lying
c- Lithotomy
d- Trendelenburg

43- An adult has low back pain. Which position is likely to be most comfortable for the
client:
a- Prone
b- Supine
c- Side-lying with knees flexed
d- Semi-sitting with legs extended

44- When planning the client’s postoperative care, which is the least desirable position
in which the nurse can place the client:
a- Lying prone
b- Lying supine
c- Sitting in a chair
d- Standing to shower

45- Which of the following is the appropriate position for a patient who is suffering
from acute asthma attack is:
a- Right lateral
b- Supine
c- High fowler
d- Prone
13
46- A patient with COPD is admitted to the hospital. How can the nurse best position
the patient to improve gas exchange:
a- Sitting up at the bedside in a chair and leaning slightly forward
b- Resting in bed with the head elevated to 45 to 60 degrees
c- In the Trendelenburg's position with several pillows behind the head
d- Resting in bed in a high-Fowler's position with the knees flexed

47- Following a tympanoplasty, the nurse should maintain the client in which
position:
a- Semi-Fowler’s with the operative ear facing down
b- Low Trendelenburg’s with the head in neutral position
c- Flat with the head turned to the side with the operative ear facing up
d- Supine with a small neck roll to allow for drainage

48- Which of the following is the proper position for a patient undergoing a
cardiac surgery:
a- Supine
b- Prone
c- Lateral
d- Fowler’s

49- A nurse is caring for a toddler after surgical repair of a cleft palate. The nurse
should position the child:
a- On his back
b- On his stomach
c- On his back with his head slightly elevated
d- For comfort
14
50- The nurse observes that the infant's anterior fontanelle is bulging after
placement of a ventriculoperitoneal shunt. The nurse positions this infant:
a- Prone, with the head of the bed elevated
b- Supine, with the head flat
c- Side-lying on the operative side
d- In a semi-fowler's position

51- A nurse is providing care for a client following surgery to remove a cataract
from the right eye. In which position should the nurse place the client?
a- Right-side lying c- Supine
b- Prone d- Trendelenburg’s

52- An adult is admitted for a neurological workup and is scheduled for a spinal
tap. When preparing the client for the procedure, the nurse should position the
client in which position:
a- Prone
b- On the side with knees drawn up to chest
c- Lithotomy
d- Semi-sitting

53- When suctioning the oropharynx, which of the following is the proper position
of the patient:
a- Prone
b- Supine
c- Semi- Fowler’s
d- Trendelenburg’s

15
54- To prevent headache after spinal anesthesia the patient should be positioned:
a- Semi- fowler’s
b- Flat on bed for 6 to 8 hours
c- Prone position
d- Modified Trendelenburg

55- If a patient develops autonomic hyperreflexia the first action his caring nurse
should take is to:
a- Elevate head of bed 90 degree (move from supine to sitting)
b- Make the bed in flat position
c- Apply ice on the axillary and groin
d- But the patient in trendelenburg position

56- To protect susceptible patients in the hospital from aspiration pneumonia, the
nurse will plan to:
a- Turn and reposition immobile patients at least every 2 hours.
b- Position patients with altered consciousness in lateral positions.
c- Monitor frequently for respiratory symptoms in patients who are immunosuppressed.
d- Provide for continuous subglottic aspiration in patients receiving enteral feedings.

57- Which nursing action is essential when providing continuous enteral feeding?
a- Elevating the head of the bed
b- Positioning the patient on the left side
c- Warming the formula before administrating it
d- Hanging a full day’s worth formula at one time

16
58- Which of the following positions is appropriate for the patient who has
hypovolemic shock:
a- Prone
b- Supine
c- Semi-fowler’s
d- Flat with elevating the feet

59- When caring for a patient admitted post-stroke (CVA) who has altered
consciousness, the nurse should place the patient in which position:
a- Side-lying c- Prone
b- Supine d- Semi-fowler's

60- A nurse is providing instructions to a client and the family regarding home care
after right eye cataract removal. Which statement by the client would indicate an
understanding of the instructions:
a- “I should not sleep on my left side.”
b- “I should not sleep on my right side.”
c- “I should not sleep with my head elevated.”
d- “I should not wear my glasses at any time.”

61- The day after an amputation, the client begins to hemorrhage from his stump.
What action should the nurse take first:
a- Apply a pressure dressing to the stump
b- Place a tourniquet above the stump
c- Notify the physician
d- Apply an ice pack to the stump

17
62- A 72-year-old male client has a total hip replacement for long-standing
degenerative bone disease of the hip. When assessing this client postoperatively,
the nurse considers that the most common complication of hip surgery is:
a- Pneumonia c- Wound infection
b- Hemorrhage d- Pulmonary embolism

63- The nurse is caring for a client who had a right below-the-knee amputation
three days ago. The client complains of pain in the right foot and asks for pain
medication. What nursing action is appropriate initially?
a- Elevate the stump
b- Administer a placebo
c- Administer ordered medications
d- Encourage the client to discuss his feelings

64- During the assessment of an injury for a possible fracture, which of the
following manifestations would make you suspicious that a fracture has
occurred:
a- Impaired sensation c- Loss of function
b- Hotness d- Pointed tenderness

65- A client who has had an above-the-knee amputation develops a dime-sized


bright red spot on the dressing after 45 minutes in the postanesthesia recovery
unit. The nurse should:
a- Elevate the stump
b- Reinforce the dressing
c- Call the surgeon
d- Draw a mark around the site
18
66- How you will interfere to relieve swelling of the lower extremity in a cast for 3 days
now:
a- Change the cast c- Elevate the extremity
b- Call the doctor d- Give massage

67- The nurse is developing a bowel-retraining plan for a client with multiple sclerosis.
Which measure is likely to be least helpful to the client:
a- Limiting fluid intake to 1000mL per day
b- Providing a high-roughage diet
c- Elevating the toilet seat for easy access
d- Establishing a regular schedule for toileting

68- A diabetic patient with foot gangrene undergone above knee amputation, while the
nurse changing the dressing he complains of pain on the same knee which was
amputated. What should the nurse do:
a- Inform physician about it
b- Re-do dressing to Assessment the wound
c- Psychiatry consultation to the patient because he wound above knee
d- Give analgesic as needed

69- A diabetic patient with foot gangrene undergone above knee amputation he
complain of pain and swelling at the wound site which is oozing bus and has a bad
odor, the physician ordered cephalexin and metronidazole. Which of the following
you should do immediately:
a- Give cephalexin direct first action
b- Give metronidazole direct first action
c- Do wash on wound with N/Sand put bacitracin
d- Do wash on wound with N/Sand put hydrocortisone
19
70- The physician has prescribed a cleansing enema to a client scheduled for colon
surgery. The nurse would place the client:
a- Prone
b- Supine
c- Left sim’s ( left lateral )
d- Dorsal recumbent

71- At which side of the patient will the nurse stand when inserting a rectal enema:
a- Right side
b- Left side
c- Any side
d- Both sides

72- While undergoing a soapsuds enema, the client complains of abdominal cramping.
The nurse should:
a- Immediately stop the infusion
b- Lower the height of the enema container
c- Advance the enema tubing 2 to 3 inches
d- Clamp the tubing

73- The nurse must administer an enema to an adult patient with constipation. Which
of the following would be a safe and effective distance for the nurse to insert the
tubing into the patient's rectum:
a- 1:2 cm c- 5.5:6.5 cm
b- 3:4 cm d- 6.5:8 cm

20
74- The nurse is caring for a patient receiving IV furosemide (Lasix) 40 mg and
Enalapril (Vasotec) 5 mg PO bid for ADHF with severe orthopnea. When evaluating
the patient response to the medications, the best indicator that the treatment has
been effective is:
a- Weight loss of 2 pounds overnight.
b- Improvement in hourly urinary output.
c- Reduction in systolic bp.
d- Decreased dyspnea with the head of the bed at 30 degrees.

75- The nurse in preparing to insert RYLE’S tube (NGT) into an infant, the nurse
knows that the length of the tube should be taken as following:
a- From the nose down to the chin and then to the umbilicus
b- From the nose to the earlobe and then to the xiphoid process
c- From the nose to the mouth to the xiphoid process
d- From the nose to the earlobe to the umbilicus

76- The nurse is caring for a client who has had a chest tube inserted and connected to
water seal drainage. The nurse determines the drainage system is functioning
correctly when which of the following is observed:
a- Continuous bubbling in the water seal chamber
b- Fluctuation in the water seal chamber
c- Suction tubing attached to a wall unit
d- Vesicular breath sounds throughout the lung fields

21
77- The nurse is caring for a client who has just had a chest tube attached to a water
seal drainage system. To ensure that the system is functioning effectively the nurse
should:
a- Observe for intermittent bubbling in the water seal chamber
b- Flush the chest tubes with 30-60 ml of NSS every 4-6 hours
c- Maintain the client in an extreme lateral position
d- Strip the chest tubes in the direction of the client

78- The nurse enters the room of a client who has a chest tube attached to a water seal
drainage system and notices the chest tube is dislodged from the chest. The most
appropriate nursing intervention is to:
a- Notify the physician
b- Insert a new chest tube
c- Cover the insertion site with petroleum gauze
d- Instruct the client to breathe deeply until help arrives

79- which type of isolation category is indicated for patient with tuberculosis:
a- Airborne isolation c- Reverse isolation
b- Strict isolation d- Contact isolation

80- While attempting to get out of bed, a patient accidentally disconnects the chest tube
from the Pleur-evac drainage system. Which of the following actions should the
nurse take first?
a- Insert the end of the chest tube in a container of sterile solution
b- Clamp the chest tube near the Pleur-evac drainage system
c- Raise the end of the chest tube above the level of the insertion of the chest tube
d- Apply pressure dressing to the chest tube insertion site

22
81- which type of isolation category is indicated for a burn patient:
a- Airborne isolation
b- Strict isolation
c- Reverse isolation
d- No isolation required

82- which type of isolation category is indicated for patient with diphtheria:
a- Airborne c- Contact
b- Droplet d- Blood

83- A 68-years-old woman diagnosed with thrombocytopenia due to acute lymphocytic


leukemia is admitted to the hospital. The nurse should assign the patient to a:
a- To a private room so she will not infect other patients and health care workers.
b- To a private room so she will not be infected by other patients and health care
workers.
c- To a semiprivate room so she will have stimulation during her hospitalization.
d- To a semiprivate room so she will have the opportunity to express her feelings about
her illness.

84- A nurse who begins to administer medications to a client via a nasogastric feeding
tube suspects that the tube has become clogged. The nurse should take which safe
action first:
a- Aspirate the tube
b- Flush the tube with warm water
c- Prepare to remove and replace the tube
d- Flush with a carbonated liquid such as cola

23
85- Which of the following instructions is appropriate for the nurse to give to a female
client a who complains of abdominal upset after cholecystectomy operation:
a- Increase fluid intake
b- Avoid fatty meals
c- Increase protein intake
d- Daily exercise

86- Which of the following pulses should be checked before administrating Digoxin:
a- Apical pulse
b- Radial pulse
c- Femoral pulse
d- Dorsalis pedis pulse

87- which of the following interventions must the nurse take when administrating
digoxin to the patient:
a- Give him the medication with a glass of orange juice
b- Check him for signs of hypokalemia before giving the medication
c- Instruct him to place the medication under the tongue
d- Withhold the medication if his pulse is less than 60 beats/ minute

88- The nurse must withhold Digoxin from a patient if his pulse rate is:
a- Less than 45/m
b- More than 60/m
c- Less than 60/m
d- More than 100/m

24
89- The nurse is assessing the client for possible evidence of digitalis toxicity. The nurse
acknowledges that which is included in the signs and symptoms for digitalis toxicity:
a- Pulse (heart) rate of 100 beats/min
b- Pulse of 72 with an irregular rate
c- Pulse greater than 60 beats/min and irregular rate
d- Pulse below 60 beats/min and irregular rate

90- A newly admitted client takes digoxin 0.25 mg/day. The nurse knows that the
serum therapeutic range for digoxin is:
a- 0.1 to 1.5 ng/mL
b- 0.5 to 2.0 ng/mL
c- 1.0 to 2.5 ng/mL
d- 2.0 to 4.0 ng/mL

91- A patient complains of severe pain which he stated to be 9/10, the physician
ordered morphine 50 mg IV every 4 hours, the last dose was given 2 hours ago, what
is the best action his caring nurse would take:
a- Give another dose of morphine
b- Inform the doctor to change the order
c- Distract the patient by TV, radio or games for 2 hours
d- Ignore the patient completely

92- The nurse is caring for a patient admitted 1 week ago with an acute spinal cord
injury. Which of the following assessment findings would alert the nurse to the
presence of autonomic dysreflexia (hyperreflexia)?
a- Tachycardia c- Hot, dry skin
b- Hypotension d- Throbbing headache

25
93- Which of the following is the best way for long term feeding for a patient
experiencing severe dysphagia:
a- NGT
b- Naso-dudenal tube
c- Gastrostomy
d- Parenteral

94- Patient records are as following, he had 650cc of IV fluid (n/s), 50 ml of orange
Juice in the NG tube, 100 water with the medications, and urinate 500 cc, and 100 cc
collected in the NG bag. What is the total intake for this patient?
a- 800 cc c- 200 cc
b- 650 cc d- 600 cc

95- The physician ordered to give the patient 10,000 unit of heparin, the preparation
available is 40,000/ml. how much would the nurse give this patient:
a- 4 ml c- 0.25 ml
b- 2.5 ml d- 25 ml

96- The nurse is caring for a client with a fracture. The client develops a deep vein
thrombosis in the opposite extremity. Physician orders include a heparin drip of
D5W 250 mL with heparin 12,500 units at 16 mL/hr. How many units of heparin is
this client receiving per hour:
a- 200 unit/h
b- 400 unit/h
c- 600 unit/h
d- 800 unit/h

26
97- A patient suffers from diarrhea and muscle weakness, the nurse would assess this
patient for which of the following abnormal laboratory blood tests:
a- Blood calcium c- Blood potassium
b- Blood magnesium d- Blood sodium

100- When assessing for therapeutic effects of mannitol, the nurse would expect to see:
a- Decreased intracranial pressure
b- Decreased excretion of therapeutic medications
c- Increased urine osmolality
d- Decreased serum osmolality

101- Which drug would be used to treat a patient who has increased intracranial
pressure (ICP) resulting from head trauma after an accident:
a- Mannitol
b- Atropine sulfate
c- Epinephrine hydrochloride
d- Sodium bicarbonate

102- Which of the following is the drug of choice for treatment of cerebral edema:
a- Mannitol
b- Atropine sulfate
c- Epinephrine hydrochloride
d- Sodium bicarbonate

27
103- The nurse would assess which laboratory value to determine the effectiveness of
intravenous heparin:
a- Complete blood count (CBC)
b- Activated partial thromboplastin time (aPTT)
c- Prothrombin time (PT)
d- BUN

104- The antidote to heparin is:


a- Aspirin c- Warfarin
b- Vitamin k d- Protamine sulfate

105- How do we monitor the effectiveness of Coumadin (Warfarin):


a- Monitor vital signs
b- PT/INR
c- Weight
d- Ask the patient how they feel

106- Which medication is used to treat a patient suffering from severe adverse effects of
a narcotic analgesic:
a- Naloxone (Narcan)
b- Acetylcysteine (Mucomyst)
c- Methylprednisolone (Solu-Medrol)
d- Protamine sulfate

107-AA patient who has been anticoagulated with warfarin (Coumadin) is admitted
with gastrointestinal bleeding. The nurse will anticipate administering which
substance:
a- Vitamin E c- Protamine sulfate
b- Vitamin K d- Calcium gluconate
28
108- While admitting a patient for treatment of an acetaminophen overdose, the nurse
prepares to administer which of the following medications to prevent toxicity:
a- Naloxone (Narcan)
b- Acetylcysteine (Mucomyst)
c- Methylprednisolone (Solu-Medrol)
d- Vitamin K

109- If the nurse gave a patient antihypertensive drug that is prescribed for another
patient, what should she do:
a- Ignore what she has done
b- Call urgent CPR
c- Document that in nursing report
d- Inform doctor after 2 days

110- When a physician orders an arterial blood gas, which artery is the appropriate one
to use to obtain the specimen:
a- Cerebral artery
b- Radial artery
c- Carotid artery
d- Ulnar artery

111-AA client has an order to have a set of arterial blood gases (ABG's) drawn. The
intended site is the radial artery. The nurse ensures that which of the following is
positive before the ABGs are drawn
a- Homan’s sign c- Babinski reflex
b- Brudzinski's sign d- Allen's test

29
112-AA client is scheduled for blood to be drawn from the radial artery for an ABG
determination. A nurse assists with performing Allen's test before drawing the
blood to determine the adequacy of the:
a- Ulnar circulation
b- Carotid circulation
c- Femoral circulation
d- Brachial circulation

113-AA nurse has administered approximately half of an enema solution to a


preoperative client when the client complains of pain and cramping. Which
nursing action is the most appropriate:
a- Raise the enema bag so that the solution can be instilled quickly.
b- Clamp the tubing for 30 seconds and restart the flow at a slower rate.
c- Reassure the client and continue the flow.
d- Discontinue the enema and notify the physician

114- When caring for a client who is receiving total parenteral nutrition (TPN), which
of the following complications would be most important for the nurse to assess:
a- Chest pain
b- Hemorrhage and air embolus
c- Pneumonia and hyperglycemia
d- Electrolyte imbalance and sepsis

115-AA client has a nasogastric tube after a gastric resection. The nurse should
expectto observe:
a- Vomiting
b- Gastric distention
c- Intermittent periods of diarrhea
d- Bloody drainage for the first 12 hours
30
116- While assessing a client with hypoparathyroidism, the nurse taps the client's facial
nerve and observes twitching of the mouth and tightening of the jaw. The nurse
would document this finding as which of the following:
a- Positive Trousseau's sign
b- Positive Chvostek's sign
c- Tetany
d- Hyperactive deep tendon reflex

levels are very high:


a- Hyperactivity
b- Blurred vision
c- Oliguria
d- Increased energy

hyperthyroidism:
a- Tetany
b- Buffalo hump
c- Exophthalmos
d- Striae

119-AA client is receiving long-term treatment with high-dose corticosteroids. Whichof


the following would the nurse expect the client to exhibit:
a- Weight loss
b- Pale thick skin
c- Hypotension
d- Moon face
31
120- After undergoing a thyroidectomy, a client develops hypocalcemia and tetany.
Which electrolyte should the nurse anticipate administering:
a- Sodium phosphorus
b- Sodium bicarbonate
c- Calcium gluconate
d- Potassium chloride

121- The nurse who elicits a positive Chvostek’s sign would suspect that the patient has
which condition:
a- Hyperkalemia
b- Hypocalcemia
c- Hypercalcemia
d- Hypernatremia

122- Which of the following is the best method to decrease confusion and irritability
for asthmatic patient:
a- Give antibiotic
b- Give sedative
c- Give vasodilator
d- Give oxygen

123- Which one of the baseline vital signs that has the most effect on spO2:
a- Heart rate c- Blood pressure
b- Respiratory rate d- Temperature

32
124- AA patient is admitted to the emergency department with a stab wound to the
right chest. Air can be heard entering his chest with each inspiration. To decrease
the possibility of a tension pneumothorax in the patient, the nurse should:
a- Position the patient so that the right chest is dependent.
b- Administer high-flow oxygen using a non-rebreathing mask.
c- Cover the sucking chest wound with an occlusive dressing.
d- Tape a nonporous dressing on three sides over the chest wound.

125- AA client with pneumonia has a temperature of 102.6 ° F (39.2 ° C), is


diaphoretic, and has a productive cough. The nurse should include which of
the following measures in the plan of care:
a- Position changes every 4 hours.
b- Nasotracheal suctioning to clear secretions.
c- Frequent linen changes
d- Frequent offering of a bedpan

126- The nurse reviews an arterial blood gas report for a client with chronic
obstructive pulmonary disease (COPD). PH 7.35; PC02 62; PO2 70; HCO3 34
the nurse should:
a- Apply a 100% non-rebreather mask.
b- Assess the vital signs.
c- Reposition the client.
d- Prepare for intubation

127- If the nurse notes the following symptoms after the client begins taking
sertraline (Zoloft), which one is most likely drug-related:
a- Polyuria c- Drooling
b- Diplopia d- Insomnia
33
128- Which of the following diets would be most appropriate for a client with
chronic obstructive pulmonary disease (COPD):
a- Low-fat, low-cholesterol diet.
b- Bland, soft diet.
c- Low-sodium diet.
d- High-calorie, high-protein diet.

129- The physician has prescribed sertraline (Zoloft) 50 mg daily for a client with
depression. Which finding should be reported to the physician?
a- The client takes Tagamet (cimetidine) for acid reflux
b- The client takes the medication with meals
c- The client takes the medication once a day at bedtime
d- The client takes Aleve (naproxen) for arthritis

130- AA patient was diagnosed with depression 6 weeks ago, and was prescribed
anti-depressant drug.1 week ago the patient started to complain of irritability
and had difficult sleep for only 3 hours daily. Which of the following is the
best nursing action:
a- Inform the doctor to increase the dose
b- Ignore the patient complaint
c- That’s a side effect of the medication
d- Give the patient hypnotics

131- 16 years old girl tried to suicide by taking a large amount of valium
(benzodiazepines), the nurse knows that the antidote for this drug is:
a- Flumazenil c- Meperidine
b- Benztropine d- Naloxone

34
132- The nurse would expect to find which information when reviewing the history
of a client diagnosed with multiple sclerosis:
a- Visual problems
b- Increased sensitivity to pain
c- Ascending weakness and numbness
d- Confusion and disorientation

133- The antidote for morphine overdose is:


a- Naloxone (Narcan)
b- Flumazenil (Romazicon)
c- Benztropine (Cogentin)
d- Meperidine (Demerol)

134- The nurse assesses a respiratory rate of 10 on a client with cancer who has just
received a hydromorphone hydrochloride (Dilaudid) injection. Which drug
should the nurse be prepared to administer:
a- Naloxone (Narcan)
b- Flumazenil (Romazicon)
c- Benztropine (Cogentin)
d- Meperidine (Demerol)

135- Which of the following drugs is the antidote for magnesium toxicity:
a- Calcium gluconate (Kalcinate)
b- Hydralazine (Apresoline)
c- Naloxone (Narcan)
d- Rho (D) immune globulin (RhoGAM)

35
136-A A client with myasthenia gravis is suspected of having cholinergic crisis.
Which of the following indicate that this crisis exists:
a- Ataxia c- Hypotension
b- Mouth sores d- Hypertension

137- AA client with myasthenia gravis is receiving pyridostigmine (Mestinon). The


nurse monitors for signs and symptoms of cholinergic crisis caused by
overdose of the medication. The nurse checks the medication supply to ensure
that which medication is available for administration if a cholinergic crisis
occurs:
a- Vitamin K
b- Atropine sulfate
c- Protamine sulfate
d- Acetylcysteine (Mucomyst)

138- Which of the following is the purpose of providing atropine prior to anesthesia
and surgery:
a- To decrease salivation (decrease secretions)
b- To increase blood pressure
c- To facilitate breathing
d- To decrease muscle tone

139- AA client is diagnosed as having secondary Cushing’s syndrome. The nurse


knows that the client has most likely been taking which medication:
a- Estrogen
b- Penicillin
c- Lovastatin
d- Prednisone
36
140- AA patient has the following preoperative medication order: morphine 10 mg
with atropine 0.4 mg IM. The nurse informs the patient that this injection will:
a- Decrease nausea and vomiting during and after surgery
b- Decrease oral and respiratory secretions, thereby drying the mouth
c- Decrease anxiety and produce amnesia of the preoperative period
d- Induce sleep, so the patient will not be aware during transport to the operating
room

141- AA 40-year-old woman is admitted in labor with high blood pressure, edema,
and proteinuria. She is started on magnesium sulfate. The nurse caring for her
should be sure to keep which drug at the bedside:
a- Calcium gluconate
b- Naloxone (Narcan)
c- Phenytoin (Dilantin)
d- Glucose

142- Which of the following diets would be most appropriate for a patient has
Cushing syndrome:
a- High protein diet
b- High potassium diet
c- High calcium diet
d- High iron diet

143- Vitamin K is prescribed for a neonate. A nurse prepares to administer the


medication in which muscle site:
a- Deltoid c- Vastus lateralis
b- Triceps d- Biceps

37
144- Which of the following interventions should be taken to help an older client to
prevent osteoporosis:
a- Decrease dietary calcium intake
b- Increase sedentary lifestyles
c- Increase dietary protein intake
d- Encourage regular exercise

145- An adult is admitted with heart failure. The nurse notes that he has neck vein
distention and slight peripheral edema. The nurse knows that these signs
indicate which of the following:
a- Pneumothorax
b- Right-sided heart failure
c- Cardiogenic shock
d- Left-sided heart failure

146- An adult client is admitted with a diagnosis of left-sided congestive heart


failure. Which assessment finding would most likely be present?
a- Distended neck veins
b- Dyspnea
c- Hepatomegaly
d- Pitting edema

147- When the nurse on duty accidentally bumps the bassinet, the neonate throws
out its arms, hands opened, and begins to cry. The nurse interprets this
reaction as indicative of which of the following reflexes:
a- Moro Reflex c- Grasping Reflex
b- Babinski reflex d- Tonic Neck Reflex

38
148- AA patient who has required prolonged mechanical ventilation has the
following arterial blood gas results: pH 7.48, PaO2 85 mm Hg, PaCO2 32 mm
Hg, and HCO3 25 mEq/L. The nurse interprets these results as:
a- Metabolic acidosis
b- Metabolic alkalosis
c- Respiratory acidosis
d- Respiratory alkalosis

149- AA patient has the following ABG results: pH 7.32, PaO2 88 mm Hg, PaCO2
37 mm Hg, and HCO3 16 mEq/L. The nurse interprets these results as:
a- Respiratory acidosis
b- Respiratory alkalosis
c- Metabolic acidosis
d- Metabolic alkalosis

150- The nurse forgot to give the patient his prescribed drug, when the head nurse
asked who is responsible for this the nurse replayed that was my wrong this
considered:
a- Responsibility c- Abandonment
b- Ignorance d- Honor

151- When the patient vomits postoperatively, the most important nursing
objective is to prevent:
a- Dehydration
b- Aspiration
c- Rupture of suture line
d- Met. Alkalosis

39
152- The nurse is discussing dietary sources of iron with a client who has iron
deficiency anemia. Which menu, if selected by the client, indicates the best
understanding of the diet?
a- Milkshake, hot dog, and beets
b- Beef steak, spinach, and grape juice
c- Chicken salad, green peas, and coffee
d- Macaroni and cheese, coleslaw, and lemonade

153- AA client is recovering from abdominal surgery and has a large abdominal
wound. A nurse encourages the client to eat which food item that is naturally
high in vitamin C to promote wound healing:
a- Milk
b- Oranges
c- Bananas
d- Chickens

154- Which of the following reflexes is considered a normal reflex of the newborn:
a- Moro reflex
b- Rooting reflex
c- Planter reflex
d- Tonic neck reflex

155- Patient with aortic aneurysm, which of the following is the most dangerous
complications:
a- Embolism
b- Rupture
c- Stenosis
d- Hypotension

40
156- 56 years old patient suffers from heart failure, when the nurse examines his
lungs she noticed the presence of crackles which means that the patient has:
a- Right-sided- failure
b- Left-sided- failure
c- Biventricular failure
d- Congestive heart failure

157- The nurse teaches a client with chronic obstructive pulmonary disease
(COPD) to assess for signs and symptoms of right-sided heart failure. Which
of the following signs and symptoms should be included in the teaching plan:
a- Clubbing of nail beds
b- Hypertension
c- Peripheral edema
d- Increased appetite

158-A A surgery procedure that must be done within 24 hours is called:


a- Selective surgery
b- Elective surgery
c- Urgent surgery
d- Emergency surgery

159- Early sign of ARDS in a patient at risk:


a- Elevated CO2 level
b- Hypoxia not responsive to O2 (cyanosis)
c- Metabolic acidosis
d- Severe, unexplained light imbalance

41
160-AA nurse is assessing a female client with multiple trauma who is at risk for
developing acute respiratory distress syndrome. The nurse assesses for which
earliest sign of acute respiratory distress syndrome:
a- Bilateral wheezing
b- Inspiratory crackles
c- Intercostal retractions
d- Increased respiratory rate

161- 36 years old male patient complaining of fever and headache for 3 days now,
when examining this patent which of the following signs would indicate that
this patient may has meningitis:
a- Positive Kernig’s sign
b- Negative Brudzinski’s sign
c- Positive homan’s sign
d- Negative Kernig’s sign

162- AA patient presented with high fever, headache, vomiting and neck stiffness for
the past 3 days, which of the following is the first diagnostic intervention for
this patient:
a- Urine and stool analysis
b- Lumber puncture with CSF aspiration
c- Complete blood count
d- Chest and abdomen x-ray

163- MRSA (methicillin resistant Staphylococcus aureus) is transmitted by:


a- Contact c- Droplet
b- Airborne d- Standard

42
164- What is the best indicator of effective treatment of CHF in patient taking
Lasix?
a- Calculating total intake daily
b- Weighting himself daily
c- Calculating urine output daily
d- Assessing fluid status daily

165- What supplemental medication is most frequently ordered in conjunction with


furosemide (Lasix)?
a- Chloride c- Potassium
b- Digoxin d- Sodium

166- What is the first intervention for a client experiencing myocardial infarction?
a- Administer morphine
b- Administer oxygen
c- Administer sublingual nitroglycerine
d- Obtain an electrocardiogram

167- Which type of medications is commonly used to treat rheumatoid arthritis?


a- Glucocorticoids
b- Non- steroidal ant- inflammatory drugs (NSAIDs)
c- Antimalarial drugs
d- Gold salt

43
168- AA male client undergoes total gastrectomy. Several hours after surgery, the
nurse notes that the client’s nasogastric (NG) tube has stopped draining. How
should the nurse respond?
a- Notify the physician
b- Reposition the tube
c- Irrigate the tube
d- Increase the suction level

169- Postoperatively, patient is expected to void after:


a- 6-8 hours
b- 2-4 hours
c- 12-24 hours
d- 10-12 hours

170- AA female client has severe menstrual pain, which of the following drugs
you would recommend for this patient to relieve her dysmenorrhea:
a- Ibuprofen c- Aspirin
b- Zantac d- Cortisone

171- The nurse must suction a child with a tracheostomy. Interventions should
include:
a- Encouraging the child to cough to raise the secretions before suctioning
b- Selecting a catheter with a diameter three fourths as large as the diameter of the
tracheostomy tube
c- Ensuring that each pass of the suction catheter take no longer than 5 seconds
d- Allowing the child to rest after every five times the suction catheter is passed

44
172- AA 40 years old female complains of abdominal pain. It is worse after eating,
especially if she has a meal that is spicy or high in fat. She has tried over-the-
counter antacids, but they have not helped the pain. After examining her
abdomen, you strongly suspect cholecystitis. Which sign on examination
increases your suspicion for this diagnosis:
a- Psoas sign
b- Rovsing’s sign
c- Murphy’s sign
d- Grey turner’s sign

173- Which pulse should the nurse palpate during rapid assessment of an
unconscious male adult?
a- Radial c- Femoral
b- Brachial d- Carotid

174- The most important nursing measure in the prevention of thrombophlebitis


for the post-partum mother is:
a- Elastic stocking
b- Early ambulation
c- Anticoagulants
d- Isometric exercises

175- Which of the following is the best aid to prevent breast cancer:
a- Teaching women about breast cancer
b- Public knowledge about chemotherapy
c- To eat fruits and vegetables only
d- Encourage women to perform self-breast examination monthly

45
176- Which of the following factors would contribute to a high risk pregnancy:
a- Blood type O positive
b- First pregnancy at 33 years old
c- History of allergy to honey bee pollen
d- History of insulin dependent DM

177- When the nurse puts an infant with high temperature in cold water in order to
lower his temperature, she knows that the mechanism of heat loss is called:
a- Evaporation
b- Conduction
c- Radiation
d- Convection

178- AA child is admitted to the hospital with a diagnosis of Wilm's tumor, Stage
II. Which of the following statements most accurately describes this stage:
a- The tumor is less than 3 cm. in size and requires no chemotherapy
b- The tumor did not extend beyond the kidney and was completely resected
c- The tumor extended beyond the kidney but was completely resected
d- The tumor has spread into the abdominal cavity and cannot be resected

179- When assessing a 12 year old child with Wilm's tumor, the nurse should keep
in mind that it most important to avoid which of the following:
a- Measuring the child's chest circumference
b- Palpating the child's abdomen
c- Placing the child in an upright position
d- Measuring the child's occipitofrontal circumference

46
180- The nurse is teaching a 45 year old woman how to increase the potassium in
her diet. The woman says she knows bananas are high in potassium, but she
doesn't like their taste. What foods should the nurse recommend the client
include in her diet:
a- Carrots, broccoli, yogurt
b- Rhubarb, tofu, celery
c- Potatoes, spinach, raisins
d- Onions, corn, oatmeal

181- When developing a plan of care for a patient with SIADH (syndrome of
inappropriate antidiuretic hormone secretion), which interventions will the
nurse include:
a- Encourage fluids to 2000 ml/day
b- Long-term fluid restriction
c- Monitor for increased peripheral edema
d- Keep head of bed elevated to 30 degrees

182- AA client is admitted for treatment of the syndrome of inappropriate


antidiuretic hormone (SIADH). Which nursing intervention is appropriate?
a- Infusing I.V. fluids rapidly as ordered
b- Administering glucose-containing I.V. fluids as ordered
c- Restricting fluids
d- Encouraging increased oral intake

183- How streptococci infection is transmitted from person to another:


a- Airborne c- Slandered
b- Contact d- Blood

47
184- AA hospitalized client is found to be comatose and hypoglycemic with a
blood sugar of 50 mg/dL. Which of the following would the nurse do first?
a- Infuse 1000 mL D5W over a 12-hour period
b- Encourage the client to drink orange juice with added sugar
c- Check the client's urine for the presence of sugar and acetone
d- Administer 50% glucose intravenously

185- AA patient receives TPN, which of the following nursing actions is


importantto minimize the risk for fluid volume excess:
a- Increase diuretic dose if swelling occurs
b- Limit the amount of free water in relation to sodium intake
c- Monitor his or her skin turgor
d- Weigh the patient daily on the same scale

186- Which of the following findings would most likely indicate the presence of a
respiratory infection in a client with asthma:
a- Cough productive of yellow sputum
b- Bilateral expiratory wheezing
c- Chest tightness
d- Respiratory rate of 30 breaths/ minute

187- When teaching a mother how to administer eye drops, where should the nurse
tell her to place them?
a- In the conjunctival sac that is formed when the lower lid is pulled down
b- Carefully under the eye lid while it is gently pulled upward
c- On the sclera while the child looks to the side
d- Anywhere as long as drops contact the eye's surface

48
188- One nursing intervention for patient with asthma is to facilitate removal of
secretions. This can be done by:
a- Encourage the patient to perform slow and shallow breathing
b- Encourage the patient to increase fluid intake
c- Encourage the patient to hyperventilate
d- Encourage the patient to decrease fluid intake

189- The nurse is caring for a 6-year-old boy following revision of


ventriculoperitoneal shunt. An expected nursing intervention is:
a- Request for an x-ray to evaluate shunt placement
b- Daily measurement of head circumference
c- Frequent palpation of the fontanels
d- Maintaining the child in a prone position

190- the patient have an itchy, erythematous, vesicular, weeping, and crusting
patches on his skin which skin disease is this considered:
a- Psoriasis
b- Impetigo
c- Eczema
d- Urticaria

191- Giving an intramuscular injection to an infant in the Gluteus Maximus muscle


may produce an injury to which of the following nerves:
a- Femoral nerve
b- Sciatic nerve
c- Vagus nerve
d- Popliteal nerve

49
192- Which of the following is one of the side effects of albuterol nebulizer:
a- Hypertension
b- Hypotension
c- Fast irregular heart beats
d- Tachypnea

193- AA patient, age 49, returns from the postanesthesia care unit after a total
abdominal hysterectomy and bilateral salpingo-oophorectomy to treat cervical
cancer. Which nursing intervention has the highest priority at this time?
a- Monitor the patient for indications of hemorrhage
b- Assess the patient’s pain level and response to analgesics
c- Encourage the patient to do deep breathing and leg exercises
d- Provide emotional support to the patient

194- When planning the care of a client who has undergone an abdominal
hysterectomy, which nursing measure is most helpful for preventing
postoperative complications and facilitating an early discharge?
a- Reestablishing oral fluids and nutrition
b- Promoting ambulation and movement
c- Maintaining accurate intake and output
d- Exploring feelings about altered image

195- Which of the following is considered one of the common complications


following abdominal total hysterectomy:
a- DVT
b- Paralytic ileus
c- Constipation
d- Perforation

50
196- AA client had a hysterectomy 10 hours ago. The nurse assesses the client and
finds that her blood pressure has fallen abruptly. Which action by the nurse is
most appropriate at this time?
a- Continue to monitor the blood pressure every 15 minutes
b- Document the information on the client’s chart
c- Inform the surgeon about the client’s condition
d- Change the client to a Fowler’s position

197- The nurse is teaching a client regarding risk factors for stroke (CVA). The
greatest risk factor is which of the following:
a- Diabetes
b- Heart disease
c- Renal insufficiency
d- Hypertension

198- The nurse recognizes that the most common type of brain attack (CVA) is
related to which of the following:
a- Ischemia c- Headache
b- Hemorrhage d- Vomiting

199- AA woman comes to the clinic and states, "I've been sick for so long! My eyes
have gotten so puffy, and my eyebrows and hair have become coarse and dry."
The nurse will assess for other signs and symptoms of:
a- Cachexia
b- Parkinson's syndrome
c- Myxedema
d- Scleroderma

51
200- AA 32 year-old man is brought to the emergency department by a friend with
whom he had been playing football with earlier that evening. His symptoms
came on after twenty minutes of the football match. He appears anxious and
restless. Auscultation of the lung reveals wheezing on exhalation. The
patient states that he is unable to get a full breath of air. He had this
problem as a child a couple of times after exercising in cold weather.
Blood pressure 126/72mmhg
Heart rate 90/min
Respiratory rate 28/min shallow
Temperature 37.0C tympanic
Oxygen saturation 94% on room air
Which position is the best for this patient?
a- Supine
b- Reverse Trendelenburg’s
c- Prone
d- High fowler’s

201- AA 43 years- old man is 30- hour post- operative following placement of a
partial thickness skin autograft for a burn injury on the lower anterior leg.
During a routine assessment, the nurse observes the wound is bleeding
continuously.
What is the priority nursing action?
a- Use a pen to outline and monitor the area
b- Perform a wound swab for laboratory analysis
c- Incise and drain fluids from the wound bed
d- Apply firm and direct pressure for 10 minutes

52
202- Coarse hair may indicate:
a- Dehydration c- Under nutrition
b- Inflammation d- Anemia

203- The nurse performs a home visit for a 32 years- old woman who had given
birth to her first infant three days before. The mother has concerns about
breastfeeding and the nurse observes the infant feeding. The mother sits
supported upright with cushions and the infant positioned in a cradle hold.
The infant’s head and body are aligned against the mother’s abdomen. The
infant sucks intermittently with the lips turned outwards and the nipple in the
mouth.
Which intervention is most appropriate?
a- Reposition the infant
b- Assist the infant to turn lips inwards
c- Ensure the latch includes the areola
d- Stimulate the infant to suck constantly

204- AA 52 year- old woman is scheduled to undergo an abdomino- perineal


resection in three days for removal of a cancer of the rectum. The nurse
reviews the care plan with the patient. The patient will receive prophylactic
antibiotics and will be given a mechanical bowel preparation the day before.
Which additional preparation should the patient undertake at this time?
a- Wear pressure stockings
b- Perform leg strengthening exercises
c- Maintain high- protein, low- residue diet
d- Take daily ferrous iron tablets

53
205-A A patient is preparing for a scheduled hip
replacement. Which lab value should be reported to
the physician?
Test Result Normal values
Sodium 145 134-164 mmol/L
Potassium 2.9 3.5-5.1 mmol/L
Calcium 2.80 2.15-2.62mmol/L
Magnesium 4.8 1.2-2 mmol/L
a- Sodium
b- Calcium
c- Potassium
d- Magnesium

206- AA nurse is admitting a six month- old infant with pneumonia. Which of
the following interventions supports this infant’s emotional needs?
a- Allow the parents to leave the room during painful procedures
b- Encourage parents to distract the infant from crying
c- Interview the patents to learn the infant’s comforting habits
d- Enforce strict visiting schedule and routines

207- While caring for a patient in the post- anesthesia care unit (PACU) who has
developed hypovolemic shock, a nurse should position the patient:
a- Flat with legs elevated
b- In trendelenburg position
c- With the head of the bed elevated 45 degrees
d- Completely flat

54
208- AA five years – old child girl had presented with tenderness, headache and
petechiae. She was pale and complains of joint pain. On palpation there was
an enlarged spleen, liver and lymph nodes. A lumbar puncture showed central
nervous system involvement. The child underwent chemotherapy treatment
and is now attended by the nurse for regular routine examinations.
Which condition would require more attention from the nurse?
a- Increased leukocytes
b- Lack of muscle coordination
c- Bleeding while brushing teeth
d- Occasional nausea and vomiting

209- During the evaluation at a community clinic, the patient completes the medical
history. Which of the following is NOT a risk factor for an acute myocardial
infarction?
a- Coronary artery disease
b- Smoking
c- Hemophilia
d- Hyperlipidemia

210- AA postoperative patient has the nursing diagnosis of ineffective tissue


perfusion. To assess for tissue perfusion the nurse should check all of the
following except:
a- Skin and nail bed color
b- Temperature of extremities
c- Respiratory rate
d- Peripheral pulses

55
211- AA patient visits the clinic for a 2- week checkup after a corneal
transplantation (keratoplasty). The nurse observes the patent’s sclera is red
and the patient complains of the eye feeling irritated. The nurse suspects the
patient may have:
a- Infection
b- Hemorrhage
c- Graft rejection
d- Postoperative glaucoma

212- The nurse is assessing a patient who just had surgery under general
anesthesia. The patient’s respiration rate is 4 per minute and the O2
saturation on 3L per minute of oxygen via nasal cannula is 84%.
The nurse is awaiting the results of an arterial blood gas (ABG) and
anticipates that which of the following will be ELEVATED?
a- Arterial oxygen saturation (SaO2)
b- Hydrogen ion concentration (pH)
c- Partial pressure of arterial oxygen (PaO2)
d- Partial pressure of arterial carbon dioxide (PaCO2)

213- During the immediate postoperative period, a patient reveals an oxygen


saturation of 91%. The nurse should:
a- Position the patient on the left side
b- Administer supplemental oxygen
c- Continue to provide supportive care
d- Lower the temperature of the room

56
214- The nurse performs an assessment of a 23 year- old man who believes that
people are spying on him. During the interview, he keeps his eyes to the floor
and answers questions awkwardly. He has never had an intimate relationship
and avoids contact with his family members. He has never been employed and
tells the nurse that he is not looking for a job. The nurse considers Erickson’s
theory of psychosocial development.
Which stage is this patient most likely experiencing?
a- Autonomy versus shame and doubt
b- Initiative versus guilt
c- Trust versus mistrust
d- Identity versus confusion

215- AA 32 year- old woman with a motor neuron disease has progressive
weakness of cranial nerves V, IX, XII. She has recently been experiencing
sudden outbursts of crying and laughing. Both stroke and myasthenia gravis
have been ruled out.
Which voluntary muscle activity would be most significantly impacted?
a- Walking
b- Swallowing
c- Breathing
d- Smelling

216- Causes of primary hypothyroidism in adults include:


a- Malignant or benign thyroid nodules
b- Surgical removal or failure of the pituitary gland
c- Surgical removal or radiation of thyroid gland
d- Autoimmune-induced atrophy of the gland

57
217- AA 72-year-old patient is diagnosed with hypothyroidism, and
levothyroxine (Synthroid) is prescribed. During initiation of thyroid
replacement for the patient, it is most important for the nurse to assess:
a- Mental status
b- Nutritional level
c- Cardiac function
d- Fluid balance

218- AA client presents with Hypocalcemia, hyperphosphatemia, muscle cramps,


and positive Trousseau's sign. What diagnosis does this support?
a- Diabetes insipidus
b- Conn's syndrome
c- Hypoparathyroidism
d- Acromegaly

219- The nurse acknowledges that the first-line drug for treating of client's blood
pressure might be which drug:
a- Diuretic
b- Alpha blocker
c- ACE inhibitor
d- Alpha/beta blocker

220- The nurse is teaching a client about clopidogrel (Plavix). What is important
information to include?
a- Constipation may occur
b- Hypotension may occur
c- Bleeding may increase when taken with aspirin
d- Normal dose is 25 mg tablet per day

58
221- The nurse knows that the client's cholesterol level should be within which
range:
a- 150 to 200 mg/dL
b- 200 to 225 mg/dL
c- 225 to 250 mg/dL
d- Greater than 250 mg/dL

222- The nurse is performing an assessment in a client with a suspected diagnosis


of cataract. The chief clinical manifestation that the nurse would expect to
note in the early stages of cataract formation is:
a- Eye pain c- Blurred vision
b- Floating spots d- Diplopia

223- AA client who has a history of Crohn's disease is admitted to the hospital
with fever, diarrhea, cramping, abdominal pain, and weight loss. The nurse
should monitor the client for:
a- Hyperalbuminemia
b- Thrombocytopenia
c- Hypokalemia
d- Hypercalcemia

224- Adult patient complains of diarrhea, vomiting, abdomen cramp and pain
within the past 2 weeks. The patient reported that the pain increases when he
eats and relieves when he passes stool. Which of the following may be the
cause:
a- Appendicitis c- Ectopic pregnant
b- Crohn’s disease d- Cholecystitis

59
225- When caring for a 3-day-old neonate who is receiving phototherapy to treat
jaundice, the nurse in charge would expect to do which of the following:
a- Turn the neonate every 6 hours
b- Encourage the mother to discontinue breast-feeding
c- Notify the physician if the skin becomes bronze in color
d- Check the vital signs every 2 to 4 hours

226- After 3 days of breast-feeding, a postpartum patient reports nipple soreness.


To relieve her discomfort, the nurse should suggest that she:
a- Apply warm compresses to her nipples just before feedings
b- Lubricate her nipples with expressed milk before feeding
c- Dry her nipples with a soft towel after feedings
d- Apply soap directly to her nipples, and then rinse

227- Which of the following interventions would be helpful to a breastfeeding


mother who is experiencing engorged breasts:
a- Applying ice
b- Applying a breast binder
c- Teaching how to express her breasts in a warm shower
d- Administering bromocriptine (Parlodel)

228- What type of milk is present in the breasts 7 to 10 days postpartum?


a- Colostrum
b- Hind milk
c- Mature milk
d- Transitional milk

60
229- Which of the following is a sign of rejection after a renal transplantation
operation:
a- Decrease potassium
b- Decrease sodium
c- Decrease creatinine
d- Decrease urine out put

230- Which of the following is among the signs of magnesium toxicity:


a- Cardiac arrhythmia
b- Loss of deep tendon reflex
c- Patient confusion and irritability
d- Respiratory failure

231- AA nurse is caring for a client who has a potassium level of 5.4 mEq/L. The
nurse should assess the client for:
a- ECG changes
b- Constipation
c- Polyuria
d- Hypotension

232- Moderate to severe hypokalemia in a patient will cause the nurse to observe:
a- Muscle spasms and slow respirations
b- Muscle weakness and cardiac dysrhythmias
c- Confusion and irritability
d- Vomiting and diarrhea

61
233- Which of the following electrolytes decrease would cause PVC (premature
ventricular contraction):
a- Potassium
b- Sodium
c- Calcium
d- Magnesium

234- Which of the following electrolytes decrease would cause recurrent vomiting
and general weakness:
a- Potassium
b- Sodium
c- Calcium
d- Magnesium

235- The nurse is educating a couple who has had difficulty with conception. The
client asks about the hormone, which is responsible for the production of eggs.
The nurse answers that this hormone is called:
a- Melanocyte-stimulating hormone
b- Luteinizing hormone (LH)
c- Follicle-stimulating hormone (FSH)
d- Interstitial cell-stimulating hormone (ICSH)

236- The patient has an order to receive 45 mg of prednisone by mouth daily.


Available are 10-mg tablets. How many tablets should the nurse prepare to
give?
a- 1.6 c- 4.5
b- 2 d- 6

62
237- The nurse caring for a 54-year-old patient hospitalized with diabetes mellitus
would look for which of the following laboratory test results to obtain
information on the patient's past glucose control:
a- Prealbumin level
b- Urine ketone level
c- Fasting glucose level
d- Glycosylated hemoglobin level

238- Which laboratory test is most important for the nurse to monitor to determine
how effectively the client’s diabetes is being managed?
a- Fasting blood glucose
b- Blood chemistry profile
c- Complete blood count
d- Glycosylated hemoglobin (HbA1c)

239- Blood sugar for new employee at company with sedentary life style and obese
and by result of Fasting blood Sugar 6.9 mmol (abnormal reading ) what is the
best next step to do to estimate his blood sugar condition:
a- Draw blood sample for HbA1c
b- Repeat the test for the next 2 days
c- Estimate the postprandial sugar level
d- Write a recommendation note for not hiring him

240- HbA1c as a measurement instrument for glucose level in the blood, it


measures blood glucose level within:
a- Last 3 months c- Last 7 months
b- Last 5 months d- Last 1 year

63
241- Which laboratory test indicates liver cirrhosis?
a- Decreased red blood cell count
b- Decreased serum acid phosphatase level
c- Elevated white blood cell count
d- Elevated serum aminotransferase

242- Which statement best describes the difference between the pain of angina and
the pain of myocardial infarction:
a- Pain associated with angina is relieved by rest
b- Pain associated with myocardial infarction is always more severe
c- Pain associated with angina is confined to the chest area
d- Pain associated with myocardial infarction is referred to the left arm

243- The nurse is developing a bowel-retraining plan for a client with multiple
sclerosis. Which measure is likely to be least helpful to the client?
a- Limiting fluid intake to 1000mL per day
b- Providing a high-roughage diet
c- Elevating the toilet seat for easy access
d- Establishing a regular schedule for toileting

244- Which organism is responsible for the onset of rheumatic fever?


a- Staphylococcus aureus
b- Listeriosis
c- Group A β-hemolytic streptococcus
d- Epstein-Barr virus

64
245- For what reason are patients instructed to do Kegel exercises when they are
discharged after childbirth:
a- They help her get back in shape
b- They prevent her breasts from becoming engorged
c- They help strengthen the perineal muscles
d- They help reduce strain on the back

246- You are preparing to discharge a patient who suffered third degree burn,
which of the following instructions is appropriate to give the patient to
decrease infection:
a- You must take large amount of fluids
b- You should wash your hands thoroughly
c- You better increase vitamin intake
d- You should avoid fatty meals

247- The most effective procedure to prevent spared of infection is:


a- Wearing gloves
b- Using antiseptic
c- Sterilization
d- Hand washing

248- What is the normal pulse range for an adult?


a- 120 to 160 beats per minute
b- 90 to 140 beats per minute
c- 60 to 100 beats per minute
d- 50 to 80 beats per minute

65
249- AA four year- old girl was playing outside in a park when she came runningto
her mother crying and holding her right, upper arm. Two hours later they
presented to the emergency department. There was a swelling over the upper
right arm, with pain and itching at the site as well as swelling of the oral
mucosa. The child seems anxious.
What is the next most appropriate step in management?
a- Administer subcutaneous epinephrine
b- Maintain a patent airway
c- Administer oral diphenhydramine
d- Prepare intubation equipment

250- AA 65 year- old man presents with a resting tremor in the right forearm. An
assessment of gait reveals decreased arm swinging and slight dragging of the
foot on the right side. His body movements are slow. He has not been sleeping
well at night for the past six months due to leg pain and says that he feels
constantly tired and weak. He reports that he has not suffered any recent fall
and that the symptoms seem to be slowly worsening.
Which medication is most likely to be administered?
a- Levodopa c- Phenytoin
b- Haloperidol d- Benzodiazepine

251- When planning discharge teaching for the parent of an infant with
bronchiolitis, the nurse should EMPHASIZE:
a- Use of supplemental oxygen at night
b- Frequent hand washing
c- Sleeping in the supine position
d- Rice- thickened formula during night- time feedings

66
252- AA home care nurse visits a diabetic patient who was started on insulin
injections. Upon examination, the nurse observes small lumps and dents on
the right upper arm where the patient has injected insulin.
What is the BEST nursing intervention?
a- Refer the patient to a dermatologist for diabetic cellulitis
b- Instruct the patient to rotate the sites of injection
c- Refer the patient to an endocrinologist for better control of glucose level
d- Instruct the patient to inject in the muscular area instead of the subcutaneous
area

253- AA patient has an order for 1000 milliliters (ml) of intravenous (IV) fluid to
infuse over eight hours. The available IV tubing has a drip factor of 10 gtts/ml.
Which of the following rates is correct?
a- 125 ml/hour
b- 125 drop/minute volume (ml) X drop factor (gtts / ml)
= gtts / min
---------------------------------------------
c- 21 drops/minute time (min)
(flow rate)

d- 21 ml/hour

254- When is the correct time for the nurse to administer the child’s morning dose
of a combination regular and NPH insulin?
a- 30 minutes before breakfast is served
b- 15 minutes before breakfast is served
c- 30 minutes after breakfast is served
d- 15 minutes after breakfast is served

67
255- The client has an order for administration of 10 units of regular insulin to be
given at 7:00 a.m. The nurse should offer a snack at:
a- 3:00 p.m. c- 11:00 a.m.
b- 1:00 p.m. d- 9:00 a.m.

256- The client is being treated with NPH insulin at 8:00 a.m. The nurse should
offer a snack at:
a- 9:00 a.m. c- 6:00 p.m.
b- 11:00 a.m. d- 2:00 p.m.

257- AA client with diabetes comes to the emergency department. The nurse
obtains a blood glucose measurement with a glucometer and notes that it is
510 mg/dL. The physician orders I.V. insulin. Which type of insulin can be
given both intravenously and subcutaneously?
a- Regular c- Lente
b- NPH d- 70/30

258- If a client with type 1 (insulin-dependent) diabetes mellitus receives 5 units of


NPH insulin every morning at 7 a.m., the nurse should closely monitor the
client for signs of hypoglycemia at what time:
a- 3:00 p.m. c- 7:30 a.m.
b- 12:00 a.m. d- 10:00 p.m.

259- NPH is an example of which type of insulin:


a- Short-acting
b- Intermediate-acting
c- Rapid-acting
d- Long-acting

68
260- AA 70 years- old woman presents with increasing dyspnea on exertion. She
feels breathless and restless while performing household tasks, such as making
the bed and sweeping the floor. Her previous medical history includes a
myocardial infarction at 57 years old. She sleeps with her head elevated on
three pillows. Examination reveals bilateral basal crackles and cold, damp
skin.
Blood pressure 172/94 mmhg
Heart rate 94/min
Respiratory rate 36/min
Temperature 37.1 C
Oxygen saturation 90% on room air
Which heart chamber most likely failed first?
a- Right atrial
b- Right ventricle
c- Left ventricle
d- Left atrial

261- The nurse is preparing a discharge plan for a 65 year- old man with a new
diagnosis of congestive heart failure. The discharge orders include furosemide
40 milligrams by mouth twice per day. The nurse recommends food to reduce
unwanted medication side effects.
Which of the following food would be most appropriate to include in the
teaching plan?
a- Green and leafy vegetables
b- Bananas and oranges
c- Chicken and fish
d- Whole wheat grains

69
262- AA child comes into the clinic with several lesions to the scalp. The
round lesions have dandruff like scaling with hair loss.
What is the MOST likely diagnosis for this skin condition?
a- Impetigo
b- Ringworm (tinea capitis)
c- Pediculosis capitis
d- Scabies

263- AA 40 years- old woman suffered an automobile accident one month ago.
The accident resulted in a C-4 spinal cord injury. Her preventive care
includes independent daily performance of coughing and deep breathing.
Which range- of- motion exercise would be most beneficial for this patient?
a- Active
b- Passive
c- Combined
d- Resistive

264- AA nine year- old girl has a nursing diagnosis of altered body image relatedto
changes in appearance secondary to varicella infection. The child’s body is
covered with a rash and many large, weeping pustules. The nurse provides
counseling to the mother who is concerned that the child will develop scarring.
Which intervention is most appropriate?
a- Keep the skin out of direct sunlight
b- Apply calamine lotion to skin every two hours
c- Wash pustules with soap and keep dry
d- Soak in colloidal oatmeal bath three times daily

70
265- AA nurse is assessing a client with possible Cushing's syndrome. In a client
with Cushing's syndrome, the nurse expects to find:
a- Weight gain in arms and legs
b- Thick, coarse skin
c- Hypotension
d- Deposits of adipose tissue in the trunk and dorsocervical area

266- Which vitamin deficiency is most likely to be a long-term consequence of a


full-thickness burn injury?
a- Vitamin A c- Vitamin C
b- Vitamin B d- Vitamin D

267- Sildenafil (Viagra) is prescribed to treat a client with erectile dysfunction. A


nurse reviews the client's medical record and would question the prescription
if which of the following is noted in the client's history:
a- Neuralgia c- Use of nitroglycerin
b- Insomnia d- Use of multivitamins

268- The physician ordered 1 liter of normal saline infusion for four hours, how
much N/S should be delivered to the patient in one hour:
a- 300 ml c- 200 ml
b- 250 ml d- 150 ml

269- Best diagnostic test for suspected leukemia is:


a- Bone marrow aspiration
b- Blood chemistry
c- CBC
d- PT AND PTT

71
270- The primary nursing goal when caring of a child with leukemia is to:
a- Take vital signs
b- Provide good nutrition
c- Give analgesics
d- Prevent infection

271- The oxygen flow rate for the nasal cannula is considered to be:
a- 1-6 liter/m
b- 3-8 liter/m
c- 5-10 liter/m
d- 10-15 liter/m

272- If the client develops a thrombus in one of the leg veins, which client response
would the nurse expect when eliciting Homans’ sign?
a- Sharp, immediate calf pain
b- Sudden numbness in the foot
c- Inability to bend the knee when asked
d- Tingling throughout the affected leg

273- When the nurse assesses the client for Homans’ sign, which technique is most
accurate?
a- Have the client push each foot against the mattress.
b- Have the client extend the legs and flex each foot toward the head
c- Ask the client to sit up in bed and point all the toes forward
d- Ask the client to contract the thigh muscles

274- The phase that include action of the nursing care plan is called:
a- Implementation c- Assessment
b- Diagnosis d- Evaluation
72
275- After vein ligation and stripping operation, the patient should:
a- Wear elastic stocking
b- Do exercises
c- Rest the leg
d- Avoid walking

276- Rabies disease is considered:


a- Viral disease
b- Bacterial disease
c- Protozoan disease
d- Fungal disease

277- AA client receiving IV infusion. The skin around the IV insertion site is
red, warm to touch and painful. The nurse should first:
a- Discontinue the IV
b- Slow the IV rate for 30 minutes then reassess the site
c- Place a cold compression on the area
d- Place a warm compression on the area

278- Clients who have casts applied to an extremity must be monitored for
complications. The most significant complication for which the nurse should
assess the client's extremity is:
a- Warmth
b- Numbness
c- Skin desquamation
d- Generalized discomfort

73
279- AA nurse is assessing the legs of a client who's 36 weeks pregnant.
Which finding should the nurse expect?
a- Absent pedal pulses
b- Bilateral dependent edema
c- Sluggish capillary refill
d- Unilateral calf enlargement

280- Which of the following is the best nursing action to assess pulse in a toddler
patient:
a- Assess the brachial artery with the middle finger and index
b- Assess the Dorsalis pedis artery with your palm
c- Assess the carotid artery with the four fingers
d- Assess the radial artery using three finger

281- One nursing intervention for patient with productive cough is to facilitate
removal of secretions. This can be done by:
a- Encourage the patient to perform slow and shallow breathing
b- Encourage the patient to increase fluid intake
c- Encourage the patient to hyperventilate
d- Encourage the patient to decrease fluid intake

282- A nurse examines a client’s level of responsiveness. She finds that the patient
opens his eyes to verbal commands, obeys verbal commands, and is oriented to
time, place, and person. What’s the client’s Glasgow Coma Scale:
a- 11/15
b- 12/15
c- 13/15
d- 14/15

74
283- A 73 year old patient looks at you when you speak to her. When you ask her
the date, she says "blue." You note left-sided weakness when she grips your
fingers. What is her Glasgow Coma Score?
a- 11/15
b- 12/15
c- 13/15
d- 14/15

284- A patient during examination opens his eyes in response to pain, makes no
verbal response, but withdraws from pain. What is the Glasgow Coma Score
(GCS) for this patient?
a- 3 c- 7
b- 5 d- 11

285- AA newborn APGAR score at 1 and 5 minutes is 5 and 10, half an hour later
the baby became bluish in color with heart rate of 140/m, your first action
would be:
a- Estimate the score again
b- Shower the baby with warm water
c- Give oxygen immediately
d- Ignore the finding because it is normal

286- The primary critical observation for Apgar scoring is the:


a- Heart rate
b- Respiratory rate
c- Presence of meconium
d- Evaluation of the Moro reflex

75
287- When performing a newborn assessment, the nurse should measure the vital
signs in the following sequence:
a- Pulse, respirations, temperature
b- Temperature, pulse, respirations
c- Respirations, temperature, pulse
d- Respirations, pulse, temperature

288- Within 3 minutes after birth the normal heart rate of the infant may range
between:
a- 100 and 180
b- 130 and 170
c- 120 and 160
d- 100 and 130

289- The expected respiratory rate of a neonate within 3 minutes of birth may be as
high as:
a- 50
b- 60
c- 80
d- 100

290- When performing nursing care for a neonate after a birth, which intervention
has the highest nursing priority:
a- Obtain a dextrostix
b- Give the initial bath
c- Give the vitamin K injection
d- Cover the neonates head with a cap

76
291- Which of the following instructions are appropriate for the nurse to give the
patient about the time of taking omeprazole tablet:
a- Take the tablet during the meal
b- Take the tablet 2 hours after the meal
c- Take the tablet 30-45 minute before the meal
d- Take the tablet before bed time

292- Patient obtained second degree burn to his abdomen and his entire back,
according to rule of nine what is the percentage of burn of total body surface
area:
a- 9%
b- 18%
c- 27%
d- 36%

293- R.T.A. Patient arrived to E.R, on examination the patient has increased heart
rate, low blood pressure and decreased level of consciousness, you must think
that the patient is developing:
a- Coma
b- UTI
c- Shock
d- Respiratory arrest

294- Which of the following disease has no vaccinations:


a- Small pox
b- Measles
c- Poliomyelitis
d- Chicken pox
77
295- The type of burn in which all the dermis and epidermis, is destroyed and there
is involvement of underlying structures is called:
a- Superficial or first degree burn
b- Partial thickness or second degree burn
c- Full-thickness or third degree burn
d- Fourth degree burn

296- Which of the following is the drug of choice for treatment of rheumatic fever:
a- Amoxicillin c- Cephalexin
b- Azithromycin d- Penicillin

297- What disease is associated with a Vitamin C deficiency?


a- Pellegria
b- Neural tube defects
c- Scurvy
d- Pitted edema

298- AA 59 year old woman is diagnosed with Hemolytic anemia. What foods
would be recommended for her to eat:
a- Fish oils c- Yellow fruits
b- Eggs, cheese d- Grains, nuts

299- The physician orders an intradermal injection of 5 tuberculin units/0.1 ml of


tuberculin purified derivative. Which needle is appropriate for this injection:
a- 12 gauge needle
b- 18 gauge needle
c- 22 gauge needle
d- 26 gauge needle

78
300- Which nursing diagnosis takes priority for a newly diagnosed patient with a
left- sided stroke?
a- Risk of impaired swallowing related to absent gag reflex
b- Risk for impaired skin integrity related to immobility
c- Risk for infection related to invasive line placement
d- Risk for impaired speech related to left sided stroke

301- When caring for a patient with left-sided homonymous hemianopsia resulting
from a stroke, which intervention should the nurse include in the plan of care
during the acute period of the stroke?
a- Apply an eye patch to the left eye
b- Approach the patient from the left side
c- Place objects needed for activities of daily living on the patient's right side
d- Reassure the patient that the visual deficit will resolve as the stroke progresses

302- AA client with a serum glucose level of 618mg/dl is admitted to the facility. He’s
awake and oriented, has hot dry skin, and has the following vital signs:
Temperature 38.1º C
Heart rate 116 beats/minute
Blood pressure 108/70 mm Hg
Based on these assessment findings, which nursing diagnosis takes the highest
priority?
a- Deficient fluid volume related to osmotic diuresis
b- Decreased cardiac output related to elevated heart rate
c- Imbalanced nutrition: less than body requirements related to insulin deficiency
d- Ineffective thermoregulation related to dehydration

79
303- AA patient has a blood glucose level of 60mg/dl. The patient is to receive 15
gof carbohydrate if the blood glucose is less than 70mg/dl.
How many mls of orange juice should the patient receive?
a- 120 ml
b- 90 ml
c- 60 ml
d- 30 ml

304-AA patient has a blood glucose level of 60mg/dl. The patient is to receive 15 g of
carbohydrate if the blood glucose is less than 70mg/dl.
How many candies should the patient receive?
a- 4 candies
b- 6 candies
c- 8 candies
d- 10 candies

305-A A patient is scheduled for a pneumonectomy in the morning.


Which of the following diagnosis is the MOST likely indication for this type
of surgery?
a- Lung carcinoma
b- Pulmonary tuberculosis
c- Benign pulmonary nodule
d- Mediastinal shift

80
306- The nurse is caring for a patient with a deep vein thrombosis (DVT). The
patient’s heparin sodium infusion has been discontinued and the patient is
receiving prescribed warfarin sodium (Coumadin)
The nurse should advise the patient that which of the following needs to be
continued?
a- Daily complete blood count (CBC)
b- Laboratory tests for partial thromboplastin time (PTT)
c- Strict Bedrest
d- Wearing elasticized support stockings

307-AA patient receiving a total parenteral nutrition through a central line


suddenly has difficulty breathing and is restless. Chest auscultation reveals a
heart murmur.
Blood pressure 90/60 mmhg
Heart rate 120/min
Respiratory rate 22/min
Temperature 37.1 C
Oxygen saturation 90% on room air
What is the most appropriate initial nursing action?
a- Notify the physician
b- Administer 100% oxygen by face mask
c- Place patient in left side lateral decubitus
d- Obtain stat blood glucose measurement

81
308-A A 16 year- old boy fractured his right tibia and fibula during a football match.
Eight weeks later, the fractures were successfully healed and the cast was
removed.
Which range- of- motion exercises would be most beneficial?
a- Active c- Combined
b- Passive d- Resistive

309- AA 65 year- old woman who was diagnosed with hypothyroidism at the age of
45 is brought to the clinic by ambulance. On arrival, she had a severely
decreased level of consciousness. Her breathing is shallow and irregular. The
skin is cool, dry and pale. There is generalized non- pitting edema of all
extremities and face.
Blood pressure 70/40 mmhg
Heart rate 60/min
Respiratory rate 12/ min
Body temperature 35.5 C tympanic
What is the most appropriate method to re-warm this patient?
a- Warmed intravenous isotonic fluids
b- Place the patient is a warm bath
c- Apply heat packs to the head and neck
d- Cover the patient with hospital blankets

310- The physician ordered a blood glucose test for the neonates the nurse knows
the best site to puncture is usually:
a- The lateral heel c- Finger tip
b- Anterior sole d- Anterior scalp

82
311- The nurse is teaching a patient who was recently diagnosed with rheumatoid
arthritis affecting the hands.
Which of the following treatment should the nurse discuss with the patient
at this time?
a- Transcutaneous electric nerve stimulation (TENS)
b- Iontophoresis
c- Nonsteroidal anti-inflammatory drugs (NSAIDs)
d- Synovectomy

Problems The patient will


A Altered tissue perfusion related to hypertension Maintain blood pressure below 130/90 mmhg
B Fluid volume excess related to lower extremity edema Not have signs of edema or unexpected weight gain
C Knowledge deficit related to disease process, hyper Verbalize understanding of disease process,
tension symptoms and medication management
D Alternation in comfort related to chronic headaches Report an absence of headaches each week
312- AA patient has come to the clinic for follow- up one week after being
discharged from the hospital for treatment of a hypertensive crisis. Blood
pressure stabilized at 124/78 mm Hg. The patient reports feeling well, has no
edema, no longer has daily headaches. Blood pressure is 156/90 mm Hg.
During evaluation the patient admits to having stopped taking medication
thathad been ordered because headaches are no longer present. Unless the
symptoms return, the patient states he will not be returning to the clinic.
What should the nurse do?
a- Resolve and discontinue the entire care plan per patient request, suggest
psychology consult
b- Resolve B and D; continue A and C
c- Continue entire care plan as written
d- Add a new problem to the care plan, non-compliance and interventions to
determine potential reasons.

83
313- AA 34 year- old quadriplegia patient resides at home with his wife. In order to
prevent contractures of all extremities, the community care nurse will instruct
the patient’s wife in performance of:
a- Active range of motion exercises
b- Passive range of motion exercises
c- Active- assistive range of motion exercises
d- Resistive range of motion exercises

314- AA six years- old patient has presented to the clinic with fever, malaise, and
anorexia. The patient was treated 2 weeks ago for a streptococcal infection of
the throat.
The nurse should expect the physician to order what test?
a- Electrocardiogram
b- Jones test
c- Spinal tap
d- Heart biopsy

315- AA 40 year- old woman presented with right hip pain. Palpation of the pelvic
girdle is normal. An X- ray shows bone deformities, with osteolytic lesions and
bone enlargement. The patient has not suffered any trauma and has been
generally healthy.
Which serum laboratory analysis would be most useful?
a- Prothrombin time
b- Alkaline phosphatase
c- Acid phosphatase
d- Parathyroid hormone

84
316- AA client complains of itching about 20 minutes after he began his blood
infusion, which of the following is the best nursing action his caring nurse
must take:
a- Stop the infusion immediately and call the physician
b- Call the physician immediately and the stop the infusion
c- Culminate the client and continue infusion
d- Continue the infusion while monitoring the client closely

317- AA 32 year- old woman with diabetes mellitus type 1 underwent a


cholecystectomy and is now on day two of recovery. The patient’s bowel
sounds have returned and she has resumed a normal diet but has been
finishing less than half of each meal on the tray. The nurse enters the room to
perform a routine assessment and finds the patient confused and shaky
Blood pressure 110/60 mmhg
Heart rate 96/min
Respiratory rate 22/min
Temperature 37.0 C oral
What is the most appropriate initial intervention?
a- Administer glucagon
b- Notify the physician
c- Give an insulin injection
d- Offer a glass of juice

318- AA Cancer patient wants to stop chemotherapy and change to palliative


care, this situation indicates which psychological status:
a- Depression c- Anxiety
b- Hopelessness d- Restlessness

85
319- What do we call the type of treatment of terminal illness patients?
a- Conservative
b- Curative
c- Palliative
d- Selective

320- Psychiatric patient appear violent for himself and others was put in the room
alone during the period of exacerbation, then patient calm down and informed
the nurse I am ok now let me with others, but the nurse refused that as a
punishment way. At which underline Label will nurse accused under the
court:
a- Abandonment
b- False imprisonment
c- Negligence
d- Duty to act

321- Which of the following actions is the first priority of care for a client
exhibiting signs and symptoms of coronary artery disease?
a- Decrease anxiety
b- Enhance myocardial oxygenation
c- Administer sublingual nitroglycerin
d- Educate the client about his symptoms

322- What is the first intervention for a client experiencing MI?


a- Administer morphine
b- Administer oxygen
c- Administer sublingual nitroglycerin
d- Obtain an ECG
86
323- AA patient complains of stable angina presented with chest pain. The
physician ordered Nitroglycerine tablet sublingual. The nurse knows that the
right dose to give the patient is:
a- 0.1 mg c- 0.4 mg
b- 0.2 mg d- 0.8 mg

324- Cancer patient during the shift and he surrounded by visitor and laughing
loudly before the shift end patient call the nurse and asked for pain
medication and said that his level of pain 8/10 (sever pain) patient appears not
on pain, what is the best intervention for this situation:
a- Give the patient extra dose of his pain medication
b- Ask the patient about the last time he had his medication
c- Ignore the patient as you are going to leave
d- Check the patient’s file for history of drug abuse

325- AA patient with pain was prescribed Ibuprofen 300 mg q 4 hours PRN
and morphine 5mg q 4 hours PRN. After 2 hours of receiving Ibuprofen
the patient still complaining of pain, what is the best intervention:
a- Wait 2 hours and give a second dose of ibuprofen
b- Give the patient his dose of morphine
c- Wait 1 hour and give the patient his morphine
d- Ask the patient to bear the pain

326- Which instruction should a nurse give the parents of an infant undergoing
cleft lip repair?
a- Offer the pacifier as needed
b- Lay the infant on his abdomen for sleep
c- Sit the infant up for each feeding
d- Loosen arm restraints every hour

87
327- Patient with abdominal incision, seven days postoperatively the patient has
pain at the site of incision which increased by walking and the incision appear
red and there’s pus surrounding the incision site. This may indicate which of
the following:
a- Normal finding
b- Inflammation of the incision
c- Under nutrition
d- Good indicator

328-A A preschooler is admitted to the hospital the day before scheduled surgery.
This is the child’s first hospitalization. Which action will best help reduce the
child’s anxiety about the upcoming surgery:
a- Begin preoperative teaching immediately
b- Describe preoperative and postoperative procedures in detail
c- Give the child dolls and medical equipment to play out the experience
d- Explain that the child will be put to sleep during surgery and won’t feel anything

329- Which teaching aid provided by the nurse is developmentally appropriate for
a preschooler who is about to have a bone marrow puncture:
a- Dolls or puppets
b- Pamphlets or booklets
c- Colored diagrams
d- Commercial videotapes
330-AA nurse is caring for an infant with spina bifida. Which assessment
findings suggest hydrocephalus?
a- Depressed fontanels and suture lines
b- Deep-set eyes, which appear to look upward only
c- Rapid increase in head size and irritability
d- Motor and sensory dysfunction in the foot and leg
88
331- Parents of a newborn with a unilateral cleft lip are concerned about having
the defect repaired. The nurse explains that a child with a cleft lip usually
undergoes surgical repair:
a- Immediately after birth
b- By 3 months of age
c- After 12 months of age
d- Varies in every case

332- The hepatitis B vaccine series should begin at what age:


a- Newborn c- 6 months
b- 2 months d- 12 months

333-AA school age client admitted to the hospital because of decreased urine
output and per-orbital edema and diagnosed with glomerulonephritis.
Whichof the following interventions would receive the highest priority:
a- Monitoring the vital signs every 4 hours
b- Monitoring intake and output every 12 hours
c- Monitoring the client weight daily
d- Monitoring serum electrolyte daily

334- The nurse is caring for a client with renal failure who is being treated with
peritoneal dialysis. Which assessment before and after peritoneal dialysis is
most valuable in evaluating the outcome of treatment:
a- Pulse rate
b- Body weight
c- Abdominal girth
d- Urine output

89
335- Which finding provides the best evidence that peritoneal dialysis is achieving a
therapeutic effect:
a- Urine output increases
b- Appetite improves
c- Potassium level falls
d- Red blood cell count is lower

336- When the nurse is advising the client about the potential complications
associated with peritoneal dialysis, which complication is most important to
include:
a- Pulmonary edema
b- Abdominal peritonitis
c- Abdominal hernia
d- Ruptured aorta

337-A A client receiving hemodialysis treatment arrives at the hospital. He


complains of shortness of breath, and pedal edema is noted. His last
hemodialysis treatment was yesterday.
Blood pressure 200/100
Heart rate 110/min
Respiratory rate 36/m
Oxygen saturation 89% on room air
Which of the following interventions should be done first?
a- Administer oxygen
b- Elevate the foot of the bed
c- Restrict the client’s fluids
d- Prepare the client for hemodialysis

90
338- Changes in personality and judgment are often associated with a lesion in
which of the following:
a- Frontal lobe
b- Parietal lobe
c- Broca's area
d- Wernicke's area

339- Anterior lobe of the brain is responsible for which of the following:
a- Personality
b- Movement
c- Speech
d- Memory

340-AA history of smoking, abnormal permanent enlargement of the alveoli, cough,


and dyspnea suggest:
a- Asthma
b- Emphysema
c- Chronic bronchitis
d- Obstructive sleep apnea

341- Which of the following is the end of the first stage of labor:
a- Cervix dilated to 10 cm
b- Crowning of the presenting part
c- Increased bloody show
d- Contractions lasting up to 60 seconds

91
342- During the latent phase of the first stage of labor, how often should the nurse
plan to assess the fetal heart rate?
a- Every 5 to 15 minutes
b- Every 15 to 30 minutes
c- Every 30 to 60 minutes
d- Every 60 to 90 minutes

343- The nurse explains that oxytocin (Pitocin) is given after delivery of the baby
and placenta for which purpose:
a- To increase the blood pressure
b- To prevent the uterus from inverting
c- To decrease the likelihood of hemorrhage
d- To prevent rupture of the uterus

344-AA nurse in the delivery room is assisting with the delivery of a newborn
infant. After the delivery of the newborn, the nurse assists in delivering the
placenta. Which observation would indicate that the placenta has separated
from the uterine wall and is ready for delivery:
a- The umbilical cord shortens in length and changes in color
b- A soft and boggy uterus
c- Maternal complaints of severe uterine cramping
d- A sudden gush of dark blood from the introitus

345- Which vitamin deficiency increase the risk of neural tube defects:
a- Folic acid
b- Vitamin C
c- Vitamin B12
d- Calcium

92
346-AA client has been diagnosed with folic acid deficiency. The nurse’s discharge
teaching will focus on foods high in folic acid. Which of the following foods has
the highest folic acid level:
a- Citrus fruits
b- Raisins
c- Brewer’s yeast
d- Eggs

347- Which of the following is the recommended daily dose of folic acid during
pregnancy:
a- 200 mcg
b- 400 mcg
c- 800 mcg
d- 1000 mcg

348- Lochia normally disappears after how many days postpartum:


a- 5 days
b- 7-10 days
c- 18-21 days
d- 28-30 days

349- If the maintenance requirement of fluid for a child is 900 ml/ day, each daily
feeding account 120 ml, what is the number of feeding is required to achieve
this maintenance:
a- 5 feedings
b- 6 feedings
c- 7 feedings
d- 8 feedings

93
350-AA 12 year- old patient had a cast removed from the left leg after wearing if
for eight weeks. The patient wants to resume sports as soon as possible. In
order to regain muscle strength lost while wearing cast, the nurse will
instructthe patient in performance of:
a- Resistive range of motion exercises to left leg
b- Passive range of motion exercises to right leg
c- Active- assistive range of motion exercises to the right leg
d- Active range of motion exercises to both legs

351-AA 31 years- old woman with diabetes type 1 presents to the clinic with fatigue,
blurred vision, and loss of appetite. Her breath smells like fruit and she
leaves the room twice during the examination to use the toilet. She has
brought a little bottle of water with her that she finishes while at the clinic.
She reports that she has had a cold for the past three days, but has not taken
additional insulin during the illness
Blood pressure 130/70 mmhg
Heart rate 90/min
Respiratory rate 20/min
Body temperature 38.0 C oral
What is the most appropriate nursing diagnosis
a- Risk for impaired skin integrity related to circulation
b- Deficient knowledge related to illness management
c- Risk for fluid volume excess related to fluid intake
d- Imbalanced nutrition related to decreased appetite

94
352- Adult patient suffers from hoarseness of voice, and difficulty in speech may be
suffering from an injury of which of the following cranial nerves:
a- IX c- XI
b- X d- XII

353- The parent of a child with chronic asthma is hesitant to discipline because the
child often doesn’t feel well. The nurse should encourage the parent to:
a- Set consistent behavior limits
b- Be more lenient during times of illness
c- Cherish the limited time the child has to live
d- Avoid upsetting the child with limit- setting

354-AA 55 year- old woman presented with pain in the joints of the hands. She feels
generally tired and has had a low grade fever for the past week. On
examination, the joints of the fingers on both hands are symmetrically swollen.
Blood pressure 120/70 mmhg
Heart rate 82/min
Respiratory rate 18/min
Temperature 38.0 C
Oxygen saturation 99% on room air
Which type of arthritis is most likely?
a- Rheumatoid
b- Septic
c- Psoriatic
d- Osteoarthritis

95
355-AA patient presented to the office for a physical examination. The
patientis found to be healthy and fit but occasionally drinks alcohol and
has unprotected sex.
What is the BEST nursing diagnosis?
a- Health- seeking behavior
b- Knowledge deficit, high- risk behaviors
c- Low self- esteem
d- Altered thought process

356-AA seven year- old child presented to the emergency room with a fracture of the
right arm.
What would be the long- term goal for this patient?
a- Patient verbalizes decreased pain level
b- Nurse notes neurovascular checks are within normal limits
c- Nurse notes decreased swelling of the right arm
d- Patient performs activities of daily living without difficulty

357- The nurse is caring for a woman whose husband beats her regularly. Which is
the most important long-term goal for this woman?
a- Provide a long-term support group
b- Help her feel like a survivor
c- Point out the ways she behaved
d- Be able to blame the abuser

358- In what position should a dyspneic patient be placed?


a- Prone
b- Recumbent
c- Semi-fowler’s
d- Trendelenburg
96
359-A A nurse is admitting a two year- old child with an umbilical
hernia. Which of the following interventions does NOT meet the
child’s developmental needs?
a- Allowing the child to make choices when possible
b- Providing rooming in the unlimited visitation
c- Attempting to continue rituals used at home
d- Maintaining strict bedrest

360- AA six year- old boy is in the postoperative care unit following a tonsillectomy.
The nurse observes that his gag reflex has returned and removes the artificial
airway. The patient then begins to cry and tells the nurse that his throat hurts
badly. What type of data is the patient providing the nurse?
a- Objective c- Comparative
b- Inferential d- Subjective

361- AA first time mother of a three week- old breastfed baby brings the infant to
the clinic and complains that her child has been forcefully vomiting after
feeding. He was born 40 week’s gestation. Weighing 3.6 kilograms. He is
constantly hungry and irritable. Examination reveals a swollen abdomen and
apalpable mass in the middle upper right quadrant.
What is the most likely health problem?
a- Intussusception
b- Pyloric stenosis
c- Gastroeosophageal reflux
d- Diaphragmatic hernia

97
362- AA home care nurse reviews the second follow- up laboratory test results of a
patient with iron deficiency anemia. The nurse compares the recent laboratory
results to the results taken 3 months ago. The patient has been eating food rich
in iron as prescribed by the nutritionist.
What can be inferred about the progress of the treatment by comparing the
two laboratory values?
Laboratory values three months prior
Test Result Normal value
Albumin 35 34-56 g/L
Glucose 5.3 3.5-6.5 mmol/L
HCT 0.32 0.41-0.50
Hb 121 120-140 g/L
Blood urea nitrogen 6.1 2.8 to 8.9 mmol/L
Creatinine 60 58-145 μmol/L
Current laboratory values
Test Result Normal value
Albumin 33 34-56 g/L
Glucose 5.7 3.5-6.5 mmol/L
HCT 0.41 0.41-0.50
Hb 139 120-140 g/L
Blood urea nitrogen 5.7 2.8 to 8.9 mmol/L
Creatinine 60 58-145 μmol/L
a- Patient’s uptake of iron by diet is sufficient, and no additional intervention is
necessary
b- Patient’s uptake of iron by diet is sufficient, but additional intervention is
necessary
c- Patient’s uptake of iron by diet is insufficient, and additional intervention is
necessary
d- Patient’s uptake of iron by diet is insufficient, and the physician’s immediate
action is necessary

98
363- Which assessment finding would the nurse expect in an infant diagnosed with
pyloric stenosis?
a- Abdominal rigidity
b- Ribbon-like stools
c- Visible waves of peristalsis
d- Rectal prolapse

364- AA six year-old child with type 1 diabetes has an uncontrolled blood glucose
level. The child has been given regular insulin with minimal change in glucose
level in the first 30 minutes.
Which type of insulin has an onset of 15 minutes and a duration of no more
than 4 hours?
Type of insulin Onset(minutes) Maximal Duration
a- Aspart (Novolog) activity(hours) (hours)

b- Lispro (Humalog) Aspart(Novolog) 15 1-3 3-5


Lipsro(Humalog) 10-30 1-2 2-4
c- Semilete regular 30-60 2-4 6-9
semilente 30-60 2-4 10-12
d- Neutral protamine hagedom (NPH) NPH 120 4-12 24
Lente 120 8-10 24

365-AA patient with cerebrovascular accident, left-sided hemiplegia, and aphasia


has nursing diagnosis of risk for aspiration related to swallowing difficulties
with a short-term goal that the patient will not aspirate. The patient has
undergone insertion of a percutaneous gastric endoscopy and has all nutrition
and fluid administered through the tube. The nurse should:
a- Continue the care plan as written
b- Discontinue the risk for aspiration diagnosis
c- Revise the goals and interventions for the diagnosis
d- Add a nursing diagnosis of ineffective health maintenance

99
366-A A nurse is evaluating a patient 5 days after a right total hip replacement.
Which of the following goals is appropriate for the patient?
a- Maintain abduction without dislocation
b- Rest with legs elevated while sitting
c- Tie shoes and put on undergarments without assistive devices
d- Perform scissor-like exercises daily

367-AA three year- old child was admitted to the postoperative care unit following
a heart transplant. The nurse administers cyclosporine by intravenous
infusion. Fifteen minutes later the child has difficulty breathing, his skin feels
cold and clammy and he appears restless. Which is the most appropriate
initial nursing action?
a- Ensure airway patency
b- Administer oxygen therapy
c- Discontinue intravenous infusion
d- Administer intramuscular epinephrine

368-A A 40 year-old woman is a gravida2, para 2 and is currently trying to conceive.


Her previous pregnancy resulted in the birth of a baby with cleft lip and
palate. The patient is anxious and concerned about future pregnancies and the
nurse provides genetic counseling and reassurance.
Which food would most effectively prevent recurrence?
a- Green vegetables and citrus fruit
b- Eggs, milk and dairy products
c- Wheat, corn, rice, oats and rye
d- Beef, chicken and yellow vegetables

100
369- AA 50 year-old woman presented with poor balance and coordination. She
says that she has developed pain on the outer aspect of the thighs and the
inner side of the arch of the foot. The nurse wishes to examine the integrity of
the affected lumbar spinal nerve root and performs a deep tendon reflex
examination.
Which reflexes would most likely be diminished?
a- Planter
b- Brachioradialis
c- Patellar
d- Achilles tendon

370- When administrating an intramuscular injection to an infant, which of the


following sites is appropriate for the nurse to use?
a- Rectus femoris c- Dorsogluteal
b- Deltoid d- Ventrogluteal

371- AA 66 years-old woman is admitted to the hospital with a history of


hypertension. She presented with breathing difficulties that worsen with
activity and while sleeping. She is generally weak and feels that her heart
misses beats and that it sometimes beats loudly. An electrocardiogram shows
atrial fibrillation, right ventricular hypertrophy and deviation toward the
right.
Which nursing intervention is most appropriate for this patient?
a- Provide a bedside commode (portable toilet)
b- Place in right side-lying position
c- Encourage family and friends to visit
d- Encourage independent hygienic activities

101
372- Oral iron supplements are prescribed for a 6-year-old child with iron
deficiency anemia. The nurse instructs the mother to administer the iron with
which of the following food item to enhance absorption of iron:
a- Milk
b- Water
c- Apple juice
d- Orange juice

373- 9 months old infant was brought to the hospital by his parents because he is
crying most of the time. The examination revealed that he is suffering from
otitis media. Which of the following instructions you would include in the care
plan you suggest to the parents:
a- Give antibiotic as prescribed
b- Try to calm down your baby
c- Wash the ears regularly
d- Ignore the child’s pain

374- AA child with pneumonia was prescribed penicillin injection. Before giving the
injection to the child the nurse performed skin allergy test which showed that
the child has penicillin allergy. Which of the following is the next step the
nurse should take:
a- Stop the treatment and inform the doctor
b- Administer penicillin intramuscular
c- Postpone the does until the child is well
d- Ignore the test result and give the injection

102
375- The following table represents the blood pressure value for a patient in 3
successive days. Regarding the information given in the tablet, this patient is
considered: First day 120/80
a- Normal blood pressure 122/87
Second day
b- Pre-hypertension
Third day 133/88
c- Hypertensive
d- Hypotensive

376- AA patient with pneumonia with excessive mucous in the left lung, which of
the following is the best position to facilitate drainage of the mucous from the
lower left lung:
a- On the right side with trendelenburg position
b- On the left side with elevation of HOB
c- On the right side with elevation of the HOB
d- On the left side with trendelenburg position

377- AA patient complains of left eye redness and itching, the doctor told you to put
atropine eye drops for the patient to examine his eye. The nurse should instill
the eye drops into:
a- The left eye
b- The right eye
c- Both right and left eyes
d- Neither of the eyes

103
378- The district nurse visits a 30 year-old woman at home following the delivery of
her second child, a full term girl. Following the delivery of her first child, she
had developed a breast infection and stopped breastfeeding because of the
pain. She asks the nurse how she can best prevent it with this infant.
What is the most appropriate response?
a- Provide feeding on demand
b- Apply vitamin E cream to the nipples
c- Request a prophylactic antibiotic
d- Apply heat to the breasts after feeding

379- In planning home care for an immunocompromised child, the nurse instructs
the parents to use cream or emollients to prevent or manage dry and cracked
skin. A parent will BEST demonstrate understanding of the rationale for this
be stating:
a- Creams will prevent breaks in the skin and decrease the chance of infection
b- Pleasantly scented creams will mask other less pleasant smells
c- Micronutrients in the creams will help prevent malnutrition
d- Creams will help prevent dehydration when my child does not drink enough

380- The nurse is teaching a patient about spironolactone (Aldactone).


Which of the following instructions should review with the patient?
a- Increasing the intake of foods that are high in potassium
b- Taking the medication right before going to sleep
c- Avoid seasonings that are labeled as salt substitutes
d- Scheduling the medication so that a multivitamin is taken an hour later

104
381- AA 49 year-old man with a diagnosis of alcoholic cirrhosis and ascites is
discharged from the hospital with a priority nursing diagnosis of altered
nutrition, less than body requirements.
Which dietary plan is most appropriate?
a- Low-protein, low sodium
b- Low sodium, high- protein
c- High- protein, low potassium
d- Low potassium, high protein

382- AA patient who is 4 days postoperatively after a total hip replacement


surgery, is obese and has not been able to ambulate since the surgery. The
patient is now diaphoretic, has chills, and complains of pain in the thigh. The
MOST likely cause is:
a- Wound infection
b- Deep vein thrombosis (DVT)
c- Pulmonary edema
d- Dehydration

383- Which statement by the patient with hyperlipidemia shows a basic


understanding of the disease and its treatment?
a- Exercises has no effect on cholesterol levels
b- Hyperlipidemia is usually asymptomatic until significant target organ damage is
done
c- HDL cholesterol level of greater than 60mg/dL increases the chance of coronary
artery disease
d- Cholestyramine (questran) should be taken in the morning with other
medications

105
384- Which of the following describe the correct sequence of nursing process:
a- Assessment, planning, diagnosis, implementation and evaluation
b- Planning, assessment, diagnosis, implementation and evaluation
c- Planning, diagnosis, implementation, assessment and evaluation
d- Assessment, implementation, evaluation, planning and diagnosis

385- AA 14 year-old boy presented to the emergency department. He


complainsof having progressively worsening stomach pain for the past
eight hours. On assessment, the pain is localized in the lower right
quadrant. He rates it as a level ten on the one-ten pain scale. Abdominal
palpation shows rebound tenderness in the lower right quadrant, and
positive McBurney and psoas signs.
Blood pressure 134-78 mmhg
Heart rate 88/min
Respiratory rate 24/min
Temperature 38.2 C
Test result normal values
WBC 12.4 4-10.5 x 109
Which intervention would be most appropriate to alleviate this patient’s
pain?
a- Apply warm packs to affected area
b- Maintain semi-fowler’s with legs up
c- Withhold solids and liquids
d- Administer analgesics

106
386- Female patient with a diagnosis of atrial fibrillation, the physician ordered
you to give her digoxin IV, but the patient refused to be injected and told that
she feel she is going to vomit, what you should do:
a- Change the IV digoxin to oral form
b- Force the patient to accept the medication
c- Inform the doctor about the situation
d- Don’t give any medications to this patient

387- 40 years old client complaining from abdominal pain, laboratory result for
stool analysis showed that there’s occult blood in the stool. Which of the
following procedure would help for diagnosis of this patient:
a- Abdominal ultrasound c- Colonoscopy
b- Abdominal x ray d- MRI

388- AA 7-year-old client is brought to the E.R. He’s tachypneic and afebrile and
has a respiratory rate of 36 breaths/minute and a nonproductive cough. He
recently had a cold. From his history, the client may have which of the
following:
a- Acute asthma
b- Bronchial pneumonia
c- Chronic obstructive pulmonary disease (COPD)
d- Emphysema

389- Which of the following is a priority goal for the client with COPD:
a- Maintaining functional ability
b- Minimizing chest pain
c- Increasing carbon dioxide levels in the blood
d- Treating infectious agents

107
390- AA 34-year-old woman with a history of asthma is admitted to the emergency
department. The nurse notes that the client is dyspneic, with a respiratory rate
of 35 breaths/minute, nasal flaring, and use of accessory muscles. Auscultation
of the lung fields reveals greatly diminished breath sounds. Based on these
findings, what action should the nurse take to initiate care of the client:
a- Initiate oxygen therapy and reassess the client in 10 minutes.
b- Draw blood for an ABG analysis and send the client for a chest x-ray.
c- Encourage the client to relax and breathe slowly through the mouth
d- Administer bronchodilators

391- Which of the following is the best breathing pattern you should teach a client
with COPD:
a- Pursed-lip breathing c- Abdominal breathing
b- Deep breathing d- Slow lite breathing

392- AA nurse instructs a female client to use the pursed-lip method of breathing and
the client asks the nurse about the purpose of this type of breathing. The nurse
responds, knowing that the primary purpose of pursed-lip breathing is to:
a- Promote oxygen intake
b- Strengthen the diaphragm
c- Strengthen the intercostal muscles
d- Promote carbon dioxide elimination

393- An infant with tetralogy of Fallot is experiencing an attack involving cyanosis


and dyspnea. Which position should the infant be placed in:
a- Fowler's c- Trendelenburg's
b- Knee-chest d- Prone

108
394-A A client is unresponsive and has been brought to the emergency department.
Initial laboratory results reveal:
Serum K+ 3.6 mmol/L
Glucose 26 mg/dl
Hemoglobin 12. 6 gm/dl
Carbon dioxide 26. 2 mmol/L. The nurse will anticipate:
a- Dextrose by mouth
b- 50% dextrose IV
c- 1 unit of packed cells
d- 10 mEq of KCl over 0.5 hour

395- When a client has a nephrostomy tube, the priority nursing care is to:
a- Ensure drainage of urine
b- “Milk” the tube every 2 hours
c- Keep an accurate record of intake and output
d- Instill 2 ml of normal saline solution every shift

396- Which of the following is the most common symptom of myocardial infarction
(MI):
a- Chest pain c- Edema
b- Dyspnea d- Palpitations

397-AA client has surgery for a perforated appendix with localized


peritonitis.In which position should the nurse place the client:
a- Sims position
b- Trendelenburg
c- Semi-fowlers
d- Dorsal recumbent

109
398- Which of the following position should the client with appendicitis assume to
relieve pain:
a- Prone
b- Sitting
c- Supine
d- Lying with legs drawn up

399-AA women breast feed her infant for one or two hours and her infant cries most
of the time and she feels pain in her breast, which of the following instructions
are appropriate for the nurse to give the mother:
a- Regulate breast feeding every 3 hours
b- That’s normal feeding problem
c- Shift to bottle feeding
d- Start weaning your baby

400- While caring for a child with a ventriculoperitoneal shunt revision, the nurse
finds the patient lying with the head and feet flexed back. The nurse should
call for help and prepare for a (n):
a- Spinal tap
b- Shunt culture
c- Electrocardiogram
d- Ventricular tap

401- While assessing a child with pulmonary stenosis, the nurse should give
PRIORITY to:
a- Deep tendon reflexes
b- Urinary output
c- Exercise tolerance
d- Pattern of food intake
110
402-AA 25 years-old male patient suffered a spinal cord injury at the T-4 level andis
being cared for in hospital. The nurse enters the patient’s room and finds the
patient sitting upright and looking anxious and restless. He complains of
sudden headache and nausea. Sweat forms on his forehead yet his feet are cool
to touch
Blood pressure 150/100 mmhg
Heart rate 55/min
Respiratory rate 28/min
Temperature 37.1C
What nursing intervention is initially most appropriate?
a- Assess for a full bladder
b- Lower the head of the bed 30 degrees
c- Loosen clothing and bed sheets
d- Apply heating pad to lower extremities

403- An 80 year-old man presents to the hospital with chronic fatigue, dyspepsia
and constipation. On examination he is jaundiced, has red palms, dilated veins
around the umbilicus, the abdomen is very distended and he has black, tarry
stool on a rectal exam. He is noted to be lethargic and have a flat tone. A
paracentesis reveals clear colored fluid with low protein content.
What is most likely diagnosis?
a- Gilbert’s syndrome
b- Thalassemia
c- Sickle cell crisis
d- Cirrhosis

111
404-
Admission Report at 22:45 (10:45p.m.) Diagnosis
14 years old admitted Remains febrile, maximum temperature Bacterial
18 hours ago this shift 39.3C pneumonia
Respiration rate 12-14 per minute
Heart rate 100-180 per minute
Blood pressure level within baseline
normal range
Next dose of intravenous anti-infective is
due at 08:00 (8:00 a.m.)
Refer to the accompanying figure.
The nurse should plan to monitor which of the following at least every 4
hours?
a- Vital signs c- Food intake
b- Urinary output d- Level of activity

405- Which instruction take priority in reducing anxiety related to surgery?


a- Surgical procedure and postoperative exercises
b- Risk of infection after surgery
c- Advanced directive and what it means
d- Pre-operative laboratory result and what to expect on it

406- When caring for a patient with an ostomy, the nurse knows that extra
protection for peristomal skin is MOST important for those patients with a
(an):
a- Ileostomy
b- Ascending colostomy
c- Transverse colostomy
d- Sigmoid colostomy

112
407- Which of the following nursing diagnosis takes PRIORITY for a patient
admitted to the critical care unit with diabetic ketoacidosis?
a- Deficient fluid volume secondary to hyperglycemia
b- Risk of infection secondary to weakened immune system
c- Deficient knowledge of cause and prevention
d- Imbalanced nutrition related to hyperglycemic state

408-AA 69 year-old man is admitted to the intensive care unit following cardiac
surgery. Two hours after admission, the nurse performs a routine assessment
and notes the patient’s chest tube drainage is 200 milliliters and a dark red
color. He has had 60 milliliters output from the indwelling urinary catheter
Blood pressure 138/68 mmhg
Heart rate 76/min
Respiratory rate 16/min
Body temperature 37.0C oral
Oxygen saturation 94% 6L/min nasal cannula
Which finding should be reported to the doctor?
a- Dark red chest tube drainage
b- Urinary output
c- Oxygen saturation
d- Chest tube output volume

409- To minimize a toddler from scratching and picking at healing skin graft, the
nurse should utilize:
a- Mild sedatives c- Punishment for picking
b- Hand mittens d- Distractions

113
410- The following pain medications are ordered for a patient who had a right leg
amputation: Oxycodone 5 mg every 4 hours as needed and morphine 5 mg
every 4 hours as needed. The nurse administered oxycodone 2 hours ago, but
the patient reports pain rated 8 on a scale of 0 (no pain) to 10 (severe pain) as
the dressing change begins
After evaluating the effectiveness of the pain medication, what action should
the nurse take?
Blood pressure 169/98 mmhg
Heart rate 112/min
Respiratory rate 22/min
Temperature 36.7C
a- Administer additional oxycodone 5 mg
b- Administer morphine 5 mg
c- Change the dressing quickly
d- Ask the patient to wait 2 hours

411-AA 55 year-old man presented to the clinic with complaints of numbness and
tingling in his feet for the past 3 months. He appears thin and his skin is pale.
Examination confirms the loss of vibration sense. He also has altered
proprioception. He had a subtotal gastrectomy two years ago (see lab results)
Test result normal values
MCH 2.72 1.45-2.01 fmol/cell
HCT 0.31 0.41-0.50
MCV 105 81-97 fl
Reticulocyte count 1.5 0.5%-2.5%
Which supplement would most likely improve his condition?
a- Iron c- Folate
b- B12 d- Calcium
114
412-AA 35 year-old man is hospitalized following a blunt chest injury. He has
chest pain, breathing difficulty and asymmetrical lung expansion. Chest
auscultation reveals decreased lung sounds on the left side. The jugular veins
are distended and there is tracheal deviation to the right.
Blood pressure 92/54 mmhg
Heart rate 120/min
Respiratory rate 24/min
Temperature 36.2C
SaO2 88% on oxygen
Which initial intervention is most appropriate?
a- Send patient for X-ray examination
b- Administer bronchodilators
c- Administer intravenous fluid bolus
d- Prepare for needle decompression

413-AA patient is being followed in the clinic for hypertension, adult onset
diabetes, and obesity. The patient is apathetic about learning about
nutritional guidelines to reach the goals of weight loss and consumption of a
healthy diet. The patient admitted to eating “whatever is put in front of me”
Which of the following actions would the nurse take?
a- Collaborate with the patient to set goals
b- Add a nursing diagnosis of non-compliance
c- Refer for psychiatric screening for depression
d- Discuss nutritional interventions with the spouse

115
414- The nurse is measuring the chest tube drainage of a patient who had open
heart surgery 4 hours ago.
Which of the following is the MAXIMUM hourly amount of chest tube
drainage is expected in this timeframe?
a- 100 ml
b- 200 ml
c- 300 ml
d- 400 ml

415-A A nurse is caring for a three year-old child with a fractured arm.
Which of the following interventions is the MOST appropriate for pain
management?
a- Administer analgesics when necessary
b- Assess pain once a shift
c- Anticipate pain and intervene early
d- Encourage the use of self-quieting techniques

416- After a hearing restoration operation, a patient has no signs of complications


and soon recovers.
Which of the following is an expected outcome 5 days after the hearing
restoration surgery?
a- Regain full hearing
b- Minimal facial nerve paralysis
c- Minimal urinary incontinence
d- Ambulates without difficulty

116
Azyma142@hotmail.com ©0560903480
417-
Goals- for the next three months, the patient will:
A Not have any migraine headaches
B Decrease the number of migraine headache
C Learn to tolerate her migraine headaches
D Continue to take prescribed preventative medications daily
E Initiate complementary pain management strategies as needed
F Wean self from all pain medications
G Keep a pain diary and recognize triggers and auras before migraine headaches commence
An adolescent patient with a history of migraine headaches for the past
year has been followed for pain management. The headaches have
decreased in severity and now occur only occasionally
What are the BEST three long term goals?
a- A, D, F
b- B, D, F
c- C, E, F
d- D, E, G

418-AA community health nurse visits a patient who had a cerebrovascular


accident. The patient is at risk for deficient fluid volume due to voluntary
reduction of fluid intake to avoid the use of the bathroom. The nurse educates
the patient on the importance of drinking fluids and maintaining hydration/
Which of the following indicates the efficacy of the nursing intervention?
a- Amber color urine
b- Respiration of 35
c- Tachycardia
d- Moist mucous membrane

419-AA patient with diabetes mellitus and multiple sclerosis has been prescribed
baclofen (Lioresal). The nurse knows this medication may result in?
a- Increased insulin needs c- Optic neuritis
b- Renal failure d- Muscle tremors
117
420-
Nursing diagnosis
A Impaired physical mobility
B Activity intolerance
C Alternation of comfort
D Risk of alternation in skin integrity
E Knowledge deficit
F Decreased cardiac output
A patient with Alzheimer’s disease and severe cardiomyopathy presents to the
hospital with a fractured left hip. The patient is on bed rest until a cardiologist
clears the patient to have surgery.
Which nursing diagnosis have the highest priority?
a- E and F
b- D and C
c- C and F
d- D and F

421- AA 19 year-old woman telephones the nurse and complains of difficulty of


breathing. The symptoms developed while exercising that morning. She has a
productive cough with thick mucus secretions and wheezing. The nurse hears
that the breathing problems do not interfere with talking on the phone. The
woman plans to attend the clinic but must wait for two hours before she can
be driven there.
Which treatment should be recommended first by the physician?
a- Purse-lip breathing
b- Deep breathing and coughing
c- Decrease physical activity
d- B2 agonist administration

118
422- AA patient is recovering from surgery using spinal anesthesia.
The patient developed a spinal headache.
Which of the following nursing actions would be MOST appropriate?
a- Elevate the head of the bed 30 degrees
b- Keep the patient well hydrated
c- Limit intake of salty foods
d- Lower the temperature of the room

423- AA patient is preparing for a total knee replacement. During the


preoperative interview process the patient reports an allergic reaction to
penicillin.
Which of the following is considered a side effect and not a true allergy to
medication?
a- Shortness of breath
b- Tingling lips and tongue
c- Rash
d- Upset stomach

424- Prior to providing care for a hospitalized infant, the nurse who focuses on
preventive measures must:
a- Introduce self to parents
b- Perform hand hygiene
c- Have a witness present
d- Assess the child’s developmental level

425- As identified by Dr. Elizabeth Kubler-Ross, which stage of dying is


characterized by the transition from, “NO, not me” to “Yes, me, but…”
a- Anger c- Acceptance
b- Depression d- Bargaining
119
426- A 50 year-old male presents to the medical office 3 weeks after cardiac surgery
with complaints of a feeling of weakness, difficulty breathing, and joint pains.
Upon examination the nurse finds a fever and a friction rub on auscultation of
the chest. The nurse recognizes that the MOST likely surgical complication is:
a- Neuropsychological dysfunction
b- Postpericardiotomy syndrome
c- Cardiac tamponade
d- Phrenic nerve damage

427- AA patient with heart failure has the following vital signs, which of these vital
signs should be reported to the physician prior to administrating the next dose
of digoxin?
Blood pressure 136/84 mmhg
Heart rate 48/min
Respiratory rate 20/min
Temperature 37.1C
a- Blood pressure
b- Pulse
c- Temperature
d- Respiratory rate

428- The floating ribs that are not attached with sternum are:
a- Ribs 7&8
b- Ribs 9&10
c- Ribs 11&12
d- Ribs 9&10&11&12

120
429- AA pediatric nurse is providing medication instructions to the mother of a 13
year-old boy who is starting anti-depressant therapy. The mother appears
unfocused, agitated and confused and asks the nurse to repeat the instructions
several times. She tells the nurse that she is concerned that she might
administer the medication incorrectly. The nurse is concerned that the mother
is not following the instructions.
Which intervention would be most appropriate?
a- Reassure her that doubts are normal
b- Reinforce the importance of correct dosage
c- Refocus the teaching with printed material
d- Take a small break and the continue

430- AA child is treated for superficial (first-degree) thermal burns to the thigh. The
child is in great discomfort and does not eat.
Which of the following diagnosis should receive PRIORITY?
a- Altered nutrition
b- Impaired skin integrity
c- Risk of infection
d- Acute pain

431- AA 3-week-old infant is hospitalized with jaundice. When considering the fluid
needs relative to body size of the infant as compared to the fluid needs of an
adult. The nurse knows this infant requires:
a- Less fluids
b- More fluids
c- Same amount of fluids
d- Much less fluids

121
432- Which of the following statement is most accurate?

a- Girls have more ADHD than boys


b- Boys are more frequently diagnosed with chronic illnesses
c- Boys are less frequently diagnosed with chronic illnesses
d- Girls have more other chronic illness than ADHD

433- What is the percentage of boy with attention deficit hyperactivity disorder?

a- 12%
b- 16%
c- 8%
d- 4%

122
434- The nurse is observing a detailed neurological assessment on a client with a
suspected brain tumor. When performing the Romberg test, the client sways
when the eyes are both open and closed. What does this indicate?
a- The problem is probably in the cerebellum
b- It is a position sense abnormality
c- This is not an abnormal test result
d- The client has lost proprioception

435- An 18 year-old woman who broke her right ankle is seen in the physician’s
office one week after the cast was removed.
Which of the following is the short term goal for this patient?
a- Walk 100 feet with crutches
b- Walk completely independent
c- Relieve the pain
d- Strict bedrest

436- AA client with multiple sclerosis has been prescribed the drug baclofen
(Lioresal). What is the action of this drug?
a- Reduces spasticity
b- Skeletal muscle relaxation
c- Immune suppression
d- Prevents viral infections

437- What is the average incubation period of Hepatitis A?


a- 30 days
b- 60 days
c- 50 days
d- 14 days
123
438- After the health care team meets to discuss the client’s nursing needs, the
nursing diagnosis “Disturbed body image” is added to the care plan. The best
rationale for adding this nursing diagnosis to the care plan in the case of a
female is that females with Cushing’s syndrome typically experience which
physiologic effect?
a- Masculine characteristics
b- Heavy menstrual flow
c- Extreme weight loss
d- Large, pendulous breasts

439- AA 64-year-old client with uterine cancer is scheduled to undergo an abdominal


hysterectomy under general anesthesia. Before the client returns from the
postanesthesia care unit, the registered nurse asks the licensed practical nurse
to help revise the care plan for the client who has undergone a hysterectomy.
Which nursing diagnosis is most appropriate for the nurse to add to the
client’s care plan at this time?
a- Risk for ineffective airway clearance
b- Risk for imbalanced nutrition
c- Ineffective coping
d- Impaired verbal communication

440- The nurse initiates a teaching plan for the client with Parkinson’s disease.
Which instruction should be the nurse’s priority in this situation?
a- Steps to enhance the client’s immune system
b- Importance of maintaining a balanced diet
c- Need to remove all safety hazards
d- Importance of social interactions

124
441- The charge nurse enters the nursing diagnosis “Risk for ineffective airway
clearance related to an inability to swallow” on the client’s care plan. Which
nursing intervention is most appropriate for managing the identified
problem?
a- Keeping the client supine
b- Removing all head pillows
c- Performing oral suctioning
d- Providing frequent oral hygiene

442- The client has returned from surgery with a leg cast, and the nurse is assisting
the client back to bed. Which of the following would the nurse identify as the
highest priority when documenting the postoperative circulation status of the
recently casted extremity?
a- Adequate neurovascular functioning
b- Minimal pain on movement
c- Vital signs within normal limits
d- No drainage noted on the case

443- After X-rays are taken of the head, neck, and spine, the client is diagnosed
with a head injury and admitted for inpatient care. When assessing the client
with a head injury, which of the following should receive priority attention?
a- Lung sounds
b- Clarity of speech
c- Mobility of fingers
d- Pupillary responses

125
444- The client experiencing a severe allergic reaction becomes pulseless. The nurse
shakes the client, shout the client’s name but gets no response, and activates
the emergency medical response system. Which nursing action becomes the
next priority?
a- Administer a single blow to the sternum
b- Give two quick breaths that make the chest visibly rise
c- Begin chest compressions at a rate of 100 per minute
d- Administer an epinephrine (Adrenalin) injection

445- AA 50-year-old client is scheduled for a heart transplant tomorrow. The night
nurse is asked to review the surgical procedure with the client. Becauseof the
client’s anxiety, the client has difficulty comprehending the nurse’s
information. During the postoperative period, what is the best rationale for
the nurse frequently assessing the client’s fluid status?
a- Urine retention is common after a heart transplant
b- Urine output is an indication of perfusion to the kidneys
c- Hydration determines when the client needs to be transfused
d- Hydration indicates when fluids should be increased

446-A A nurse is assigned to care for a patient with a diagnosis of thrombotic stroke.
The nurse knows that this type of stroke is MOST LIKELY caused by:
a- Blockage of large vessels as a result of atherosclerosis
b- Emboli produced from valvular heart disease
c- Decreased cerebral blood flow due to circulatory failure
d- A temporary disruption in oxygenation of the brain

126
447- AA nurse obtains a urine dipstick analysis sample from a 35 year-old woman
who reports having burning sensation with urination and a sense of urgency
and frequency. She had been diagnosed with the condition six months
previously and was prescribed a course of antibiotics.
Urinalysis Results Normal values
Colour Dark yellow Straw-coloured
Odour Abnormal Almost nothing
Appearance Turbid Clear
Leukocyte esterase Positive Negative
Nitrites Positive Negative
Which type of pharmacological treatment is most likely?
a- Anti-viral
b- Anti-fungal
c- Anti-bacterial
d- Anti-parasitic

448- AA 30 year-old woman has been prescribe albuterol PRN and prophylactic
inhaled corticosteroids to be taken once per day. She has been taking the
prophylactic as prescribed but has needed to use albuterol more often than
usually. She has a chronic cough and often has air hunger.
Which intervention is initially most appropriate?
a- Refer for a chest X-ray
b- Administer magnesium sulfate
c- Assess peak flow measure
d- Perform arterial blood gas sampling

449- Which of the following drugs is not used in the treatment of pulmonary
embolism?
a- Heparin c- Digoxin
b- Warfarin d- Streptokinase

127
450- AA home care patient with chronic obstructive pulmonary disease (COPD)
reports an upset stomach. The patient is taking theophylline (Theo-Dur) and
triamcinolone acetonide (Azmacort). The nurse should instruct the patient to
take:
a- Theo-Dur on an empty stomach
b- Theo-Dur and Azmacort at same time
c- Theo-Dur and Azmacort 12 hours apart
d- Theo-Dur with milk or crackers

451- AA 65 year-old woman presented to her care provider with complaints of


bright red blood in the stool, a loss of appetite, a feeling of fullness and fatigue.
She had lost 5 kilograms in the past three weeks without dieting. A faecal
occult blood test is positive and the patient is scheduled for additional
screening test. Which screening test is most likely for this patient?
a- Barium enema
b- Colonoscopy
c- Endoscopy
d- Computed tomography scan

452- AA baby girl was born prematurely at 33 weeks gestation due to placenta
abruption. She is now two months old and has a potential hearing deficit. In
the neonatal intensive care unit (NICU) she had passed the initial screening
tests for hearing but has significant risk factors that suggest further
evaluation.
Which autonomic reflex should be further evaluated?
a- Moro c- Pupillary
b- Babinski d- Tonic neck

128
453- AA 45 year-old woman presented with a generalized rash that is not itchy. She
reports that she has had the problem for the past 15 years. Examination
reveals a well-outlined, reddish plaque over the right gluteal fold. The plaque
has scales over it and is cracked in some areas.
Which intervention is initially appropriate?
a- Apply topical cream to the affected area
b- Expose area to sunlight for twenty minutes daily
c- Maintain immunosuppressant therapy regimen
d- Increase dietary intake of vitamin A

454- The patient is receiving mechanical ventilation set at fraction of inspired


oxygen (FiO2) 100%.
The nurse should understand that which of the following can improve this
patient’s oxygenation?
a- Adding positive end expiratory pressure (PEEP)
b- Place the patient in trendelenburg position
c- Increasing the FiO2
d- Suctioning the patient hourly

455-A A nurse is caring for a 3-week-old infant who just admitted to the hospital.
Which of the following nursing interventions does NOT support this infant’s
basic emotional and social needs?
a- Provide for continual contact between parents and infant
b- Actively involve parents in caring for the infant
c- Keep the infant’s environment quite, dim and free of sensory stimulation
d- Foster infant-sibling relationships as appropriate

129
456- AA community health nurse is teaching a health class about infectious
diseases process.
The nurse instructs the class that rabies would be considered which of the
following type of infection?
a- Viral
b- Protozoan
c- Fungal
d- Bacterial

457- AA couple who wants to conceive but has been unsuccessful during the last 2
years has undergone many diagnostic procedures. When discussing the
situation with the nurse, one partner states, “We know several friends in our
age group and all of them have their own child already, why can’t we have
one?” Which of the following would be the most pertinent nursing diagnosis
for this couple?
a- Fear related to the unknown
b- Pain related to numerous procedures
c- Ineffective family coping related to infertility
d- Self-esteem disturbance related to infertility

458- AA 64 year old male client with a long history of cardiovascular problem
including hypertension and angina is to be scheduled for cardiac
catheterization. During pre-cardiac catheterization teaching, the nurse should
inform the client that the primary purpose of the procedure is:
a- To determine the existence of CHD
b- To visualize the disease process in the coronary arteries
c- To obtain the heart chambers pressure
d- To measure oxygen content of different heart chambers

130
459- AA gravida 3 para 2 is admitted to the labor unit. Vaginal exam reveals that
the client's cervix is 4cm dilated. The patient complains of pain which she
stated to be 7/10.at this time and until a full cervix dilatation is achieved, what
is the priority nursing goal for such patient at this time is:
a- Pain management
b- Prevent fetal distress
c- Preparing the patient for anesthesia
d- Keeping this patient NPO

460- AA client with iron deficiency anemia is scheduled for discharge. Which
instruction about prescribed ferrous gluconate therapy should the nurse
include in the teaching plan?
a- “Take the medication with an antacid.”
b- “Take the medication with a glass of milk.”
c- “Take the medication with cereal.”
d- “Take the medication on an empty stomach.”

461- After 4 hours of active labor, the nurse notes that the contractions of a
primigravida client are not strong enough to dilate the cervix. Which of the
following would the nurse anticipate doing?
a- Obtaining an order to begin IV oxytocin infusion
b- Administering a light sedative to allow the patient to rest for several hour
c- Preparing for a cesarean section for failure to progress
d- Increasing the encouragement to the patient when pushing begins

131
462- AA 24 year-old woman was prescribed loratidine (Claritin) 10 mg tablet
q 12 hours a.c. for allergy.
The nurse reviews the medication order and explains to the patient the
relation of this drug to meals is as following:
a- The drug is to be taken after meals
b- The drug is to be taken during meals
c- The drug is to be taken before meals
d- The drug is to be taken away from meals

463- An elderly client is experiencing an alteration in his equilibrium and


coordinated muscle movements. The nurse realizes that these functions are
controlled by which area of the nervous system?
a- Brain stem
b- Cerebrum
c- Diencephalon
d- Cerebellum

464- AA 32-year-old female is admitted for a hemorrhoidectomy. During the


nursing assessment, all of the following factors are elicited. Which one is most
likely to have contributed to the development of hemorrhoids?
a- The client states that she usually cleans herself from back to front after a bowel
movement
b- The client says her mother and grandmother had hemorrhoids
c- The client has had four pregnancies
d- The client eats bran every day

132
465- A client who has hepatitis A asks, “How could I have gotten this disease?”
what is the nurse’s best response?
a- You may have eaten contaminated food
b- You could have contracted the disease by using intravenous drugs
c- You must have received and infected blood
d- You could have contracted the disease by engaging in unprotected sex

466- AA client undergoes right mastectomy for carcinoma. When teaching


the client post-mastectomy exercises, it is important for the nurse to:
a- Exercise both arms simultaneously
b- Exercise the right arm only
c- Have the client wear a sling between exercise periods
d- Wait until the incision has healed

467- The following picture represents a newborn reflex known as?


a- Rooting reflex
b- Moro reflex
c- Grasping reflex
d- Startle reflex

468- The nurse is caring for a client who has had a right modified radical
mastectomy this morning. Which exercise should the nurse encourage the
client to perform this evening?
a- Hair combing exercises with the right arm
b- Wall climbing exercises with the right arm
c- Movement of the fingers and wrists of the right arm
d- Exercises of the left arm only

133
469- AA 38-year-old client who has mitral stenosis is hospitalized for a valve
replacement. Which condition is the client most likely to report having had
earlier in life?
a- Meningitis
b- Syphilis
c- Rheumatic fever
d- Rubella

470- An 82-year-old woman who has Alzheimer’s disease is admitted to the acute
care unit. She frequently gets out of bed and wanders in the hall, unable to
find her way back to her room. She even gets in the beds of other clients. What
nursing action is most appropriate for this client?
a- Restrain her so she will not wander in the halls
b- Ask her roommate to call the nurse whenever she leaves the room
c- Punish her when she gets in a bed other than her own
d- Put her favorite picture on the door to her room

471- AA 35-year-old man is admitted with severe renal colic. The nurse should
monitor this man for possible complications. Which of the following is a
complication of renal colic?
a- Anemia
b- Polyuria
c- Hypertension
d- Oliguria

134
472- AA nurse discusses high-risk complications with a group of women at a
prenatal clinic. Which client would the nurse identify as being at highest risk
for developing complications during pregnancy?
a- A 25-year-old gravida I client
b- A client with the placenta implanted on the fundus of the uterus
c- A client who has nausea and vomiting during the first trimester
d- A 30-year-old client with DM

473- AA 56 year-old man was brought to the emergency room by his relatives, on
examination he appears sick and has severe weakness. One of his relatives told
the nurse that the man eats nearly nothing and have been crying most of the
time for 3 months now since he lost his son in an accident. The nurse knows
that this patient is suffering from:
a- Mania
b- Insomnia
c- Depression
d- Schizophrenia

474- Which of the following drugs is used for the treatment of digitalis toxicity?
a- Protamine sulfate
b- Streptokinase
c- Theophylline
d- Digoxin immune Fab

135
475- A female client is admitted with a diagnosis of acute renal failure. She is
awake, alert, oriented, and complaining of severe back pain, nausea and
vomiting and abdominal cramps.
Blood pressure 100/70 mm Hg
Pulse 110
Respiratory rate 30
Temperature 38°C oral
Sodium 120 mEq/L
Potassium 5.2mEq/L
Urinary output for the first 8 hours is 50 ml
The client is displaying signs of which electrolyte imbalance?
a- Hyponatremia
b- Hyperkalemia
c- Hyperphosphatemia
d- Hypercalcemia

136
Some important notes to remember
➢ Renal failure – acute: low protein, high carbohydrate, low sodium (oliguric phase),
high protein, high calorie, restricted fluid (diuretic phase)

➢ Renal failure – chronic: low protein, low sodium, low potassium

➢ Normal color of stoma (colonostomy): pink or red

➢ Teething starts at: 7 months and till 16 months

➢ After Laparoscopic cholecystectomy: patient regain normal activity after 12-14 days

➢ After renal surgery the most common complications: DVT

➢ Typhoid: transmission (contact) – isolation (contact isolation)

➢ Food rich in vitamin C: orange, Broccoli, strawberries, tomato

➢ MRSA isolation: contact

➢ Complication of hemorrhoidectomy: infection and thrombosis

➢ HCV transmitted by: blood

➢ Test to assess anemia patient: ferritin and hemoglobin

➢ Psoriasis takes the shape of: scales

137
➢ Normal capillary refill: less than 3 seconds

➢ Child has limb swelling and ascites, the main cause of swelling: Increased
hydrostatic pressure

➢ Sign of malnutrition for child: open fontanel, low weight for his age

138

You might also like